1. Trang chủ
  2. » Văn Hóa - Nghệ Thuật

Tạp chí Epsilon số 14

240 79 0

Đang tải... (xem toàn văn)

Tài liệu hạn chế xem trước, để xem đầy đủ mời bạn chọn Tải xuống

THÔNG TIN TÀI LIỆU

Nội dung

V ậy, chúng ta cũng sẽ hình dung chuyển động một vật trên bầu trời hay trong hạt nhân nguyên tử như một điểm, và bằng một cách nào đó, nó tương ứng “ qua l ạ i ” với các tọa độ trên màn [r]

(1)

MỘT SỐ BÀI TOÁN TRÊN TÂM ĐƯỜNG TRỊN EULER

TỐN HỌC VÀ ẢO THUẬT Nguyễn Hùng Sơn

NHỮNG CƠ HỘI MỞ RA SAU NGÀY HỘI TOÁN HỌC MỞ

VÀ CÁC CHUYÊN MỤC KHÁC

“Xin nhấn mạnh: xem thống kê học cách suy nghĩ BẢY TRỤ CỘT THÔNG THÁI CỦA

THỐNG KÊ HỌC

Nguyễn Văn T uấn

Khơng phải thực tế mà phép toán thứ sinh số và cho chúng tính chất!”

SỐ VÀ LỊCH SỬ PHÁT TRIỂN LOÀI NGƯỜI – Nguyễn Lê Anh

(2)

BIÊN TẬP VIÊN:

LÊ VIẾT ÂN

LÊ PHÚC LỮ

NGUYỄN TẤT THU

VÕ QUỐC BÁ CẨN

TRẦN QUANG HÙNG

NGUYỄN

VĂN HUYỆN

ĐẶNG NGUYỄN ĐỨC

TIẾN

TRẦN NAM DŨNG

(3)

Bạn có tin vào chuyện cổtích?

Cáchđây năm, chúng tơi, với nhữngđộc giả đầu tiênđã bắtđầu câu chuyện cổtích việc cho rađời số tạp chí Epsilon, tạp chí với mục tiêu vơ tư, phát hành online miễn phí, phục vụ người u thích tốn học Một tạp chí xây dựng cộng đồng cho cộng đồng Câu chuyện cổtích thử nghiệm lan truyềnđi, đông đảo bạn đọc đón nhận, chia sẻ, chung sức xây dựng Câu chuyện chúng tơi có kết thúc rấtđẹp: dừng lại ởcon số13, Epsilon đangở độ chín Chúng tơiđã dừng lại tựhào,đểtừ mở đường cho người anh em khác, tạp chí Pi Chúng tơi dừng lại, phần nàođó cá nhân chúng tơi luyến tiếc vềnhững ngày thángđầy thơmộng Epsilon Rồi thời gian dần trôi, Pi ngày vững mạnh, tờbáo quen thuộc với người yêu toán Dẫu vậy, lịng bạn u tốn nhớvềEpsilon họln hỏi chúng tơi liệu Epsilon có ngày trởlại?

Vào ngàyđẹp trời mùa hè 2018,được thúc giấc mơ đẹp, tổng biên tập Trần Nam Dũng quyếtđịnhđểngười em Epsilon tái xuất, cùngđồng hành với anh Pi Vậy tất cảchúng tơi tái hợp

Ban biên tập Epsilon ngàyđầu tiên phần lớn vốn cậu trai trẻ, độc thân có nhiều thời gian, đềuđã có giađình riêng,đã biết thếnào vất vảmưu sinh sống Nhuệkhí tuổi trẻ bắtđầu chuyển thành kinh nghiệm ngườiđàn ông trải Mặc dù vậy, nghe lời hiệu triệu tổng biên tập, tất cảchúng không chútđắnđo, trởlại Chúng lại cốgắng liên lạc với từngđồng nghiệpđểcó viết tốt nhất, lại làm việc thâuđêmđểcóđược biên tậpđúng hạn chất lượng Tất cảcóđược chúng tơi tin vào nhau, tin vào nhữngđiều kỳdiệu câu chuyện cổtích

Epsilon trởlại giữvững triết lýcủa mình: phục vụcho cộngđồng u thích tốn, đặc biệtđề cao tính "khả đọc" viết Epsilon ln ln miễn phí, ln ln phát hành online tương lai chúng tơi sẽtừ từchuyển hồn tồn sang 100% online: xuất theo kiểu tập tinđịnh dạng pdf, tương lai sẽsửdụng sang nhữngđịnh dạng khác, giúp cho tính khả đọc khơng với nội dung mà cịn với hình thức trình bày Epsilon sẽcốgắng mởrộng hơnđối tượng tác giả, sẽsửdụng hệthống bình duyệt người có chun mơn Chúng sẽtập trung cao vềchất lượng, vậy, chúng tơi chỉduy trì xuất sốmỗi năm, vào ngày 13 tháng 13 tháng 12

Epsilon 14 giữbản sắc với chuyên mục quen thuộc nhưbài toán hay - lời giải đẹp, điểm sách, tốn học giải trí, lời giải bình luận thi, tất nhiên, thiếu câu chuyện tốn họcđẹp nhưcổtích

(4)

Nguyn Lê Anh

Sốvà lịch sửphát triển loài người 6 Vaselin Dimitrov

Định lýkhôngđiểm tổhợp - Combinatorial Nullstenllensatz 16 Nguyn Hùng Sơn

Toán học vàẢo thuật 24 Ban Biên tp Epsilon

Thếnào tưduy Logic - Hành trình tìm kiếm máy bay tích MiG-21U 29 Võ Nht Vinh

Giới thiệu toán tốiưu hai lớp 49 Nguyn Song Minh

Tính chất Phi Archimedean củaĐịnh giá P-Adic 55 Võ Quc Bá Cn

Phương pháp thêm biến giải phương trình hàm 66 Ngô Văn Thái

Sáng tạo - Làm chặt 79 Nguyn Văn Tun

Bảy trụcột thông thái thống kê học 93 Trn Quang Hùng

Một sốbài tốn tâmđường trịn Euler 98 Nguyn Trường Sơn

Điểm Humpty-Dumpty tam giác vàứng dụng 109 Lê Viết Ân

Một sốbổ đềhữu dụng tiếp cận lời giải tốn hình học 134 Nguyn Duy Liên

Bài toán hay - Lời giảiđẹp 164 Lê Phúc L

(5)

TrườngĐơng Tốn học miền Nam năm 2018 toán hay 214 Phm Hy Hưng

Những cơhội mởra sau ngày hội toán học mở 233 Trn Nam Dũng

(6)

SỐ

V

À

LỊCH

SỬ

PHÁT

TRI

N

LOÀI

NGƯỜ

I

Nguyễn Lê Anh

Tôi thường bắtđầu giảngđầu tiên cho sinh viên năm thứnhất việc thông báo tên sốquyđịnh Những quyđịnhđầu tiên sinh viên không cần phải xin phépđể vào khỏi lớp Vàđiều có hàmýcác sinh viên khơng nên chỉvì phảiđếnđúng giờmà phân tâm khiđiđường vơ tìnhđểbịxảy tai nạn giao thơng Tơi cịn nói nhiềuđiều nữa, tơi hỏi

- Cóđúng emđã thi vào trườngđại học cách trung thực không ? Gần nhưngay emđều khẳngđịnh

- Thưa thầy,đúngạ

Tôi nhận vẻkiêu hãnh tựhào khuôn mặt bạn trẻmới bước bướcđầu tiên vào giảngđườngĐại học Tơi im lặng nói

- Các em có cho phép tơiđược kiểm tra khơng ? - Vâng

- Có sốcác em cho tơi biết!3là khơng ?

Thếrồi cảlớpồn ào, chìm vào sựtĩnh lặng Tôi gọi vài sinh viênđểhọ đứng lên trảlời Các vị dõi theođọc viết cũngđang tựtìm lấy câu trảlời Thơi thìđủcác kiểu trả lời Tôi chờcho tới sinh viên tự nhận thấy tất cảcác câu trảlời bạn khơngđúng tựhiểu khơng có câu trảlời, tơi nói:

- Nếu chắn em thi vàođại học cách trung thực Giáo Dục không trung thực khiđứng làm người cầm cân nảy mực - họ khơng cóđủkiến thứcđểlàm việcấy - trườngđại học mà emđang học không trung thực tổchức thi Chúng ta sẽcùng với em làm cho rõ chuyện

(7)

số đếm mà khơng có Phải tới 20 nghìn năm trước CN, nhu cầu mà, xuất số0 Số 0là sốkhơng có gì, tức khơng phải số Đó sản phẩm nhân tạo phi tựnhiênđầu tiênđược loài người tạo Sựxuất số0đồng với thờiđiểm xuất văn hóa

Trong việc sử dụng số đếm “1 con bị hợp lý, nhưng 1=2 con bị vơ nghĩa” Việc sử dụng sốnhư 1=2cốc nước, 2=3đoạn đường hiểu khái niệm số thayđổi Các số bịbỏ điđặc tính “nguyên khiđếm” chuyển sang loại sốthơng báo lượng Người ta hình dung sốnhưtỷlệ độdài cácđoạn thẳng vớiđoạnđược coi là1-đoạn đơn vị.Đểthông báo sốnàođó người ta thơng báo phép dựng hìnhđểtạo Và thếlà hình họcƠ-cơ-lit rađời TheoƠ-cơ-lit số đượcđồng vớiđoạn thẳng có thểdựng rađược chỉnhữngđoạn thẳng dựng rađược mớiđược coi số Nhưvậyđường chéo hình vng có cạnh bằng1là sốp2(căn bậc hai 2)

Nếu giải thíchp3là sốmà bình phương lên bằng3là vơ nghĩa, có biết số đâu mà bình phương lên !! Câu giải thích “sốp3được làđoạn thẳng sinh từhaiđiểm cắt của

hai vịng trịn bán kính nhau, vịng trịn nàyđi qua tâm vịng vịng kia” thìđúng

p

3

A

B

Và tất nhiên từ định lýPitago có thểchứng minh bình phương bằng3:

Câu hỏi “Có thểdựng rađượcđoạn thẳng cóđộdài với! là diện tích hình trịn bán

kính đơn vịhay khơng?” tốn khó phải tới kỷ19 có câu trảlời Câu trảlời “Khơng thểdựng rađượcđoạn thẳng cóđộdài với diện tích hình trịn bán kínhđơn vị”.Điều nàyđồng nghĩa với việc số! diện tích hình trịn bán kínhđơn vịkhơng phải sốtheo quan niệm củaƠ-cơ-lít

Bằng phép dựng hình chia đoạn thẳng đơn vị làm10 phần Phần thứ gọi là0, phần tiếp theođược gọi là1, cứnhưthếcho tới9 Mỗi phần lại có thểchia ra10 phần cách gọi tên củađoạn nhỏlà tên kèm theo tên củađoạn mà nằm bên Đó cách gọi theo hệthập phân mà ngày mà dùngđểgọi tên số Nếu không chia làm10phần mà chỉchia thành2phần có cách gọi nhịphân số

Như thếsốthập phân 0:99999 : : :tuần hoàn mãi số9, kýhiệuđoạn cuối trình chia10phần mãi Nếu coi trình việcăn1cái bánh, cứmỗi lầnăn hết9=10 chỉcịn lại1=10, lạiăn hết9=10phần cịn lạiấy cho “sẽchẳng

(8)

đườngđi từAđến B, khiđãđiđếnđược B tất nhiên phải có lúcđi qua vịtrí làđiểm 9=10 đoạn đường, qua vịtrí làđiểm 9=10đoạn cịn lại, Sựtự mâu thuẫn lýdo hếtđoạnđường từAđến B, mà “ép buộc, lýgiải trìnhđi thứngơn ngữphi thực tế- ngơn ngữtốn học” Điều xảy nghiên cứu thếgiới Chúng ta sẽlàm quen với khái niệm vật vừa hạt lại vừa sóng cịn nhiềuđiều nghe nhưphi lýnữa

Những người cho trìnhđi từA tới B điđếnđược B sẽcơng nhận dãy0:9I 0:99I 0:999I : : :có giới hạn bằng1; họlà mơnđệcủa lýthuyết tốn học có giới hạn Dãy “có giới

hạn” kiểu nhưdãy0:9I 0:99I 0:999I : : :như ởtrên, dãy sinh Iasinđuổi Rùa Iasin người vôđịch vềchạyởthếvận hội cổ đại Tài chạy ơng ta bịmang ví với tài chạy Rùa Khi Iasin đuổi tới chỗcon Rùa Rùa lại tí, cứnhưthếIasin lại phảiđuổi tới chỗcon Rùađangở, cứnhưthếmà tạo dãy vơ hạn có tính chất “có giới hạn” Vậy người theo trường phái Acsimet cho Iasin đuổiđược kịp Rùa, người phản bác Acsimet khơng

Nói Acsimet thường người hay nhớ câu chuyện ông ta cở truồng chạy phố reo Ơrêca tìmđược phương pháp đo thểtích ngai vàng mà khơng phải nấu chảy Mọi người biết vềviệc Acsimet ngườiđầu tiên đưa quan niệm cáchđo diện tích hình trịn bán kínhđơn vị Ơng ta dựng hìnhđa giácđều nội ngoại tiếp hình trịn, nhờviệc có thểtínhđược diện tích cácđa giác nội ngoại tiếp mà Acsimet cóđược dãy số để đánh giá cận cận cho diện tích hình trịnđơn vị Trên thực tếAcsimet sửdụng phép dựng hình đểdựng hai đoạn thẳng 31071 31070, chứng minh 31071 < ! < 31070 Ông ấyđã làm việc từ 2250 năm trước, khiấy chưa có cách ghi số 0; 1; 2; 3; : : : Các số 31071 hay 31070 thứ mà ngày diễn đạt lại cách dựng hình Acsimet Nếu làm tiếp Acsimet nhận dãy số đánh giá cận cận diện tích hình trịn bán kính đơn vị Biên độ xác đánh giá sau tốt hơn, tiến dần tới0 Và, hiểu dãy cận cận “tiến tới!”, cận cận khơng bao giờbằng Vậy Acsimetđã coi như“tồn tại” thêm thứsốchỉcó thểdựng rađược dãy cận cận dưới, sau xác hơn, mà có thểkhơng dựng rađượcđúngđoạn thẳng cóđộdài theo kiểu Ơ-cơ-lit Nhờ vào Acsimet mà có thêm số, gọi chúng chúng sốthực Chúngđược sinh nhờq trình dựng hình theoƠ-cơ-lít lấy giới hạn theo kiểu Acsimet Nhờcó cơng cụgiới hạn với sốthực Ascimet mà chúng tađịnh nghĩa tích phânđạo hàm, thếrồi tìm phương trình vật lý, Bigbang, rađiện văn minh ngày

Lại nói vềq trình Iasinđuổi Rùa, hay làđi từ điểm A tớiđiểm B Những cho Iasin đuổiđược Rùa họtheo Acsimet tạo trường phái tốn học liên tục với cơng cụgiới hạn Họ tạo vật lýhọc, rồiđiện Vậy khơng cơng nhận q trìnhđiđược từA tới B, tức vềmặt logich Iasin khơng có cách nàođuổi Rùa, họlà mơnđệcủa trường phái tốn học có cấu trúc -đó tự động hóa,đó cơng nghệthơng tin, mạng máy tính, trí tuệnhân tạo, tức bao gồm toàn bộcái thếgiớiảo ngày

(9)

ánh sáng photon, hạtđiện tử, loại phản vật chất, hạt nhưhạt Quack nhỏhơn hạt nhân nguyên tử đến nhiều tỷlần

Ơ-cơ-litđưa cáchđểkhai sinh số, Acsimetđã thêm vàođó sốthực Sốâm đờiđểlàm cho phương trình aCx D b ln có nghiệm Người ta sửdụng!3đểchỉnghiệm phương trình5Cx D2 Vàđiều quan nằmởchỗ: nhà tốn họcđã chứng minh “Khi thêm sốâm vào hệthống sốthì khơng tạo bất kểmột mâu thuẫn nào như khi khơng có chúng” Vậy sửdụng sốâm hay không sửdụng sốâm quyền người Sửdụng sốâm sẽlàm cho q trình tính tốn trởnênđơn giản hơn.Điều tương tựnhưviệc sửdụng thếgiới Âmđểgiáo dục văn hóa làm người cáchđơn giản

Người Việt quan niệm cõi Âm có tácđộng quyếtđịnh lên duyên khiến sựviệc xảy Tín ngưỡng cịn cho cõi Âm sựtiếp tục sựsống, người phải chịu trách nhiệm vềhành vi cịn sống cõi Dương Thế Tơi ln thấy vẫnđược chởche nghĩ đến cõi Âm nơi mà Ba Mẹtôiđãđến với ông bà tổtiên Nếu chấp nhận cõi Âm không bịlợi dụng không gây tự mâu thuẫn quyếtđịnh “Dương Thế”, sự chấp nhận cõi Âm thuộc phạm trù văn hóa Nó làm cho sống có thêm nhữngđiều khơng thểlý giải logich được, làm cho sống người

Một sốbạn thường vấn vương câu hỏi “Vì trừcủa trừlại cộng - tức là!.!3/lại bằng 3?” Câu trảlời nằmởchỗcác bạnấyđã cốtình hình dung “trừ” nhưlà nợ, vàđểrồi nợcủa nợ thứ khó hiểu Có mang lướiđánh bắt cá kho màăn bao giờ! Số!3 công cụ đểthực phép tốn, khơng có tựnhiên, lưới tạo ra, khơng phải cá Vậy khơng nên cốgắng hình dung !3phải gì.Điều mà có thểlàm vá lưới, tức mong muốn công cụ sốcủa phải cóđược tính chất sau

1 - Có phép toánC "(cộng nhân), hai số có thểcộng với nhân với nhauđểcho kết quảlà số

2 - Thứnhất có số1(tức cáiđoạn thẳngđơn vịthừa hưởng từthời cốnội nghìnđời ông Ơcơlít)

3 - Tiếp theo số0(là mốcđẻra văn hóa văn minh mà chúng tađang tựhãnh diện đứng trước chó)

Tiếp theo tính chất phép cộng - phép giao hoánaCb DbCa:

5 - phép kết hợp.aCb/Cc DaC.bCc/: - phép cộng với0thìaC0Da:

(10)

Thếrồi lại theo tiênđề7 có3C.!3/D0: Tổng3C.!3/C.!.!3//có cách tính

Cách thứnhấtŒ3C.!3/"C.!.!3//D0C.!.!3//D!.!3/: Cách thứhai3CŒ.!3/C.!.!3//"D3C0D3:

Do tiênđề5 vềkết hợp tiênđề4 vềgiao hóan, tiênđề6 vềcộng với0chúng ta khẳngđịnh !.!3/D3:

Nhưvậy khơng phải “nợcủa nợlàđược”.Đặc tính!.!3/D3là hệlụy việc muốn có sốâmđểtiện tính tốn mà lại muốn cóđủcác tính chất thơng thường phép tốn Đối với phép nhân mong muốn cóđược tính chất sau

8 - phép giao hốna"b Db"a:

9 - phép kết hợp.a"b/"c Da".b"c/:

10 - quy tắc hỗn hợp.aCb/"c Da"cCb"c: 11 - phép nhân vi1thỡa"1Da:

12 - mi saÔ0thỡ cú mt s, kýhiu là1=a, màa"1=aD1:

Bây giờlà lúc trảlời câu hỏi “vì bình phương sốâm lại sốdương ? Ví dụ.!3/".!3/D9”

Theođịnh nghĩa!3là3C.!3/D0 Dođó

3"3C3".!3/D3".3C.!3// D3"0D0;

hay9C3".!3/D0 Mà theo tiênđề8 thì.!3/"3D3".!3/, nên9C.!3/"3D0: Mặt khác, ta có

.!3/"3C.!3/".!3/D.!3/"Œ3C.!3/"D.!3/"0D0: Nhựvậy,.!3/"3C.!3/".!3/D0và.!3/"3C9D0, suy ra.!3/".!3/D9:

Nhưvậy “Bình phương sốâm sốdương” khơng phải Trời sinh nhưthế Sốâm khơng có, chấp nhận cho sốâm vào hàng ngũcác sốthì có tính chất nhưvậy Nó hệlụy việc muốn tn thủcác tính chất phép tốn Vậy khơng phải thực tếmà phép toán thứ đẻra sốvà cho chúng tính chất! Số âm rađời đểcho phương trìnhaCx D b ln có nghiệm Số hữu tỷra đời làđể cho phng trỡnha"x Db (aÔ0) luụn cú nghim V sthc làđời làđểcho dãy hội tụthì “tiếnđến giới hạn” giới hạnấy số

Tuy nhiên đểcho phương trình bậc haiax2CbxCc D 0ln có nghiệm phải cho thêm vào hàng ngũcác sốmột loại sốnữa Chúng gọi sốphức Số phứcđược ký hiệu làaCi b chúng có thểthực phép tốn cộng nhân nhưsau :

(11)

Vớiđịnh nghĩa nhưvậy sốphức cóđủcác tính chất từ1 tới 12 nhưnêu Và với việc sử dụng chúng phương trìnhax2CbxCc D0ln có hai nghiệm

x1 D !

bCpb2!4ac

2a ; x1 D

!b!pb2!4ac

2a :

Người tađã chứng minhđược việc thêm sốphức vào chỉcó tiện cho tính tốn mà khơng làm nảy sinh mâu thuẫn nhưtrước thêm chúng vào Vậy có thểsửdụng hay khơng sửdụng sốphức Việc sửdụng sốphức sẽlàm cho việc nghiên cứu dễdàng hơn, sốphức hàm sốmũvà hàm lượng giác có chất cosxCi#sinx Deix, các hàm logarithm hàm lượng giác ngược biểu diễnđơn giản qua

Cũng nhưviệc biểu diễn sốthực trênđường thẳng, việc sửdụng sốphức có lợi có thểcoi mỗiđiểm mặt phẳng nhưlà biểu diễn sốphức.Ở phép nhân với số phức cos˛ Ci #sin˛ D ei ˛ phép quay mặt phẳng góc là˛ Nhưvậy số phức xuất vật lý ởnhững chỗnào có daođộng, hàm lượng giác hàm sốtheo góc, quay góc đơn giản nhân với số số sử dụng số phức Cơ học lượng tửkhẳngđịnh chuyểnđộng hạt nhỏsẽcó tính chất sóng Daođộng daođộng không gian, mà mỗiđiểm không gian chiều chúng ta, hạt chuyểnđộngđến sẽcó thêm chiều nơi chúng sẽdaođộng Vậy mỗiđiểm không gian, hàm biểu diễn trạng thái hạt sẽlà hàm sốphức theo thời gian

Theo dõi lịch sửphát triển sốchúng ta thấy sốcó xuất phátđiểm từcác nhu cầu thực tếnhưng vềsau xuất sựhồn mỹcủa cơng cụ tốn học Thật đáng tiếc mọiđa thứcđều có nghiệm trường sốphức Nhưvậy việc sửdụng phép toán cộng nhânđểmởrộng trường sốvớiđủ12 tính chất nhưtrênđã khơng cịn Muốn có thêm sốnữa phải giảm thiểu tính chất mà chúng phải thỏa mãn

Mọi học sinh nhớ hằngđẳng thức a2!b2 D aCb/.a!b/ Hẳn khơng bạn nhỏ thửsức không thểkhai triểnđược hệthứca2Cb2 Vào năm 1878, Cliffordđưa raýniệm vềviệc khai triểna2Cb2thành bình phương, kiểu nhưhằngđẳng thứcđáng nhớ Ơngấy muốn có khai triểna2 Cb2 D ˛#aCˇ#b/2 với avà b Điều chỉxảy khai triển

.˛#aCˇ#b/2 D.˛#aCˇ#b/.˛#aCˇ#b/D˛2#a2Cˇ2#b2C.˛ˇCˇ˛/ab; ˛2 D 1; ˇ2 D 1 và ˛ˇCˇ˛ D 0 Lẽdĩnhiên các˛; ˇ không thểlà số được, khơng thểcó sốmà˛ˇ D !ˇ˛ Khơng phải sốthì chảsao, sẽmởrộng chúngđểsử dụng với mộtđiều phảiđược chứng minh “các ông bạn không gây sựphiền tối nào như trước cơng nhận chúng!” Chúng ta gọi chúng Clifford Dirack hình dung phương trình vi phânđạo hàm riêng bậc hai

@2' @t2 !

@2' @x2 !

@2' @y2 !

@2' @z2 D0; ởdạng bình phương phương trình bậc

@2' @t2 !

@2' @x2 !

@2' @y2 !

@2' @z2 D

! ˛# @'

@t Cˇ# @' @x C# #

@' @y C$#

@' @z

(12)

Tức tìm “số”˛; ˇ;#;$vềthực chất ma trận chiều, thỏa mãn A2!B2!C2!D2 D.˛#ACˇ#B C##C C$#D/2: Khiấy phương trình có dạng

˛# @' @t Cˇ#

@' @x C# #

@' @y C$#

@' @z D0:

Do sốClifford hình dung ma trận Với cách hình dung nhưthếchúng ta mởrộng khái niệm sốra ma trận Các ma trậnđược coi số Chúng ta có thểcộng nhân ma trận Chúng ta có thểcóđược cácđa thức với hệsốlà ma trận, thếnữa định nghĩađược hàm sốvới biến sốmà ma trận Và do˛; ˇ;#;$ ma trận nên chiều thêm vào (phầnảo)được Dirack gọi phản vật chất Nhưthếphản vật chất sinh trình hình dung sốdưới dạng Clifford

Vậy chúng tađã cóđược cơng cụsố, Chúng ta kýhiệu sốthực làRvà sốphức làC Bỏ điđiều kiện giao hoán phép nhân có thểmởrộng trường số đểnó có thêm sốnữa vàđó quanternion Sốquanternion có dạngaCi bCjcCkd trongđói2 Dj2 Dk2 D!1; k Di#j vài#j D!j #i; i #k D!k#i; j #kD!k#j:Trong trường sốnày chỉtrừtính chất giao hốn phép nhân cịn 11 tính chất tích cịn lạiđược thỏa mãn Nhưthếtích số có tính kết hợp sốkhác0thìđều có ngược

Cũng giống nhưsốphức với mặt phẳng, phép quay không gian chiều tươngứng với số quanternion Như đểbiểu diễn trạng thái vật lý mà điểm liên quan tới chuyển đổi 3đại lương vật lý hàm trạng thái mộtđiểm không gian sẽlà hàm quanternion theo thời gian

Sốlà toàn bộtài sản mà người cóđượcđểnhận biết thếgiới Từcácđường thẳng sốchúng ta dựng không gian nhiều chiều hơn, mỗiđiểm bộcác số gọi tọađộ Những không gian tưởng tượng chúng ta, khơng có tựnhiên Nó cơng cụ đểchúng ta nhận thức Chúng ta ln có mong muốn nhận biết tựnhiên Chúng ta nhìn lên bầu trời tự hỏi “vạn vật từ đâu mà ra, đi đếnđâu?” Ngày quen với việc tìm đườngđi bảnđồtrênđiện thoại Chiếc hìnhđiện thoại khơng phải thếgới thực, chỉtươngứng thếgiới thực vào với nhận thức Vậy, sẽhình dung chuyểnđộng vật bầu trời hay hạt nhân nguyên tửnhưmộtđiểm, cách nàođó, tươngứng “qua lại” với tọađộtrên hìnhđiện thoại chiều chúng ta, tức vào không gian nhận thức tạo từcác trục số Tạm thời chưa nói tới việc làm đểcó tương ứng “qua lại” bắtđầu câu hỏi điểm sẽchuyển động nhưthế mô chiếcđiện thoại chiều, tức hệtọađộmà hình dung ? Do tốcđộviết chậm tốcđô tưduy nên hành văn bịlủng củng.Điều chỉnh sửa

Dành cho bạn muốn

đọc thêm

(13)

Sauđây trình bày phần tính tốn Archimedes (287-212) Các sốmà nhìn thấy tính tốn dướiđâyđã có 2250 năm tuổi Kýhiệu sốthì khác nhiều khiấy người ta chưa biết sửdụng kýhiệu sốtheo hệthập phân nhưcủa ngày Archimedes cho diện tích hình trịn nhỏhơn diện tíchđa giác ngoại tiếp chu vi vịng trịn lớn chu vi củađa giác nội tiếp Archimedesđã tính yếu tố cần thiết phục vụcho việc tính chu vi diện tích cácđa giácđều nội ngoại tiếp hình trịn dựa vào định-lýPitago vàđịnh-lý“đường phân giác chia cạnhđối thành hai phần tỷlệvớiđộ dài hai cạnh bên”

Trong tính tốn Archimedes sửdụng sốp3 lại khơng nói tới cách trực tiếp mà dựa vàoước lượng 265

153 < p

3 < 1351 780 :

A O B

F C

D I

E

Trong tam giác vng4AOC có∠AOC D30ı, AO

AC D

p

3 > 265 153: Phân giácOD chia cạnhAC theo tỷlệ AD

DC D

AO

CO, suy raAD D

AO CO AC C

AO AC

< AO 2C 265153 D 153

571AO:

Diện-tích hình trịn!AO2thì nhỏhơn diện-tíchđa giác ngoại-tiếp12#AD#AO, ! < 153"12

571 :

Hai tam giác4AIE và4BFE đồng dạng chúng có góc tươngứng nhau, nhưvậy AI

BF D

AE BE, mà

BF

AF D

p

3 < 1351 780 nên

AI AB >

780 301334:

Chu vi vịng trịn! #AB lớn chu viđa giác nội tiếp12#AI, vậy! > 780"12 301334 :

Quá trình tính tốn tương tựnhưvậy dựa việc liên tiếp chiađơi góc

(14)

780"12 301334 <

240"24 1838119 <

66"48 100916 <

66"96

201714 <! <

153"96 467312 <

153"48 233414 <

153"24 116218 <

153"12 571 :

Do310 71 <

66"96 201714 <

153"96 467312 <

1 nên3

10

71 <! < 7:

Sửdụng hàm sốlượng giác có thể“diễnđạt” lại thuật toán nhưsau

˛ ˛ tan˛

sin˛

˛ D !

3#2n!1

Thayx D !

3#2n!1 vào hằngđẳng thức ˆ ˆ < ˆ ˆ : tanx C

1 sinx D

1 tanx

2 tanx

2 #sinx D2#sin 2x

2

tađược 8

ˆ ˆ < ˆ ˆ : 3#2n!1#tan !

3"2n!1

C 3

#2n!1#sin ! 3"2n!1

D 3 #2n#tan !

3"2n 3#2n#tan !

3#2n #3#2 n!1

#sin ! 3#2n!1 D

#

3#2n#sin2x

$2 ThayanD3#2n!1#tan

!

3#2n!1; bnD3#2 n!1

#sin !

3#2n!1 vào cơng thức trên, ta có

< :

1 an C

1 bn D

2 anC1 anC1#bnDbn2C1 dùngđểtính giá trịan; bnvớia1D3

p

3; b1 D p

3 :

(15)

đa giác nội tiếp, việc tính tốn để ước lượng cận của! thơng qua chu-vi cho kết quảchính xác thơng qua diện tích ! Chi tiết cho hiểu thêm vềArchimedes, vào thời mà việc cộng-trừ-nhân-chia hai sốvới việc làm nhiều thời gian vất vả

Việc chứng minh! số hữu tỷlà J.H Lambert tìm vào năm 1768, tức khoảng 2000 năm sau Archimedes “tìmđược giá trịcủa số!” Chứng minh sauđây Ivan Niven1.

Ivan Niven sử dụng phép phản chứng Giảsử! D ab, vớiavàb sốnguyên dương Khi đó, tađặt

f x/D x

n.a!bx/n

nŠ D

n X kD0

an!k.!b/k x nCk kŠ.n!k/Š: Khi0!x!! thìf x/ < !

nan

nŠ nên0 < Z !

0

f x/sinxdx < 2! nan nŠ :

Do lim n!1

2!nan

nŠ D0nên0 < Z !

0

f x/sinxdx < 1vớinđủlớn ."/ Mặt khácđạo hàm nhiều lầnđa thứcf x/, ta thấy

ıKhi0!j !nthìf.j /.0/D0:

ıKhin!j DnCk !2nthìf.nCk/.0/ D nCk/Š nŠ.n!k/Ša

n!k

.!b/k sốnguyên Ta tính tích phân Z

!

0

f x/sinxdx nhờcông thức tích phân phần kết hợp với nhận xét đạo hàm nhiều hơn2nlầnđa thứcf x/có bậc2nthì sẽbằng0:

Z !

f x/sinxdx D n X jD0

.!1/jf.2j /.x/cosxj!0 D n X jD0

.!1/jC1hf.2j /.!/Cf.2j /.0/i:

Dof ! !x/Df x/nênf.2j /.!/Df.2j /.0/: NhưvậyZ

!

0

f x/sinxdx D2 n X jD0

.!1/jC1f.2j /.0/là sốnguyên với mọin: ""/ Khẳngđịnh."/và.""/mâu thuẫn với nhau, vậy! không thểlà sốhữu tỷ

(16)

ĐỊ

NH L

Ý

KHÔNG

Đ

I

M T

H

P

-COMBINATORIAL

NULLSTENLLENSATZ

Vaselin Dimitrov

(Trần Nam Dũng dịch giới thiệu)

G

IỚI THIỆU

Từngười dịch tiếng Nga. Thường kết quan trọng tốn học lại có thểtrình bày ngôn ngữmà học sinh (cho dù học sinh chun) có thểhiểuđược Phương pháp Combinatorial Nullstenllensatz,được tìm 10 năm trước (bài viết nàyđượcđăng năm 2005 – người dịch tiếng Việt) nhà toán học người Do Thái Noga Alon [Nora Alon, Combinatorial Nullstellensatz, Combinatorics, Probability and Computing, (1-2), 7-29.], ngoại lệ may mắn Chúng tôiđăng viết học sinh người Bulgaria Vaselin Dimitrov vềcácứng dụng kỹthuậtđơn giản mạnh vào toán tổhợp lýthuyết số

Tađưa vào sốkýhiệu cần thiết Sốphần tửcủa tập hợpSđược kýhiệu là|S| Ta ký hiệuZnlà nhóm lớp thặng dưtheo mơ-đunntheo phép cộng; KýhiệuFp trường

các lớp thặng dư theo mô-đun nguyên tốp(các lớp thặng dư thực sựtạo thành trường, chúng có thểcộng, nhân chia) NếuF trường thìF[x1, x2, , xn]ký

hiệu tập hợp cácđa thức biếnx1, x2, , xntrên trườngF Trong sốtrường hợp, ta

thực hiệu việc tính tốn cấu trúc tươngứng (nhóm, trường, vànhđa thức) mà khơng cần phải nói rõ cho trường hợp Ví dụhệsốnhịthứcCnkvốn sốnguyên dương ta xét nhưphần tửcủa trườngF màđa thức lấy hệsố Ví dụtrongF3

ta có(1 +x)3= + 3x+ 3x2+x3 = +x3bởi vì3 = 0.

Fedor Petrov

Dẫn nhập

Mục tiêu báo qua ví dụcác toán olympiad giới thiệu với bạnđọc kỹthuậtđại sốCombinatorial Nullstenllensatz,đượcđềxuất Noga Alon viết [1]

(17)

gọi Nullstenllensatz,được dịch làđịnh lýkhôngđiểm Một kết đơn giản lại công cụ mạnh nhiều toán tổhợp

Định lý1(Combinatorial Nullstenllensatz) GiảsửF - trường vàf ∈ F[x1, x2, , xn]- làđa

thức không đồng nhất có tổng bậc !ni=1mi, trong đó hệsố của xm11 xmnn khác Khi đó

với tập hợp S1, S2, , Sn ⊆ F với |Si| > mi,1 ≤ i ≤ n, tồn ci ∈ Si sao cho

f(c1, , cn)̸=

Đểchứng minhđịnh lýnày ta cầnđến hai bổ đề

Bổ đề1. Giả sử rằngf, như mộtđa thức theo biếnxi, có bậc ti,1 ≤ i ≤ n, giảsử rằng

Si ⊆ F,|Si| > ti Khiđó nếuf(x1, , xn)với bộ(x1, , xn) ∈ (S1 ×S2× ×Sn),

f ≡0

Chứng minh. Dễdàng chứng minh quy nạp theon

Bổ đề2. Giảsửgi ="s∈Si(xi−s),1≤ i≤ nlàđa thức theo biếnxi Nếuđa thứcf bằng0 tại mọiđiểm mà tọađộlà nghiệm cácđa thứcg1, , gn(tức làf(c1, , cn) = 0vớici ∈Si),

thì tồn cácđa thứch1, , hn ∈F[x1, , xn]sao cho

f = #

1≤i≤n

higi

trongđódeghi ≤degf−deggi với mọii

Chứng minh. Theođiều kiện bổ đề

f(x1, , xn) = 0với mọi(x1, , xn)∈(S1×S2× ×Sn)

Đặtti =|Si|−1 Với mọiita khai triểnđa thứcgi(xi)theo bậc củaxi:

gi(xi) =

$

s∈Si

(xi−s) =xtii+1−

#

1≤j≤ti gijxj

Khiđó vớixi ∈Si

xti+1

i =

#

1≤j≤ti

gijxj (1)

Gọif làđa thức thuđược từ f cách sau: ta viếtf tổng cácđơn thức thay cácxmi

i , trongđómi > ti, tổhợp tuyến tính bậc nhỏhơn củaxi, sửdụng

hệthức(1)

Với mọii∈{1,2, , n}đa thức thuđược có bậc theo biếnxikhơng vượt qti Chúýlàđa thức

f thuđược từf cách trừ cácđa thức có dạnggihi, trongđódeghi ≤ degf −deggi

Và f(x1, , xn) = f(x1, , xn)với (x1, , xn) ∈ (S1 ×S2 × ×Sn) Như

vậyf(x1, , xn) = 0với mọi(x1, , xn)∈(S1×S2× ×Sn) Theo bổ đề1,f ≡0 Suy

(18)

Chứng minhđịnh lý1.Không tính tổng quát, giảsửrằng|Si|=ti+ 1với mọii Giảsửrằng

kết luận củađịnh lýkhơngđúng Khiđóđa thứcf bằng0tại mọiđiểm mà tọađộlà nghiệm cácđa thứcg1, , gn Theo bổ đề2 tồn cácđa thứch1, , hnsao cho

f = #

1≤i≤n

higi

Theođiều kiệnđịnh lý, hệsốcủa"n i=1x

ti

i khác0 Nhưngdeghi ≤degf−deggi theo bổ đề2

nên bậc củađa thứchigi =hi"s∈Si(xi−s)khơng vượt qdegf, trongđó tất cảcácđa thức bậcdegf, có tronghigi,đều chia hết choxtii+1 Nhưvậy, hệsốcủa

"n i=1x

ti

i sốhạng

higi, có nghĩa cảtổngf =!1≤i≤nhigibằng0 Mâu thuẫn

Các toán

Bài toán 1(Định lýCauchy – Davenport) Với tập hợpA, Btrong trường sốnàođó ta định nghĩaA+B = {a+b|a∈A, b ∈B} Khiđó với sốnguyên tốpvà với tập hợp A, B ⊆Zp ta có

|A+B|≥min{p,|A|+|B|−1} (2)

Li gii. Nếu như|A+B|> pthì khẳngđịnh toán hiển nhiên: trường hợp A+B =Zp Giảsử|A|+|B|≤ pvà giảsửngược lại rằng|A+B|≤ |A|+|B|−2 Khiđó

trongZptồn tập hợpCchứaA+B có|C|=|A|+|B|−2phần tử

Xétđa thức

f(x, y) = $

c∈C

(x+y−c)

trongFp[x, y] Bậc bằng|A|+|B|−2, hệsốcủax|A|−1y|B|−1bằngC||AA||−+|1B|−2 modp

Biểu thức khơng 0, |A|+|B| ≤ p Theo định lý khôngđiểm tổ hợp, tồn a∈Avàb∈B chof(a, b)̸= 0.Điều mâu thuẫn vớiđịnh nghĩa củaf Vậyđiều giảsử sai, nghĩa là|A+B|≥|A|+|B|−1

Định lýCauchy-Davenport có nhiềuứng dụng Vàứng dụng tiếng kết quảkinhđiển sau:

Bài tốn 2 (Định lýErods-Ginzburg-Ziv) Cho sốngun dươngn Khiđó từ 2n−1sốln chọnđượcnsốcó tổng chia hết chon

Li gii. Bằng quy nạp ta đưa tốn vềtrường hợpn=plà sốngun tố(Nếun=ab khẳngđịnhđãđược chứng minh vớiavàbthì từ2n−1số, ta có thểlần lượt lấyđược2b−1 a số(các bộkhông giao nhau) cho tổng sốcủa tổng chia hết cho a Chia tổng choarồi áp dụng cho thương thuđược khẳngđịnhđối vớib.)

Giảsửa1, a2, , anlà sốnguyênđã cho Khơng tính tổng qt có thểgiảsử0 ≤a1 ≤

(19)

1 ai+p−1 = với inàođó thuộc{1, , p−1} Khiđó khẳng định hiển nhiên ta có

thểchọn sốai =ai+1 = =ai+p−1

2 ai+p−1 > aivới mọii∈{1, , p−1} Trong trường hợp ta xét tập hợp

Ai ={ai, ai+p−1},1≤i≤n

Lần lượt áp dụng tốn 1, ta có

|A1+A2 + +Ap−1|≥p,

tức làA1+A2+ +Ap−1 =Zp Nói riêng−a2p−1 ∈A1 +A2+ +Ap−1 Nhưvậy tồn

các sốci ∈{ai, ai+p−1}, choc1+ +cp−1+a2p−1 = 0(trongZp)

Lưu ý định lý Erdos-Ginzburg-Ziv chứng minh cáchđộc lập mà không cần dùngđếnđịnh lýkhôngđiểm tổhợp Chúng dành việc tìm chứng minh nhưvậy cho bạnđọc nhưmột tập

Bài toán 3. Chod, n ∈Nplà sốnguyên tố Khiđó tồn sốx1, , xd∈Z, cho

xd1+xd2+ +xdd≡n( mod p)

Li gii. Ta giả sử d < p, d ≥ p theo định lý nhỏ Fermat, xd ≡

xd+1−p( mod p) Giảsửd=ad

1, trongđód|p−1,(a, p−1) = Khiđó

xd = (xa)d1. Nhưng ánh xạx )→ xa là song ánh từ F

p vào (thật vậy, tìmđược sốngun

dươngk, l choak = (p−1)l+ 1, nhưchẳng hạnxa

1 =xa2 ̸= 0thìx1 = xak1 =

xak

2 =x2)

Vì chỉcần chứng minh khẳngđịnh cho trường hợpd|p−1; giảsửp−1 = kd, k ∈N Xét đa thức

f(x1, , xd) =

$

j∈Zp\{n}

(xd1+ +xdd−j)

Bậc củaf bằng(p−1)d, hệsốcủaxp1−1 xdp−1bằng(kd)!/(k!)d̸= 0bởi vìkd=p−1< p.

Theođịnh lýkhơngđiểm tổhợp,đa thứcf khơngđồng bằng0 Vì có thểchọnđược giá trịx1, , xd, choxd1+ +xdd̸=jvới mọij ∈Zp\{n} Nhưng khiđóxd1+ +xdd=n,

chính làđiều phải chứng minh

Kết quảtiếp theo mởrộng củađịnh lýChevalley-Warning Như sẽthấy toán 5-6, hệquả đơn giản củađịnh lýkhôngđiểm tổhợp sẽlà công cụrất mạnh

Định lý 2. Choplà sốnguyên tốvà f1, , , fk ∈ Fp[x1, , xn]là các đa thứcn biến trênFp

sao cho fi(0, ,0) = với0 ≤ i ≤ k GiảsửS1, , Sn ⊆ Fp là tập củaFp, cho

0∈Sj với mọij

#

1≤j≤n

(20)

Khiđó hệ

fi(x1, , xn) = 0,1≤i≤k, (4)

Có nghiệm(x1, , xn)∈S1× ×Sn, khác(0, ,0)

Chứng minh. Xétđa thức F(x1, , xn) =

k

$

i=1

(1−fip−1(x1, , xn))−δ n

$

j=1

$

s∈Sj\{0}

(xj−s),

trongđó

δ= (−1)|S1|+ +|Sn|−n

n

$

j=1

$

s∈Sj\{0}

s−1 ̸=

được chọn choF(0, ,0) = 0(ở với mỗis∈Fp\ {0}ta kýhiệus−1 nghịchđảo

strongFp, tức phần tửs′ trongFp choss′ = 1)

Theo (3), F làđa thức bậc!1≤j≤n(|Sj|−1), trongđó hệsốcủa x|1S1|−1x| S2|−1 x|

Sn|−1

n

−δ ̸= Từ định lý không điểm tổ hợp suy tồn (c1, , cn) ∈ S1 × × Sn, cho

F(c1, , cn)̸= VìF(0, ,0) = 0nên khơng phải tất cảcácci bằng0 Nhưng khiđó n

$

j=1

$

s∈Sj\{0}

(cj−s) = 0,

Từ

fip−1(x1, , xn)̸= 1với1≤i≤k

Nhưng theođịnh lýnhỏFermatap−1 = 1trong trườngF

p nếua̸=

Suy fi(c1, , cn) = với ≤ i ≤ k Tức hệ (4) có nghiệm khác (c1, , cn) ∈

S1× ×Sn.Định lý chứng minh

KhiS1 = =Sn =Fptađượcđịnh lýChevalley-Warning Và sauđây hai hệquảtrực tiếp

nữa

Bài toán 4 (Troi-Zannier, 1997) Cho k ∈ N plà số nguyên tố Giả sử rằng S1, , Sn ⊆

{0,1, , p−1}là tập hợp có chứa0 và cho!1≤j≤n(|Sj|−1) ≥ +k(p−1) Giả

sử aji,1 ≤ j ≤ k,1 ≤ i ≤ n là sốnguyên Khi đó tồn phần tử xi ∈ Sin,

1≤i≤n, tất bằng0, cho

aj1x1+aj2x2+ +ajnxn≡( mod p)

với mọij ∈{1, , k}.

(21)

VớiS1 = = Sn = {0,1} ta thuđược kết kinhđiển Olson, sử dụng

trong chủ đềolympic tốn (ví dụtrong IMO Shortlist năm 2003,định lýOlsonđượcđềcập nhưmột toán sốhọc)

Hệquả(Olson, 1969).Chok, n ∈N plà sốnguyên tố, nữan ≥ +k(p−1) Giảsử ajilà sốnguyên bất kỳ,1≤j ≤k,1≤j ≤ n Khiđó tồn tập khác rỗng chỉsố

I ⊂{1, , n}, cho #

i∈I

aji ≡0( mod p)

với mọij = 1,2, , k

Bài tốn 5(Alon) Choplà sốngun tốvàGlàđồthịvơ hướng, trongđó bậc trung bình của cácđỉnh (tức trung bình cộng bậc cácđỉnh) không nhỏhơn2p−2và bậc cao của mộtđỉnh không lớn hơn2p−1 KhiđóGcó chứađồthịconp-đều (tức có thểbỏ số cạnh số đỉnh củađồthị đểbậc mỗiđỉnh lại bằngp)

Li gii. GọiV vàElà tập cácđỉnh cạnh củaGtươngứng Với mỗiđỉnhvvà cạnheta đặt

av,e =

%

1, nếuv ∈e 0, nếuv ̸∈e

Mỗi cạnheta cho tươngứng với biếtxe∈Fp Với mỗiđỉnhv ∈V ta xétđa thức tuyến tính

fv =

#

e∈E

av,exe

Giảthiết bậc trung bình cácđỉnh củaGkhơng nhỏhơn2p−2tươngđương với bấtđẳng thức |E|>(p−1)|V|, dođó sốcác biến sốxe(bằng|E|) thỏa mãnđánh giá

|E|>(p−1)|V|= (p−1)×[sốcácđa thức tuyến tínhfv]

Từ ta có thểáp dụngđịnh lý2 chon =|E|vàS1 = =Sn = {0,1} Thuđượcđiều sau

đây: biến sốxe cho tươngứng0 1sao cho khơng phải tất cảcác phần tử

bằng0và với mộtđỉnh, sốsố1, cho tươngứng với cách cạnh cóđầu mút đỉnhđó, bội củak Vì bậc khơng vượt q2p−1, cạnhđược cho tươngứng với1sẽtạo thành đồthịcon có bậc cácđỉnhđều bằngp

Chúng ta sẽkết thúc câu chuyện mộtứng dụng tuyệt vời sau: chứng minh ngắn sơ cấp cho giảthuyết Erdos-Heilbronn, vốn vấnđềmởsuốt 30 năm Năm 1994,đã tìmđược chứng minh phức tạp sau hai năm chứng minhđược trình bày dướiđây Xin chúývới độc giảvềsựgiống giảthuyết vớiđịnh lýCauchy-Davenport

Bài toán 6(giảthuyết Erdos-Heilbronn) Choplà sốnguyên tố VớiA, B ⊆ Zp, tađịnh nghĩa

A⊙B là tập hợp tất cảcác sốdưkhi chia chopcủa tổng dạnga+bvớia ∈A, b ∈B ba̸= Chứng minh

(22)

Li gii. Giảsử|A|=kvàm = 2k˘4 Ta chọn phần tửabất kỳthuộcAvàđặtB =A\ {a} Dễdàng kiểm trađược rằngA⊙A= A⊙Avà m =|A|+|B|−3 Ta cần chứng minh |A⊙B|≥min{p, m+ 1}

Trường hợp m + ≥ p (tức k ≥ (p + 3)/2) Ta chứng minh trường hợp A⊙A=Zp Xét phần tửmbất kỳthuộcZp Chia phần tửcủaZp thành cặp có

tổng trongZpbằngm Ta thuđược(p−1)/2cặp, trongđó có phần tửsẽ cặp với

nó Theo nguyên lýDirichlet tập hợpAchứa hai phần tửcủa cặp, từ đóm∈ A⊙A, làđiều phải chứng minh

Trường hợp 2.m+ 1< p Ta chứng minh trường hợp này|A⊙A|≥m+ Giảsử ngược lại Khiđó tồn tập conCcủaZpsao cho|C|=mvàA⊙B ⊆C

Xétđa thức

f(x, y) = (x−y)$

c∈C

(x+y−c)

trên Fp[x, y] Đa thức có bậc m+ = (k − 1) + (k − 2), bậc xk−1yk−2

bằng C2kk−−24C2kk−−14 ̸= Zp (hãy chứng minh điều này!), 2k − = m < p Vì

|A| = k > k −1và |B| = k −1 > k−2 nên áp dụng định lý khôngđiểm tổ hợp, tồn x ∈ A, y ∈ B chof(x, y) ̸= Mâu thuẫn theo định nghĩa củaf, f(a, b) = 0với a∈A, b ∈B

Kết luận

Sựphong phú phức tạp ví dụ trình bày nói nói lên tính hiệu quảcủa phương pháp Combinatorial Nullstenllensatz,đãđược thừa nhận mang tính cách mạng lýthuyết sốcộng tính Phép chứng minhđẹpđẽ“nhưsách giáo khoa” kết quảnổi tiếng, nhưbài toán 6,đã minh chứng cho sựkiện cách tiếp cậnđa thức cách tiếp cận tựnhiên toán dạng nhưvậy.Đểso sánh, chứng minh banđầu Da Silva Hamidon năm 1994 dài cáchđáng kể sửdụng kết quảcủađại sốtuyến tính lý thuyết biểu diễn nhómđối xứng

Bài tập

Bài tốn 7. Choplà sốngun tốvàGlàđồthịtrongđó có khơng hơn2p−1đỉnh Chứng minh tồn tập conU khác rỗng cácđỉnh củaG, cho số các cạnh củaG, có nhất mộtđầu mút thuộcU, chia hết chop

Bài toán 8 (Alon) Cho H1, , Hm là họ các siêu mặt phẳng trong Rn, phủ kín tất cả các

đỉnh hình lập phương đơn vị {0,1}n ngoại trừ 1 điểm Chứng minh rằng m ≥ n (Siêu mặt phẳng trong Rn là tập hợp các điểm (x

1, , xn) ∈ Rn, thỏa mãn phương trình dạng

a1x1+ +anxn =b.)

Bài toán 9(Giảthuyết Sneville,được chứng minh Alon) Chop≥3là sốnguyên tốvà giả sửA={a1, , ak}⊆Zp B ={b1, , bk}⊆Zp là tập k phần tửcủa tập hợp số

(23)(24)

TOÁN H

C VÀ

O THU

T

Nguyễn Hùng Sơn

G

IỚI THIỆU

Hãy tưởng tượng chúng tađang có mặt buổi biểu diễn nhàảo thuật gia nổi tiếng Ông rút túi bộbài gồm 32 quân mời người bất kỳlên tham gia buổi biểu diễn ngồiở5 ghế đãđược chuẩn bịsẵn Trước tiên ông mời khán giả

lên tráo (bốc phần bộbài cho xuống bộbài) Sauđó ơng chia cho người chơi quân liên tiếp vàđềnghịhọphải nhìn kỹqn nhưng giấu kín khơng cho biết Nhàảo thuật nói với người chơi:

-Đềnghịcác bạn chỉdùng thần giao cách cảmđể gửi cho thông tin vềcon của các bạn.

Một lúc sau nhàảo thuật lại nói:

-Ở có nhiều người nên không thểphân biệtđược thông tin mà bạn gửi cho Bây giờtôiđềnghịnhững bạnđang cầm quân màuđỏhãyđứng lên và thửtruyền tin cho lần nữa.

Lúc có người ngồiđầu dãyđứng lên chỉsau vài giây nhàảo thuật giađãđọc các ngườiđó là8 A Ơng cịnđọcđược xác (màu đen) ngườiđang ngồi.

Trịảo thuật hồn tồn khơng cầnđánh dấu bài, khơng cần cài người quen vào các người chơi khảnăng phi thường hết Chúng ta tìm hiểu lýthuyết tốn họcẩnđằng sau trịảo thuật này.

1 Dãy số

De Bruijn

Một sốbạn đãđốn chìa khóa qn bàiđược xếp theo thứtự đặc biệt Tuy nhiên,điều khơng giải thích làm thếnào nhàảo thuật đọc tên xác năm quân cơsở thông tin vềmàu sắc chúng Đểgiải điềuđó, phải tìm hiểu khái niệm "huyền bí",được gọi cácdãy sốDe Bruijn

(25)

Hình 1: Dãy vịngA= (1010000110010111)

Định nghĩa (dãy sốDe Bruijn bậcn) Dãy số nhịphânB(n)được gọi dãy sốDe Bruijn bậcnnếu dãy nhịphânđộdàinđều xuất hiệnđúng lần dãy vịngB(n)(ởdạng một dãy cóđộdàin).

Ví dụB(2) = (0011) dãy nhị phân cóđộdài2đều xuất dãy vòngB(2) Ta

cũng dễdàng kiểm tra rằngB(3) = 01000111 Chúng ta dễdàng nhận dãy sốDe Bruijn

bậcnchứađúng2nsốhạng.

Bạnđọc có thểtham khảo thêm thơng tin vềdãy Bruijn thuật tốn xây dựng dãy

B(n)với giá trịcủantại trang web [1]

2 Cơ

sở

toán học

Các nhàảo thuật thực sựkhơng bao tiết lộbí họ May mắn thay cho bạnđọc, nhàảo thuật mà nhà tốn học, nhiệm vụcủa tơi phải tiết lộbí mật cho bạn Trong tròảo thuật này, sẽsửdụng dãy sốBruijnB(5) Dãy số nhưsau:

B(5) = 00001001011001111100011011101010

Có cáchđơn giảnđểtạo dãy sau: bắt đầu với bít00001và bít tiếp theođược tính cách cộng bít thứnhất bít thứba (modulo 2):0 + 0, tađược0 Chúng ta thêm kết quảvào cuối dãy, nhậnđược000010 Sauđó di chuyển sang phải, nghĩa quên bít đầu lặp lại bước (tức ta xét dãy gồm bít cuối cùng00010) Tương tựnhưtrước, cộng bít thứnhất bít thứba (modulo 2): (0 + 0) mod = 0, sauđó thêm bít vào cuối dãy Sau bước này, dãy sốcó dạng0000100 Chúng ta chuyển sang bên phải lần (tức bỏ qua hai bítđầu tiên), xét dãy bít cuối00100 Lần này, tính tổng modulo bítđầu tiên thứ ba nhậnđược1 Sau thêm bít vào cuối dãy tađược00001001 Tiến hành theo cách này, sẽcóđược chuỗi nhịphân De Bruijn bậc5như viếtởtrên Chính xác hơn, dãyB(5) =a1a2 a32thỏa mãn tính chất truy hồi nhưsau:

a1a2a3a4a5 = 00001

(26)

♠ ♣ ♦ ♥ A

ab= 00 01 10 11 cde= 001 010 011 100 101 110 111 000

Hình 2: Cách mã hóa chuỗi nhịphân gồm bítabcdebằng

Đểsửdụng dãyB(5)cho tròảo thuật, quân bộbài 32 quân mã hóa dãy nhịphân abcde có chiều dài theo nguyên tắc: dùng bít (ab)để mã hố bốn chất♥,♦,♣,♠(cơ, rơ, nhép, bích) dùng bít tiếp theo(cde)đểmã hóa giá trịcủa quân : át (một), 2, 3,4,5,6,7,8 nhưtrong Hình Bạnđọc có thểnhận thấy rằng(cde)chính chữsốcuối sốtừ1đến chúngđược viếtởdạng nhịphân

Ta biết ký hiệuB(5) = a1a2 a32 dãy dạngakak+1ak+2ak+3ak+4 củaB(5) (vớik = 1,2, 32, vàa33 =a1, a34 =a2 ) tạo thành tập hợp tất cả32 dãy nhịphân có chiều dài Chúng tađể ýrằng tươngứng với bítaktại vịtrí thứkcủa dãyB(5)ta đặt qn có mã sốlàakak+1ak+2ak+3ak+4 ta cách xếp quân ởHình

Cách xếp nhưthếnày thỏa mãnđồng thời 2điều kiện:

1 Các qn màuđỏlnởvịtrí tươngứng với bít có giá trịbằng1, qnđenởcác bít có giá trịbằng0

2 chỉcần biết bít biết xác vịtrí chúngởtrong dãy de Bruijn

0 0 0 1 0 1 1100

♠A ♠2 ♠4 ♣A ♦2 ♠5 ♣3 ♦6 ♣4 ♥A ♦3 ♠7 ♣7 ♥7 ♥6 ♥4

1 0 1 1 1 0000

♥8 ♦A ♠3 ♠6 ♣5 ♥3 ♦7 ♣6 ♥5 ♥2 ♦5 ♣2 ♦4 ♣8 ♦8 ♠8 Hình 3: Cách biểu diễn dãyB(5)bằng 32 quân

3 Vén bí mật

Đến có lẽbạnđọc biết tồn tảng điều kỳdiệu Chúng ta phân tích lại lần trịảo thuậtđược giới thệuở đầu báo: có người chơi, hai người có quân màuđỏ, ba người cịn lại có qn màuđen Chúng ta sẽghi lại màuđỏlà 1, màuđen là0, vậy, nhậnđược dãy nhịphân11000 Dãy mã sốnhị phân quân tay ngườiđầu tiên:11có nghĩa làđó cơ, cịn bít000ởcuối có nghĩa số8 Qn tay ngườiđâu tiên là♥8

Để tìm tay người thứhai gì, cộng bít thứ ba theo modulo (1 + = 1), ghi vào cuối dãy Dãy sốgồm bít cuối là10001 Mã10là rơ, khi001có nghĩa làA Vì vậy, người chơiởvịtrí thứhaiđang giữ

(27)

Tiếp tục cách này, thấy ba người là: ♠3,♠6,♣5

Bạn thửtập làm biểu diễn tròảo thuật cho bạn bè người thân mình!

4 Các

ứng dụng khác

Ảo thuật lĩnh vực cácứng dụng dãy sốde Bruijn Một sốtính chất quan trọng dãy sốde Bruijn như: "khơng có dãy xuất nhiều lần", khiến cho chúng trởnên cực kỳhữu ích việc phát triển mã hóa Lưuýrằng dãy có độ dàikđều tươngứng với kýtựkhác thôngđiệpđược mã hóa!Đây cơhội

đểnhững người làm bảo mật có thểsửdụng việc mã hóa văn

Ma trận de Bruijn cách cách mởrộng dãy de Bruijn cho bảng hai chiều vàđã sửdụng rộng rãi ngành Computer Vision (thịgiác máy tính) Hãy tưởng tượng robot công nghiệp di chuyển hành lang theo hướng khác Tuy nhiên, muốn robot có thểxácđịnh xác vịtrí Một nhà tốn học nhận thấy có thểcoi mặt phẳng robot ma trận de Bruijn Nhờ tính chất: ma trận chỉxuất lần, robot có thểxácđịnh xác vịtrí Cũng dựa nguyên tắc này, công ty Anotođã thiết kếvàđưa thịtrường bút kỹthuật số.Đểcó thểsửdụng nó, bạn cần tờgiấyđặc biệtđược in với chấm cho chúng tạo thành ma trận de Bruijn Máyảnh đặt đầu bút đọc cửa sổcủa ma trận xung quanhđầu bút cơsởnày có thểxácđịnh vịtrí trang giấy

Chính cácứng dụng thực tếkểtrên tính chấtđộcđáo chúng, toán liên quan đến dãy ma trận de Bruin chủ đềcủa nhiều nghiên cứu lýthuyết

Tài liệu

[1] https://en.wikipedia.org/wiki/De_Bruijn_sequence

Nicolaas Govert de Bruijn

Ông Nicolaas Govert (Dick) de Bruijn ( sinh ngày tháng năm 1918 ngày 17 tháng năm 2012) nhà toán học người Hà Lan,được biếtđến nhiềuđóng góp lĩnh vực giải tích, lýthuyết số, tổhợp logic

(28)(29)

T

HẾ

NÀO LÀ TƯ

DUY

L

OGIC

- H

ÀNH TRÌNH

ĐI TÌM

MÁY BAY MẤ

T TÍCH

M

I

G-21U

Ban Biên t

p T

p chí Epsilon

GIỚ

I THIỆU

Ngày 14 tháng năm 2018,đài truyền hình Việt Nam (VTV) phát sóng phóng vềcuộc hành trình tìm kiếm máy bay MiG-21U thuộc trungđồn Khơng qn 921 tích từ tháng năm 1971.Đến đầu tháng 10 năm 2018, nhiều vật dụng cá nhân thuộc liệt sĩphi công Công Phương Thảo huấn luyện viên,đại úy không quân Liên Xơ Poyarkov Yuri Nikolaevichđãđược tìm thấy xác nhận cơquan có thẩm quyền Một kỳ tíchđãđược tạo nên nhờvào chuỗi suy luận dự đoán logic, vào kinh nghiệm, trìnhđộvà lịng người nhân hậu kết hợp với sựgiúpđỡ từcộngđồng Internet Một câu chuyện cổtích có thật, có hậu Chi tiết tìm kiếm bạn đọc quan tâm đọc xem lạiở VTV nhưcác báo thời gianđó

Trong Epsilon số14 này, Ban Biên tập Epsilon muốnđem lại với bạnđọc góc nhìn khác, thông tin khác từcâu chuyện kỳdiệu này: Epsilon muốn kểlại chuỗi suy luận logic trình làm việc trung thực suy diễn hệthốngđể giải vấnđề"không thểtin nổi" Cá nhân tin rằng, học toán học cách suy luận, học văn học làm người Vì thếchúng tơi hi vọng câu chuyện góp thêm phần vào chủ đềchung "Học tốnđểlàm gì"của tạp chí sốnày

Đểviết này, vinh dự đồng ýcủa thầy Nguyễn Lê Anh, nhà toán học, cựu giảng viên, người thúc đẩy cho vụtìm kiếm ơng ln khiêm tốn nói vềvai trị nhưvậy Chúng tơiđăng dựa theo ngun viết ông Facebook: "Thếnào tưduy Logich"đăng ngày tháng năm 2018 Chúng tơi có kết hợp thêm số thơng tin tìmđược qua báo chí, qua gọi điện ngắn gọn chân tình với ơngđể bổsung giải thích vào chỗvốn khó có thểdiễn giảiđầyđủchỉvới ngắn Facebook

Xinđược mở đầu cho câu chuyện lời viết ơng:

"Khi cịn nhỏtơi cóđược đọc vềmột nhà chiêm tinh tướng sốnổi tiếng về dựbáo các sựkiện Ông ta bịhỏi dự đốn xác ngày chết mình,đểrồiđến ngàyấy ông ta

(30)

Nh

ng suy lu

n ban

đầ

u

Ngày 25 tháng năm 2017, Facebook (FB) người lấy tên Nam Nguyen có viết nhưsau:

KHÔNG AI BỊQUÊN LÃNG, KHÔNG GÌĐƯỢC LÃNG QN

Cơ gái Nga Anna Poyarkova - cháu gái sỹquan Xơ viếtđã tích Việt Nam năm 1971 nỗlực tìm dấu vết người ông Câu chuyện nhưsau:

Poyarkov Yuri Nikolaevich làđại úy không quân Liên Xô sinh năm 1933,đảng viên từ1961, trungđồn phó khơng qn củađơn vịvới mã sốв\ч06858làm nhiệm vụ ởViệt Nam với vai trị phi cơng huấn luyện 30/04/1971 chuyến bay tập máy bay ôngđã bịrơi vào rừng rậm Cảmáy bay, cảthi thểngười phi cơngđều khơngđược tìm thấy, từ đóđại úy Poyarkovđược coi tích

Tình hình cịn phức tạp sựhiện diện chuyên gia Liên Xô thời giờkhơngđược loan báo nhiều, cảtrong BộQuốc phịng CCCP, kểcảnhiều năm sau Và thếlà giađình chỉcịn cóđược kỷvật sau:

- tấmảnh ông Yuri Poyarkov

- khen, huân huy chương Thủtướng, BộQuốc phòng nước Việt Nam Dân ChủCộng Hòa Riêng huân chương “Đồn Kết”được thứtrưởng BộQP kýtặng tháng sau ơng hy sinh - thẻchấm cơng củađại úy Poyarkov có ghi ông giữchức vụ“phi công huấn luyệnđể đào tạo phi công cho không quân Việt Nam bay ban ngày bayđêm cácđiều kiện khí tượngđơn giản phức tạp” – “hy sinh làm nhiệm vụbởi tai nạn bay”

Xin ghi nhớlàởthẻnày ghi rõ “hy sinh” chứkhơng phải “mất tích” Thếnhưng thi hài ơng khơng thấyđượcđưa vềnước, cịn thân nhân thìđược báo tin “mất tích”!

Vậyđiều gìđã xảy vớiđại úy Poyarkov vậy? Giađình có câu hỏi: Ơng tích hồn cảnh nào?Đó buổi bay tập hay trận khơng chiến? Đồn bay ơngđóngở đâu?

Máy bay ơng rơiởkhu vực nào?

Tại khơng thểtìm máy bay rơi, thi thểcủa phi công?

Đại úyđược chôn cất cẩn thậnởViệt Nam hay bây giờxác ông máy bay rừng sâu? Con cháu ơng cịn hy vọng, dù mong manh, mộtđiều kỳdiệu nàođóđã xảy ra, ngày ơng Poyarkov với tuổi 84 cịn sốngđâuđó làng bảnởViệt Nam Hoặc khơng họcũng muốn biết – hồn tồn có quyềnđược biết – cha ơng họ ngã xuống thếnào, không lẽ 46 năm trơi qua mà tạiđây chưa tìm máy bay? Ông Poyarkovđã chiếnđấu cho tất cảchúng ta, nên có lẽViệt Nam nợgiađình ơng câu trảlời thỏađáng Bởi khơng phải bịqn lãng khơng cóđiều có thểlãng qn

(31)

của Sergeyđã xácđịnhđược, có lẽơng Poyarkov bay huấn luyện hy sinh với phi công Công Phương Thảo vùng trời TamĐảo Vẫn cần thêm thông tin!"

Nói theo ngơn ngữcủa khoa học, "bài tốn"có thểtóm tắt nhưsau: thông tinđầu vào: máy bay bay tập phi công Công Phương Thảo thầy dạy Poyarkov bịmất tích vào ngày 30/4/1971 ởvùng TamĐảo, thơng tin cần tìm: máy bay rơiởkhu vực nào, liệu họcó cịn sống, không, thi thể đâu

Ngay khiđọcđược trên, ông Nguyễn Lê Anh viết:

"Tôi cảm thấy nhục máy bay tích khơng tìm thấyởmột nơi cách trung tâm Hà Nội khơng q 70km Những suy nghĩ kích hoạt q trình tưduy tơi.

Tơiđã viết cịm nhưsau:

1 Cần phải tra vào hồsơliệt sĩ"Cống Phương Thảo"1đểtìm thân nhân Chắc sẽcó thơng tin

tốt.

2 Cần phải hỏi thân nhânđểtìm nơi có mộcủa "Cống Phương Thảo" Nếu có mộthật tức là người nhà biếtđược bối cảnh hy sinh.

3 Trong vùng núi TamĐảo người dân có nói vềsựkiện máy bay Mỹrơi vào khu vực núi Người Mỹ tới kiểm tra vàđã xác minhđúng Không thấy người dân nói vềMiG bịrơi vào núi trong dãy TamĐảo.

4 Vào thờiđiểmấyởvùng núi TamĐảo chỉcó người dân tộc sinh sống Nên hỏi may mới có thơng tin.

5 Cách nhờchính quyền xã tồn miền núi phía Bắc hỏi người già vềkhảnăng máy bay rơi.Đi tìm trực tiếp khơng dễvì mưa gióđã làm hết dấu vết.

Đấy khởiđầu trình suy diễn dựa tưduy logich biết thông tin "ông Poyarkov cùng bay huấn luyện hy sinh với phi công Công Phương Thảo."Lẽdĩnhiên không thể

có thành cơng dễnhư vậy Chắc chắn khơng thể có thơng tin vềmộ của anh Cơng Phương Thảo, không thể bỏ qua mà không kiểm tra Tôi đã tìm cách hỏi thơng tin về gia đình Cơng Phương Thảo nhưhỏi người dân khu vực TamĐảo vềtung tích máy bay bịrơi."

(32)

tin từFB vấnđềkhôngđơn giảnđểquyếtđịnhđâu thông tinđúng, vàđúngđến Một tốn q khóđểgiải!

Ơng Nguyễn Lê Anh viết tiếp: "Để ước lượngđược vị trí MIG21 xấu số, tôiđi xác

định nguồn có thểthu lượm thơng tin Tơi điđến kết luận nguồn có thểcung cấp câu trảlời tốt là: vềhiện trường chỉcó thể đến từngười dân tộc, vềthân nhân anh Cơng Phương Thảo chỉcó thểhỏi qua phi cơng cùngđơn vị.

Tôi quen Trung tướng Phạm Tuân, cựu Phó Tư lệnh Chính trị Qn chủng Khơng qn, anh hùng phi công vũtrụ Ngày 26 tháng năm 2017 liên hệvới anh Phạm Tuân Tôi tập trung vào xác minh vấnđềkhó mà có thểquên sau nhiều năm.

- Có hay khơng sựkiện MIG 21 rơi nhưvậy - Những có thểcung cấp thơng tin tin cậy vềvụviệc (anh Phạm Tuân, anh Quang, anh Khánh Duyđại tá phi cơng trungđồn 921 với anh Cơng Phương Thảo.)

- Điều kiện thời tiết bay hôm ấy thế nào? ("thời tiếtđơn giản- tức độnhìn xa 10km, trời trong khơng mây, khơng mưa, khơng gió mạnh)

- Quỹ đạo bay thời gian bay MIG 21 ngày 30-4-1971 thếnào?

Tôi hỏiđộc lập anh Phạm Tuân, anh Quang, anh Nguyễn Khánh Duyđểso sánh kiểm chứng vàđưa kết luận."

Trong vấn ông ghi âm lại sau khiđược đồngýcủa cá nhânđược vấn, vấn cũngđã đượcđăng lên FB ơng Ơng có trị chuyện thêm với Epsilon lại tâm nhưvậy Ơng kểtrung tướng Phạm Tn có nói với ơng:"Chú mày làm mà hỏi chi tiếtấy làm gì?" "Khơng tìmđược! Tao đã bảo rồi, chảnhẽtồn qn khơngđi tìm lạiđểcho ngườiđi tìm Sao mà tìmđược!"Và ơng trả lời:"Thế hệcác anh lớn rồi, khơng cịn đi Cịn thếhệ bọn nhỏ, sẽkhơng quan tâm nữa Nếu khơng phải tụi em làm sẽlàmđây!"

Ông viết tiếp:

Sau làm việc nhiều lần với anh phi công: anh Phạm Tuân, anh Quang, anh Nguyễn Khánh Duy (phi công cùng đơn vị bay với anh Công Phương Thảo) anh Công Văn Mão (người thờcúng anh Công Phương Thảo), tạmđưa nhậnđịnh sau.

Các thông tin tin cậy (từghi âm nói chuyện):

1 Anh Phạm Tuân khẳngđịnh lượng xăng máy bay đủbay 30 phút, sựcốxảy ra vào khoảng từ10 giờtới 12 giờsáng ngày 30/4/1971 Thời tiết tốt.

2 Anh Nguyễn Khánh Duy (phi công cùngđơn vịbay với anh Công Phương Thảo) khẳngđịnh "Công Phương Thảo xin phép bay về".

3 Máy bay thực bayđộ cao trung bình (từ 2000m tới 6000m) cất cánh theo hướng

(33)

4 Tất cảcác anh Phạm Tuân Nguyễn Khánh Duyđều khẳngđịnh bộtưlệnh Khơng Qn và chính quyền cấpđã tổchức tìm kiếm không thấy.

5 Mặc dù anh Phạm Tuân anh Khánh Duy không khẳngđịnh nhiều lần nói tới khảnăng MIG 21 rơiởphía TamĐảo Bắc.

Nhưvậy sau vấn, thông tinđầu vàođã có nhiều hơn, cịn mơhồ Chúng tơi tạm tóm tắt lại qua hình

(34)

Tổng hợp thông tin tin cậy kết hợp với tọađộvà thời gian:

1 Tất cảcác phi côngđều khẳngđịnh sựcốxảy vào khoảng từ 10 giờtới 12 giờsáng ngày 30/4/1971 Ngày hơmấy trời quang mây tầm nhìn xa "ban ngày khí tượngđơn giản" Ngay sau khi xảy sự cố, quân binh chủng quyền cấp đã tổ chức tìm kiếm khơng thấy Cho tới tận ngày 29/9/2017 chưa tìm thấy máy bay nhưxác phi công.

2 Tất cảcác phi côngđều khẳngđịnh sựcốxảy q trình phi cơng Cơng Phương Thảo bay, nhắc lại với thày Yuri Poyarkov Yuri Poyarkov phi công giỏi Máy bay cất cánh từsân bay Đa Phúc, gọi sân bay Nội Bài,ởtọađộ(21.219086, 105.800507). Sau bay tập sẽquay vềhạcánh xuống sân bay này.Đường băng theo hướng Tây Bắc -Đông Nam Máy bay cất cánh theo hướngĐông Nam, hạcánh từhướng Tây Bắc.

3 Sựcốxảy tín hiệu liên lạc, máy bay không Thời gian cất cánh là khoảng 10 giờsáng Anh Phạm Tuân khẳngđịnh "lượng xăng máy bay đủbay 30 phút" Với lượng xăng nhưvậy máy bay chỉcó thểrơi phạm vi lãnh thổViệt Nam.

4 Thời gian bay tới không vực tập bay phút Thời gian bay tập dự định 10 phút Anh Nguyễn Khánh Duy (phi công cùngđơn vịbay với anh Công Phương Thảo) khẳngđịnh "Công Phương Thảo xin phép bay về" Nhưvậy sự cốxảy sau bay tậpđã hoàn thành sự

cốxảy với máy bay không thểbịcoi "trục trặc kỹthuật" Sựcốxảy nhanh vàđột ngộtđến mức cả2 phi công không kịp báo vềsởchỉhuy bay Như vậy sựcốxảy có lẽlà do yếu tốchủquan phi cơng hạ độcao có thểrơi vào vùng khơng khí nhiễu loạn (dịng

đối lưu khơng khí) rồi đâm vào lưng chừng núi Khoảng từ 10 giờ tới 12 giờ, nơi mây thường xuyên bốc lên cao bao phủ đỉnh núi.

5 Anh Nguyễn Khánh Duy khẳng định quỹ đạo bay về sân bay bay theo đường thẳng từ

không vực tớiđiểm có tọađộ(21.259882, 105.688555) (cách sân bay 10km)ởphía Nam Phúc Yênđể bắtđầu hạ cánh xuống sân bay (21.219086, 105.800507).Đường bay vềhướng thẳng vào dãy núi TamĐảo Tùy theo thờiđiểm bắtđầu quay vềmàước tính vịtrí xảy sự cố Thời tiết quang, tầm nhìn xa khảnăng nghe tốt, loại trừcác hướng bay mà người dân có thểphát hiện sựcố.

6.Động cơcủa MIG Khối lượng tấn, dài 4.5 mét,đường kính 1.5m.

Thơng tin tiếng Nga:На МиГ-21С устанавливался один турбореактивный двигатель Р-11-Ф2С-300 (Сухая масса: 1040 кг; Максимальный диаметр: 0,825 м; Длина: 4,61 м) Từtổng kết thơng tin này, ơng tính tốn đưa kết suy diễn sau (tóm tắt hình 2)

Vịng trịn tâm B (21.698588, 105.659969) bán kính từ 10km đến 15km khơng vực bay tập (chi tiết tỷlệcó thểtìm thấy bảnđồGoogle) Thời gian bay từsân bay tới không vực 5 phút Thời gian bay tập không vực 10 phút.Độcao bay từ2000m tới 3000m cách mặt

đất Kết thúc bay tập anh Công Phương Thảo thông báo xin phép quay Nhưvậy sựcố

xảy trênđường máy bay bay vềvà máy bayởtrong tình trạng kỹthuật tốt Từ suy sự

(35)

Hình 2: Các khả xảy dựa thơng tin sau vấn, tính tốn suy diễn logic Nguồn: NLA

Dựa vào hướng sân bay thấy kịch hạ cánh như sau. Đểhạ cánh được máy bay phải bayđếnđiểm C (21.259882, 105.688555)ở độcao 600m thấp dần khoảng 10km

đểtớiđường băng (đường màu xanh) Như vậy máy bay phải hạ độcao từkhoảng 2000mđến 3000m xuốngđộcao 600m quãngđường 30km (từkhông vực bay tập tớiđiểm 10km cách sân bay hạcánh).

Nếu bán kính vịng bay 15km thìđiểm máy bay sẽbắtđầu quay vềsân bay từ điểm có tọa

độ(21.704967, 105.474574) vớiđộcao khoảng 2000m tới 3000m.Địa hìnhđồi núi dọc quỹ đạo bay vềthấp 400m, sauđó máy bay bay dọc theo lưu vực "Sơng PhóĐáy"2 Nhưvậy khó

có thểxảy sựcốvới máy bay, nhưsựcốcó xảy người dân sẽnhìn thấy Vậy bán kính vịng bay 10km.Điểm bắtđầu hạ độcao từ3000mđến 4000m (so với mặt nước biển)để

bay vềcó thểlàđiểm có tọađộ(21.736703, 105.553796) Từnhậnđịnh có thểtìm thấy góc phương vịbay về(góc mầuđỏ) góc ACB, tạo tia AC BC

(36)

Khả năng 1:Đường bay về AC MIG 21 bay về quađỉnh dãy TamĐảo (nơi cóđộ cao 1400mởtọađộ(21.493228, 105.634756)3 Trong 10kmđầu tiênđộcao phải hạ được từ1500m đến 2000m xuống 1500m Nhưvậy khảnăngđâm vàođỉnh dãy TamĐảo Nam ở độ

cao từ1300 trởlên bán kính 10km,ởtọađộ(21.493228, 105.634756) từphía Tây- Bắc. Khả năng 2: Đường bay về BC MIG 21 bay về qua hướng hồ Đại Lải (nơi có đỉnh núi cao 1250mởtọađộ(21.442866, 105.687328) Sau 20km bay phải hạ 2000mđộcao xuống còn 1000mđểbay tiếp 10km phải hạ 700mđộcao xuống 600m tạiđiểm (21.259882, 105.688555) Trên quỹ đạo bay có đỉnh núi cao 1250m Vậy nếuđâm vào núi khảnăng sẽ đâmở độcao khoảng 700m trởlên, bán kính 5km quanh tọađộ(21.442866, 105.687328), từhướng Bắc - Tây Bắc.

Như cả2 phương án, khảnăng cao máy bayđều xảy tai nạnởvùng Nam Tam Đảo, theo phương án vềAC tai nạn sẽxảy raởmặt phía Tây Bắc củađỉnh TamĐảo 3, cịn theo phương án vềBC tai nạn sẽxảy raởphía Bắc - Tây Bắc so vớiđỉnh 1250mở toạ độ(21.442866, 105.687328) Hai khảnăng nàyđược tóm tắtởhình Với kết quảsuy luận này, khơng gian tìm kiếmđãđược thu hẹpđáng kểso với khảnăng "vơ vọng"nhưbanđầu

Hình 3: Hai khảnăng có thểkhi máy bay bay

Ngày 29 tháng năm 2017, cuối viết vật lý, ông (Nguyễn Lê Anh) ghi dòng ngắn:"Phảiđi vụPoliarkov", nhưvậy hành trình tìm kiếm thức bắtđầu

(37)

Lên

đườ

ng

Mặc dù phạm vi tìm kiếmđãđược thu hẹp, cịn rộng, saođó ởdạng khảnăng có thểxảy nhờvào q trình suy luận logic Cần phải có khảo sát chi tiết nhưnhiều thông tin hơn, tốt từchính người dânđịa phương Đểcóđược thơng tin, ơng Nguyễn Lê Anh tiến hành thăm dị trực tiếp cách cứcuối tuần ơngđi vào rừng thuộc dãy TamĐảo Namđểleo núi, vốn hoạtđộng quen thuộc ơng nhiều năm Ơng làm quen với người dân tộc lấy nấm rừng nhờhọhỏi người già thông tin vềcác máy bay rơi khu vực Đơi khiđó chỉlà câu hỏi bâng quơ, thông tinđược đổi lại từ việc hỗtrợ họ chi phí cho việc hái nấm Việc xác địnhđược thơng tin nghe đơn giản thực tế khơng phải nhưvậy Có người dù thật lòng chia sẻnhưng với tuổi tác kiệnđã xảy bốn thập kỷthì độchính xác thơng tin khơng cịn cao nữa, có người sẵn sàng đưa thông tin với độ tin cậy gần khơng có, vốn để đổi lấy chút tài lộc Chúng nhắc lại sựkiện xảy vào thờiđiểm chiến tranh khốc liệt, có nhiều máy bay bịbắn rơi bịtai nạn chứkhơng chỉcó máy bay cần tìm Nói thêm chút vềthơng tin MiG-21, theo Wikipedia, tên đầy đủcủa Mikoyan-Gurevich MiG-21 (tiếng Nga:Микоян и Гуревич МиГ-21 ) (tên kýhiệu NATO: Fishbed) máy bay tiêm kích phản lực, thiết kế phịng thiết kếMikoyan, Liên bang Xơ viết Ở Nga Mikoyan-Gurevich MiG-21được gọi Câyđàn Balalaika bầu trời, có hình dáng cánh tam giác giống đàn dân tộc Nga, với quânđội Việt Nam, MiG-21được gọi gương máu, huyền thoại bầu trời MiG-21 có nhiều phiên khác nhau,đa sốlàđểchiến đấu, có chỗ ngồi dành cho phi công, tìm kiếm MiG-21U, phiên huấn luyện có chỗngồi với vịtrí phía sau dành cho huấn luyện viên.Đây chi tiết quan trọng giúp cho việc xácđịnh sau

Sau thời gian khảo sát, từ nói chuyện trực tiếpđến vấn quađiện thoại, Nguyễn Lê Anh hiểu MiG-21U không bịrơi khu vực TamĐảo Nam, hay nói cách khác, cả2 khảnăng từsuy luận banđầuđều có dấu hiệu phải bịloại trừ Vậy phải tìmở đâu?

Ơng viết:

"Việc dự đốn nó[máy bay MiG21-U]đâm phải phía bắcđỉnh TamĐảo dãy TamĐảo Nam dựa giả định MiG-21U giảmđộcao từtừ Tôi tin chắn MiG-21U sẽbay theođường thẳng AC, loại bỏ khả năng MiG-21Uđâm vào khu vực phía Bắc củađỉnh TamĐảo 3, MiG-21U phải có khảnăng giảmđộcaođột ngột Tơi lại tìm cách hỏi anh Khánh Duy và được anh cho biết MiG-21U có khả năng bổ nhào giảm tốc bằng cánh Vậy rõ MiG-21U đã đột ngột giảmđộ caoở điểm A (trên hình 2) sau khi xin phép bay trởvềvà đã va vào gờnúi TamĐảo Bắc, suyđoán cáchđỉnh TamĐảo khoảng 13km theođường chim bay Chúng tơi có nhóm leo núi thểthao, chuyên leo vào cuối tuầnở

cácđỉnh núi TamĐảo 1, 2, 3; cũngđã bàn vềviệc sẽtổchứcđi tìm Tuy nhiên cơng việc hàng ngàyđã chốn hết cảtâm trí."

Ơng viết tiếp:

(38)

Hình 4: Suy đoán:"Ngay sau xin phép bay trởvềvà đã va vào gờ núi TamĐảo Bắc, suy

đoán cáchđỉnh TamĐảo khoảng 13km theođường chim bay."

năm 1971. Đặng Tuấn đã truy tìm thơng tin mạng Internet thấy được viết tơi cùng tính tốn gần giống với vịtrí máy bay rơi mà bạnấy biết từkhi nhỏ Bạnấy

đã chủ động liên hệvới tôi.

Đặng Tuấn cho biết:

"Bác cháu kể lại năm 1971 có thấy trực thăng quân đội Việt Nam hạ

cánhởchân núi nói tìm máy bay rơi khơng biết thơng tin Sauđó thời gian thì người kể lại người thợ săn họ tìm được máy bay rơi cháy

trên núi Những ngườiđầu tiên biết vềchiếc máy bayđó nhặtđược nhôm vụnđem bán, nhưng họ già lâu Những người thợsăn trẻsau có ngườiđã quay lại vịtríđó nhưng chỉthấy cục sắt lớn nghi làđộng cơvà có kích thước tủlạnh."

"Vì cháu cũngđi làm lâu nên ngày tết không dám hỏi thêm thông tin, nên cháu dùng Google thấyđược thông tin bác, cháu hy vọng thưmà cháu biết có thểgiúpđược gì

đóạ!"

Ơng nhậnđược thơng tin lúcđangởSài Gịnăn tết, ơng mua véđi Hà Nội giờsáng ngày 19 tháng năm 2018 ơng cùngĐặng Tuấnđi vền Mỹ

Ơng tiếp:

Đặng Tuấn báo với mẹlàm cơm cho ngườiđi cùngăn, chúng tôiđi thẳng tới nhà của

Đặng Tuấn anh Hiệu Anh Hiệu năm 49 tuổi Anh Hiệu cho biếtđúng anhđã lăn cái lốp máy bay từ trên đỉnh núi cao xuống vực Tuy nhiên việc này đã xảy từ hơn 20 năm về

(39)

Nhưvậy thông tin vềviệc có máy bay bịrơi trênđịnh núiđãđược người xác nhận. Tơi khơng có thói quenđưa khẳngđịnh khôngđủchứng cứkhách quan Thông báo của Hiệu thành tốrất có trọng lượng nhiên nóđã có từ hơn 20 năm Nó cầnđược kiểm chứng trực tiếp cách tìm lại lốp ấy Tơi cũngđược nghe bà của Đặng Tuấn nói về

ngườiđầu tiên phát máy bayđã lấy nhôm vềbán nhà ôngấy giầu lênđột ngột. Tơi chưa xác minhđược xác thờiđiểm người dânấy phát máy bay, nhưng qua hỏi sơbộthờiđiểm máy bayđược phát 40 năm Vềsau người ta đã nung chảy chỗchiếc máy bayđể mang về bán Vị trí nung chảy máy bayđược gọi Bãi Nhôm Hiệu cho biết anh khơng cịn nhớvịtrí Bãi Nhơmấy Vềchiếcđộng cơthì Hiệu chỉnghe nói mà chưa bao giờnhìn thấy Ngồi Hiệu nói có tinđã có ngườiđưa máy khị lênđểcắt nhỏchiếcđộng cơmangđi bán.

Nhưvậy thông tinđưa tôi đến suy nghĩ, trường hợp xấu nhất, sẽkhơng cịn có khảnăng tìm thấy dấu vết máy bay Cùng chỉ có thểtìm thấy lốp bịlăn xuống vực. TơiđềnghịHiệu tổchức nhóm thám hiểm.

Trao đổi thêm với Epsilon, ông cho biết ông phải suy nghĩ nhiều, nhanh trường hợp Làm có thểbiết liệu "người bạn mới"này cóđưa thơng tin xác hay khơng? Có họcịn nhớ đúng? Giảsửnếu nhưhọnhớ máy bay nói đếnđã MiG-21U cần tìm? Tuy nhiên vịtrí mô tả trùng khớp với kết suy luận ơng, nên mộtđộng lực thơi thúc ơng, ơng tin vào sức mạnh suy luận logic Hơn nữa, cần phảiđi nhanh chậm trễcó thểdo nơn nóng người khác sẽlàm cho tìm kiếm vốnđã khó sẽtrởnên khó khăn Do vậy, ơng quyếtđịnhđi tìm vào cuối tuầnđó, vào hai ngày 24/2 25/2 (thứBảy chủnhật) Chúng tiếp tụcđăng lại trọn vẹn lời kêu gọi ơng cho nhóm tình nguyện tìm kiếm ơng, đăng ngày 22/2/2018đểtơn vinh tầm quan trọng tưduy logic cách làm việc khoa học:

Đội tìm kiếm lư:

Việc tìm kiếm có thể sẽ gặp khó khăn trời mưa sẽ rất nhiều vắt rừng, dấu vết chiếc MiG-21U khơng cịn nhiều Trời có mưa mưa phùn nên khơng ngại,để

lâu sẽcó mưa rào khổ Ngại rét Vềbanđêm trênđỉnh núi trời có thểrét có thểxuống tới 2độC Chúng ta sẽquyếtđi tìm vào ngày 24/2/2018, tìm ngày thứ 7 ChủNhật Dù thế nào cũngđi, khơng tìm thấy khơng Vì q trình tìm kiếm khó khăn nênđộiđi tìm sẽchỉgồm người thực sựchịuđựngđược gian khổ, khơng nênđi vì háo hức Có lẽchỉnên lính đi tìm lính Vậy cần danh sách ngườiđi Chúng ta nênđi cùng ô tôđến nơi leo Những cùngđi inbox cho tơiđểchúng ta lấy

điện thoại nhau.

A - Nhiệm vụ: Nếu toàn bộsốkim loại máy bayđã bịlấyđi chỉcòn hy vọng vào lốp máy bay đã nghịch mà cho lăn xuống vực Lốp trước đường kính khoảng 0.5m, lốp sauđương kính khoảng 1m Phía bên lốp sẽcó chữ in nơi sản xuất ra chúng Từ đưa rađược quyếtđịnh có phảiđó MiG-21U hay khơng. Nhưvậy phải tìm bằngđược lốpấy Dự tính hai lốpấyđã bịlăn xuống vực sâu dốcđá thẳngđứng Rất có thểphải dùng dâyđểleo xuống Thetơi khơng nên mạo hiểm mà nên thuê thợsăn xứ tìm cùng.

Chúng ta cần mộtđội hậu cần mangđồlên cho chúng taănởtại chỗ Chúng ta cần lều ngủ

(40)

mang lên.

B - Kỹthuật leo núi: Vào mùa rừng có nhiều vắt Lên cao do đang mùa lạnh vắt sẽít Trời cũngđã ấm lên loại rắn bò kiếmăn Khi leo núi cần phải để ýquan sát.

+ Mỗi người tựin bảnđồ(lấy Google) Tôi sẽchỉrõ cho bạn vịtrí bắtđầu leo cũng như điểm phải leođến Khi di chuyển rừng bạn phải hình dung vịtrí các bạn bảnđồ Ngay cảkhi khơng có sóngđiện thoại tín hiệu vệtinh có, vậy các bạn cần nhanh chóng cập nhật bảnđồvàođiện thoạiđểbiếtđược mìnhđangở đâu. + Trường hợp bịlạc bạn phải nhanh chóng leo lên vịtrí cao khu vực, nơiấy sẽ

có sóngđiện thoạiđểgọi Các bạn phải bình tĩnhđợi, sóng có thểlúc có lúc khơng Trong mọi trường hợp phải cốdi chuyển vềvịtrí lúc xuất phát.

+ Khơng nên mạo hiểmđi Váchđá có thểrất trơn có thểtuột xuống vực dưới chân mà khơng hay.

C - Vềtrang bị:

- Cả đoàn: lều ngủvà trải cách nhiệt cho người Lương thực thực phẩm nước uống. - Cá nhân:

Mỗi người tự lo cho mình, bao gồm balo, tất cả mọi thứ trong balo đềuđược gói túi nilon đềphịng mưa ướt sẽgầm vàođồ dùng khiến chúng trở nên nặng. Đồdùng cá nhân phải có:

+ Về đồdùng gồm:đèn pin + võng + vải mưa 1.5mx2m (muaởHà Trung) + túi ngủcá nhân + găng tay, giầy leo núi (cỡto sốso với giày thườngđi) tất + quần áo mặcđểleo (2 bộ) quần áoấmđềphòng ngủlại rừng Mỗi người mang theo 1đơi giầy dựphịng

đểphịng giày hỏng (giày thểthao nhẹ) khoảng 40m dây dù nhẹmà bền.

+ Về đồ ăn phải mang theo nước uống lit +đồ ăn nhẹ đủsốngđược ngày + cồn khơ và

bật lửađềphịng tối phải ngủlại thìđốtđống lửa +điện thoại & sạc dựphịng.

Đường màu xanh làđường dự kiến sẽleo đểlênđỉnh Vùng màu đỏdự kiến vùng máy bay rơi Chắc chắn có máy bay rơi khơng rõ máy bay gì.

Khuya ngày 23 rạng sáng ngày 24 tháng năm 2018, khoảng nửa ngày trước lênđường ông viết tiếp chođoàn:

Hầu hết tất cả các núi kể cả Everest chiều cao từ chân núi lên tới đỉnh chỉ vào khoảng 1500m Như vậy vị trí mà cần phải tới được ngày hôm cao Thông thường phảiđi 12kmđểlênđược cao 1000m, nhưvậy tổngđườngđi đểlênđộcao 1500m khoảng 18km Thời gianđi khoảng Chúng tađi theo conđường tắt, nhiều chỗ

dốc thẳngđứng, thếchúng ta cần tiếngđê lên tới nơi Theo kinh nghiệm nhanh nhất khoảng giờchiều thìđồn tớiđượcđộcao 1500m.

(41)

Hình 5: Suyđốn:"Đường màu xanh làđường dựkiến sẽleođểlênđỉnh Vùng màuđỏdựkiến là vùng máy bay rơi."Nguồn: NLA

đấyđã có máy bay phải mang máy dị kim koại tổchứcđồn tìm kiếm có kinh nghiệm hơn.

Nếu việc tốtđẹp sẽxuống núi vào lúc giờchiều vào khoảng 10 đêm sẽ

xuống tới chân núi. Lưuý.

1-Đặt chế độchođiện thoại ghi nhớ đườngđi. 2-Đườngđi thường men theo vực

Có chi tiết nhỏcũng cần phải nói thêm độ tin cậy thơng tin nhậnđược Bằng suy luận, ơng có nghĩtới "thuyết âm mưu", thơng tinđều trùng khớp với tính tốn ban đầu, nên ơng tìm tạm gạt bỏ suy nghĩ Cụthể vào trước ngày đi, ông viết:

Như vậy khơng có cơhội tìm thấy máy bay nóđã bịbiến thành sắt vụn Nếu chúng ta tìm thấy lốp chúng tađọcđược chữ trên lốpấy có thểxácđịnh

được có phải MiG-21U hay khơng.

Chỉcóđiều nhưtồn bộsựviệc diễn rađúng nhưlời kểthì máy bayđã khơng bịcháy nổ Bởi cháy nổthì chảdễgì có thểgomđược mảnhđểnấu Nếu cháy nổthì lốp sẽbị

cháy khơng cịnđểcó mà lăn xuống vực Vậy từ theo thuyết âm mưu có thểcó nhiềuđiềuđể

(42)

Ngày tìm ki

ế

m th

nh

t

Hà Nội, giờsáng ngày 24 tháng năm 2018, người dũng cảm lênđường

Toàn bộthành viên tham gia gồm có người: ngườiđịa phương người tình nguyện viên, trongđó có Nguyễn Lê Anh, người lớn tuổi Ngoài người họra, cịn có hỗtrợ khácởbên mặtđất vàđơngđảo người hỗtrợtinh thần qua Internet

Nguyễn Lê Anh viết lại vềngàyđầu tiên nhưsau:

Nhóm dẫnđường gồm người Hiệu, Trung, Nam, Phú Khơng sốhọcịn nhớvị

trí Bãi Nhơm có lẽhọchưa từngđếnđấy Trong số4 người có Phú cho biết anh

đã sờtay vào chiếcđộng cơmáy bay Hiệu nói làđã lăn lốp từtrên cao xuống vực.

Nhóm tình nguyện viên gồm người, trongđó có tơi Trừtơi thành viênđều cịn trẻvà rất hưng phấn Khi nhìn thấy thành viên khơng mang theo nước uống biết họkhông thểtheo

được Vì thếtơiđã khơng kiểm tra tưtrang thành viên trước lênđường Họchưađủ

kinh nghiệm cho cuộcđi nhưvậy.

Khoảng 10 giờ sáng ngày 24/2/2018 bắtđầu leo núi Theo quyđịnh chỉ bậtđiện thoại vào phút chẵn 30 Hai thành viên sớm bỏcuộc sau leo lên cáiđồi không cao lắm Tôiđưa cho Hiệu chai nước to phân công Hiệuđi cuối Tơi ngườiđịa phương cịn lại đếnđược vịtrí cắm trại vào lúc 14:00 chiều, đúng như dự kiến Nhờcó nước uống, thành viên thứ3 tớiđược, vị tới vào lúc 19:00 tối, anhđềnghịquay lui vào sáng hôm sau.

Sáng hôm sau cửHiệuđưa anhấy quay trởvềvà dặn phải trởlại tìm tiếp4 Ngày 24/2/2018

diễn như vậy Pin điện thoại sạc dựtính cho ngày thám hiểm cũng đã khơng cịn nhiều.

Và nhưvậy, sau ngàyđầu tiên kết thúc, đồn chỉcịn lại "người thúcđẩy"Nguyễn Lê Anh, nhà toán họcđã 60 tuổi, người dânđịa phương

4Trên tranglongnguyen48.blogspot.comcó nói vềngàyđầu tiên phần ngày thứhai với góc nhìn từcác thành viên tình nguyện cịn lại nhưsau:

Chẳng hiểu lần leo núi (là nghề) GS Lê Anh nhưthếnào lần nàyđi, mưa xuân rảrích nặng hạt, trời mù Nói chung, thời tiết phức tạp cho cảviệc leo núi lẫn bay huấn luyện xưa Thậm chí tiến sĩDỗn Hà Thắng cịn khơng thểcho Drone (là thiết bịgiống flycam) hoạtđộngđược cối rậm rạp Ngay ngàyđầu tiên, bác sĩPhúcđã phải bỏcuộc ong rừngđốtđến mức tê dại hai chân, khiến chân khơng có cảm giác

để đứng thăng chuyên gia an ninh mạng Ngơ Việt Khơi sức khỏe dân phượt, đuađược với chuyên gia leo núi bỏcuộc Nhưvậy hai nhân tốtrẻcủađoànđã phải bỏcuộc tìm kiếm từngày

đầu tiên.Đây nguyên nhân mà GS Lê Anh cảm thấy sợhãi sau (sẽkể đoạn sau).

Cuộc tìm kiếm kéo dàiđến tối mịt, chúng tôiởnhà hồi hộp chờ đợi kết Nhưng lo cho tính mạng của thành viênđồn Cho tới lúc chỉcịn anh Ts Dỗn Hà Thắng người tìm kiếm ba người bản xứvốn người Kinh, cáchđây hai mươi nămđã trèo lên khu vực tìm kiếm xác máy bayđể lấy nhôm bán sắt vụn.

Sang ngày thứhai, tiến sĩDỗn Hà Thắng phải bỏcuộc timđập nhanh q, nghẹt thở, có lúc bịngất vì cú trèođá núi cheo leo.Đến lúc chỉcòn GS Lê Anh “đồngđội tìm sắt vụn” Các tính tốn bằng xác xuất toán học Lê Anh chođến giờnày xác hướng tìm kiếm Viện trưởng Mai Hương liên tục

(43)

Ngày tìm ki

ế

m th

hai

Nguyễn Lê Anh viết tiếp:

Vịtrí cắm trại phải nơi có nước Nó nằm cách nơi chúng tơi dự định tìm khoảng đi bộ Thời tiết lạnh mưa phùn nhưngày hôm trước.Đường trơn Như nói ngoại trừ

anh Phú, hai anh Nam Trung chỉngheđồn thổi vềvụmáy bay rơi mà chưa hềnhìn thấy chiếcđộng cơcũng như Bãi Nhơm Do trời mù mà phải tới 10 giờsáng ngày 25/2 bắtđầu

đi tìm chiếcđộng Tơi khơng can dự vào quyếtđịnh màđểhọtự tìm theoýcủa mình, với hy vọng sớm tìm thấy nó.

[Nếu cứtheo lời kểcủa họthì]nhiều bộphận chiếcđộng bịtháo thếkích thước mỗi chiều chỉkhoảng nửa mét Dựtính phạm vi tìm kiếm diễn phạm vi khoảng

1km2 Tơi khơng thật sựtin họsẽtìmđược một vật nhưvậy, một khơng biết chắc chắn vị trí từtrước.Đến 14:00 cảba anh vềtrại Sau nghe thơng báođã tìm nát hết vịtrí có thểtơi trầm ngâm suy nghĩ Từ10:00đến 14:00 tiếng.Đi lại nhanh hết tiếng chỉcó 3 tiếngđi tìm.Đi rừng rậm, vừađi vừa phạt câyđểlấy lốiđi, tốcđộchỉkhoảng 2km/giờ. Ba ngườiđiđược 18km Nếu tầm mắt bao quát nhìn sáng bênđược 10m phảiđi 50km qt hết diện tích1km2

Như có nghĩa họ chưa tìm phần ba khu vực cần tìm kiếm, có thểkết luận "tìm nát hết"các vịtrí Quay lại giảthuyết về"thuyết âm mưu", liệu có khảnăng tồn bộthơng tin vềBãi Nhôm lẫn chi tiết lăn lốt bánh xe xuống vực khơng xác Liệu sựkiệnđó có thật sựxảy Nếu giảnhưtất không xảy

Ơng kểlại lo lắng vào buổi chiều ngày thứhai:

Chiều muộn nhậnđược tin nhắn Nam[Nam Nguyen, người kêu gọiđầu tiên FB vào ngày 25/9/2017] thơng báo quay về vì tất cả những ngườiđịa phươngđang đi tìm với tơi đều khơng biết tí gì, vịtrí máy bay rơiởchỗkhác Tơi giữim lặngđềphòng Các bạn người

địa phương tranh luận với nhiều gọiđiện thoạiđểhỏi Họbảo khơng thểtìmđược vìđã có người mang máy dị kim loại lênđểdị hết mảnh vỡ Thậm chí tên người khị tên người bán cáiđộng cơcũngđược nói họtìm cách gọiđiện hỏi.

Chúng tơiăn tối Tơi hơmấy tơi khơng ngủvà cốhình dung vịtrí máy bay Tôi cho rằng chiếc máy bay chỉbay cao vài chục mét nóđã thốt, vịtrí va chạm của chiếc máy bay phảiởngần gờcủa núi Tơiđánh dấu bảnđồvịtrí gờnhơ lên.

(44)

Ngày tìm ki

ế

m th

ba

Nguyễn Lê Anh kểtiếp:

Sáng thứ2 trời mưa rét Hiệu gọiđiện thoại với giọng ngập ngừng Tôi hiểu nguyên nhân của sựthayđổiấy Hiệu cho biết tối hôm trước Nam, Thắng Hiệuđãđến nhà sốngười dân tộc nhận được thơng tin nào đó Cái Namđã nhắn tin cho Tôi làm như

khơng biết nói Hiệu phải lênđể gặp gỡ chúng tơi tinh thần đồngđội Trên thực tế tơi cũngđã lo, muốn phải có Hiệu.

Tôi cần phảiđưa quyếtđịnh bối cảnh nóiđúng, aiđáng tin thơng tin nàođáng tin cậy Tơi nói với người thép chiếcđộng cơrất dày khơng thểbịkhị mang đi nên tìm ngược theo suối cạn ngược lên đỉnh Vào khoảng 9:00 chúng tơi nhổtrại vừađi tìm vừa Khoảng 10:00 chúng tơi đến chỗtìm Ngay lúcấy chúng tơi gặp cậu niên dân tộc Ngoảnh trước ngoảnh sau họ rút uống hết chai nước của Chỗnày khơng có suối có nước Khơng có nước uống sẽchết khát Tơi bắtđầu thấy sợvì lời cảnh cáo Tơi có cảm giác hình nhưmọi thứ chống lại việc tơi tiếp tụcđi tìm Tuy nhiên tơi hiểu việc phải giữ vẻbềngồi bình tĩnh khơng thay

đổi quan trọng.

Quá trình tìm kiếm chậm Sau khoảng 30 phút tơi hiểu khơng thểtìmđược cáiđộng ấy giữa rừng rậm Tơi nói người lênđỉnh cao tìm cách xácđịnh vịtrí để đi tìm xuống Trên thực tếtơi chỉmuốn xácđịnhđược vịtrí nơi máy bayđã lao vào núi. Khoảng 13:00 giờchúng trèo đến đỉnh Trênđó lạnh, chúng tơi đốt lửa chờHiệu Vào khoảng 14:00 chúng tơi quyếtđịnhđi tìm tiếp Lần tơiđi trước cốtìm thấy gờmà tơi

đã thấy bản đồGoogle Tôi leo lên mộtđỉnh núi biết khi đi xuống đến gờ

tôi muốn Khoảng 14:30 chúng tôiđến chỗ Mọi người cũngđồngýlà tìm xuống từchỗ ấy. Chúng tơi chia thành nhóm đi tìm xuống Tơiđi với Phú Phú có dao phạt cây để đi, tơi khảo sát chỗ đỡdốc mà cho nơi máy bayđâm vào Khơng tìm thấy gì, tơi thất vọngđi vào phía khe suối cạn tồnđá tảng lổn nhổn Lịng suối khóđi, bịngã lần. Và bắtđầu thấy sợ, chỉcần bịngã thụt chân vào kheđá gãy chân sẽkhơng thểra khỏi rừngđược.

Tôi bắtđầu suy nghĩ Nếu cứthếnày xuốngđược tới conđường bên Sauđó phải mất giờ mới rađược khỏi rừng Như vậy Lúcấy đã khoảng giờchiều, vậy chúng tơi chỉcó thểra khỏi rừng lúc đêm.Đi rừng trời tối nhanh trời tối thìđi chậm Nếu sẽphải 10 đêm chưa rađược khỏi rừng Hiểu sựnguy hiểm, tơi nói với Phú thất bại rồi, phát lệnh "Lui Quân".

Tôi lệnh cho Phú phải phát cây, vẫnđi tìm cáiđộng cơ, tìmởconđường dốc nhất, dễ nhấtđểxuống Xét thâm tâm sợ, cần phải xuống thật nhanh khỏi rừng trước trời tối Phúđi trước phátđường vào chỗít dốc có thể, tơiđi sau Và thật kỳ

diệu, chỉphátđườngđược khoảng 5m chúng tơi nhìn thấy mảnh máy bay.

(45)

Hình 6: Vào lúc 14:50 ngày 26 tháng năm 2018,đồn tìm kiếmđã tìm thấy mảnh máy bayở vịtrí (21◦34’46.9"N 105◦33’04.9"E) cách xa khơng q 100m so với vịtríđược tính tốn dựa

vào suy luận logic Nguồn: NLA

(46)

Ông kết thúc kýsựcủa nhưsau:

Xét vềbản chất sau 47 năm mưa dầm mảnh máy bayấy sẽchỉcó thể đọng lại nơi mà dốc nhất có thể.

Chúng tơiởtrong rừng ngày 2đêm Không kểthời gianđiđường, tổng thời gian tìm khoảng 3 giờvà 30 phút Rất người biếtđược tưduy cuối cùngđưa lại thành công bắt nguồn không phải từtưduy logich mà từnỗi sợhãi tôi.

Chúng tơi nhanh chóng về 5 giờ chiều đã khỏi rừng Sau kết cục thật có hậu cho một chuyếnđi vất vả, anh em chúng tôi, Nam, Trung, Hiệu Phú thấy tin yêu Nam Nguyen lênđón tơi gửi chút q nhỏbiếu bạn cùngđi tìm.

K

ế

t thúc hành trình

Sau tìmđược mảnh vỡmáy bay bịnghi máy bay MiG-21U tích,đồn tìm kiếm buộc phải quay vềvà khơng thểtìm tiếp lýdo họ kiệt sức, nhưkhơngđủnước, lương thực cảpin cho thiết bị Họ sauđóđã bàn giao mảnh máy bay cho cácđơn vịcó thẩm quyền Với mảnh máy bay này, hàng loạt câu hỏi cần giải đáp: liệu có phải mảnh máy bay cần tìm hay khơng? Máy bay bịtai nạn nhưthếnào? Nếuđúng, liệu tìm thấy thi hài người hay khơng? Trong lúc tìm thơng tin, có người kể "thày Poyarkov cõng anh Thảo mặtđầy máuđi rừng có aiđóđã bắn chết họ", liệu có phải sựthật?

Như giới thiệu với bạnđọc, MiG-21U máy bay dịng MiG-21 có chỗngồi, may mắn mảnh vỡ tìm thấy mẫuởgiữa vịtríđó Với chuỗi logic so sánh, đối chiếu với hìnhảnh thơng tin, chun gia NhậtĐình, người bạn ông Nguyễn Lê Anhđã chứng minhđượcđó chắn phải mảnh vỡcủa MiG-21U Vềsau cơquan có thẩm quyền cũngđã xác nhận tính xác, kỳtíchđược xác nhận!

Vềcác câu hỏi cịn lại, ơng Nguyễn Lê Anh viết lại việc vào ngày 23/3/2018 nhưsau:

Ngày 28/2/2018, chúng tơi đã bàn giao mảnh ID tọađộnơi tìm và ảnh chụp vị trí mảnh ghép cho Qn chủng Khơng Qn Lẽdĩnhiên vịtrí rơi MiG-21U nằm khơng xa vịtrí mảnh máy bay tìmđược nhiều người dân biết rõ vịtríấy.

Tuy cần phải xác minh thực hư vềcâu chuyện lan đồn "thày Poyarkov cõng anh Thảo mặtđầy máuđi rừng có aiđóđã bắn chết họ."Lời nguyềnđộcđịa cần phảiđược giải thốtđểtrảlại sựvô can cho linh hồn người thợsănđã khuất.

Tôi đã leo lên núi lần thứhai Lần nhằm mụcđích xácđịnh "liệu phi cơng có khảnăng cịn sống hay không?"

(47)

Suy diễn: Mảnh ID MiG-21U nằm vịtrí máy bay rơi, caođộchênh 140m,ởkhoảng cách xa 183m Trong bán kính 200m phía mảnh ID MiG-21U khơng cịn có vịtrí nào có thểnghi nơi bịmột máy bayđâm vào Nhưvậy có thểkhẳngđịnhđược vịtrí chiếc MiG-21U anh Thảo Poyarkovđâm vào sường núi tọađộnói trên.

Rất nhiều người dân khẳngđịnh tìmđược vỏmàu vàng khơng ruột chiếcđồng hồ

Poljot nơi nghi máy bayđâm vào sường núi Nếu chắnđó chiếcđồng hồ đeo tay củaĐại úy phi cơng Poyarkov cảhai phi cơngđã hy sinh nơi máy bayđâm vào núi.

NhậtĐìnhđã xác minh mảnh vỡID MiG-21U làởphía sườn trái cabin MiG-21U Dựa vào tọađộ mảnh vỡ ID MiG-21U tìmđược, tọađộ nơi có hố vếtđâm vào núi, tính ra

được mảnh vỡ văng sớm 70m so với vịtrí hố Nhưvậy máy bayđã cà lườn phải vào vách núi khoảng 50m trước húc khoét hốmỗi chiều 2m Mảnh ID MiG-21U bịvăng máy bay cà lườn phải vào núi Mảnh ID không bịbiến dạng theo chiều dọc, bịvăng theo phương vng góc Vận tốc mảnh ID MiG-21U bắn khỏi máy bay khoảng 100km/giờ Tính tốn phù hợp với vết sạt dài khoảng 30mđểlại trường. Những giây cuối Ngay sau xin phép bay về, MIG đã bổnhào từ 4000m xuống 2000m đểbay về Nam Phúc Yên Lúc vận tốc MIG lên đến 1200km/giờ Chiếc MIG chắcđã kịp dùng bụng cản gió đểtriệt tiêu động Vận tốc giảm xuống cịn khoảng 600km/giờ Cuối tháng tư, mùa hè nóng Từ10 giờtrở mặt trời lên cao làm cho dịng khíđối lưu mạnh Nơi sátđỉnh núi khơng khí bốc lên mạnh, nơi thung lũng dịng khíđi xuống Thung lũng Hồng Nơng cóđường kính 4km, như vậy 12 giây cuối MIG21U mới phát nóđã bịhạthấpđộcao mức Chiếc MiG-21Uđã ngoặt gấp sang trái 90

độvới hy vọng vượt qua gờnúi Hồng Nơng - n Mỹ, phía thấp Nó thoát nạn nếu bay cao hơnđược 10m (bay theođúng giáo trình khơng thểphạm sai lầm nhiều) Khi phi công rời khỏi máy bay, máy bay sẽbay thẳng theo qn tính Nhưvậy thời gian bay vịng phi công vẫnởtrong máy bay. Độdàiđoạn bay thẳng cuối không 0.5km Nếu bay với tốcđộ600km/giờthì khoảng giây Nhưvậy giây thứ3 phi cơng cịn trong máy bay Thời gianđểphóng ghếra khỏi máy bay khơng giây Dođó cho dù phi cơng có khởi động ghế phóng hay khơng, họcũng đã lao vào sườn núi chết vị trí máy bay rơi.

Những suy diễn nói phù hợp với sựthật người dân cho biết vào thờiđiểm ấy máy bay trực thăng bay quầnđảo nhiều ngày phạm vi núi TamĐảo Bắc.Đó máy bay trực thăng Qn chủng Khơng Qnđi tìm MiG-21U Nếu phi cơng cịn sống họ phát tín hiệu cho trực thăng.

Nhưvậy câu chuyện lanđồn vềhìnhảnh người thày Poyarkov cõng anh Thảođi rừng chỉ

như một minh chứng cho tình cảm sự giúpđỡcủa Liên Xơ với Việt Nam Hìnhảnh rất đẹp nhưng chỉlà sự tưởng tượng Các phi côngđã chết máy bay sạt vào núi Vàđây cũng là khẳngđịnh giải lời nguyềnđộcđịa có aiđóđã bắn họ.

Xinđược mặc niệm tỏlòng biếtơn anh hùng liệt sĩ hy sinh tổquốc Việt Nam.

(48)

L

i k

ế

t

Vào ngàyđẹp trời mùa hè 2018, tổng biên tập Trần Nam Dũng cóýmuốn "hồi sinh"Epsilon sau sốcuối (số13) mắt bạnđọc năm vềtrước Chúng tự hào Epsilon ln hài lịng với quyếtđịnhđểEpsilon dừng lại khiđangđượcủng hộrất nhiệt thành từcộngđồng (đểnhường chỗcho tạp chí Pi) Tơi ln suy nghĩnếuđểEpsilon quay trởlại, tơi phải làm gìđó hơn, hay Tôi tựhỏiđâu thếmạnh lớn Epsilon tạm trảlờiđó sựtựdo Epsilon khơng gị bó vềnội dung, miễn có liên quanđến tốn, khơng áp đặt hình thức Epsilon khơng phân biệt viết từhọc sinh, hay từ giáo sư, tiến sĩ danh Ngơ Bảo Châu, tất bìnhđẳng với Vậy khơng có sốbài viết với hình thức mới, khơng phải chỉlàđềbài lời giải?

BM2E phong trào khác, khởi xướng người, Trần Nam Dũng BM2E có mục tiêu làđem tốn học tới người chủ đềgầnđây BM2E "học toánđể làm gì" Khi đọc tiêuđề đó, tơi nói với ơng: "Em khơng thấy có làm tốt huyền thoại NLA Nếu thầy mờiđược, sẽlà vinh dựvà làđột phá lớn cho BM2E", ông làmđược!

Khi ý tưởng bên thơng, thứ cịn lại đến cách tự nhiên Vốn kiến thức có từ viết FB Nguyễn Lê Anh, có lẽchúng tơi có dùng thêm 10 năm với Epsilon đăng hết! Nhưng tơi khơng muốn chọn viết toán học ông, muốn chọn vấnđề"đời"hơn, hấp dẫn Hành trình tìm MiG-21U có lẽrất nhiều người biết truyền thơngđã làm tốtđiều này, làm họtìm nó, chuỗi suy luận tính tốnđó, khơng ghi lại cho hậu thếmột cách rõ ràng chi tiết, thậtđáng tiếc Và chưa làm, chúng tơi sẽlàm

(49)

G

I

I THI

U BÀI TOÁN T

I

Ư

U HAI L

P

(B

I

-

LEVEL OPTIMISATION PROBLEM

)

Võ Nh

t Vinh

(

Đạ

i h

c Caen Normandie, Caen, Pháp)

G

IỚI THIỆU

Tại giải Penalty Challenge 2019, nhóm cầu thủsẽthách thức nhóm thủ

mơnđang thiđấu V-League 2019 với "đấu súng" từchấm 11m: cú

đá với thủ môn cầu thủmà nhóm tự chọn Mỗi thủ mơn thờiđiểm có phongđộ(xác suất chặnđược bóng) khác nhau.Ứng với cầu thủ, thủmơn có xác suất chặnđược cúđá 11m cầu thủ Câu hỏiđượcđặt là: thủ

môn chọn,đểxác suất chắn chặnđược bóng cao

1 Gi

i thi

u chung

Trong câu chuyện vừa kểphía trên, với thủ mơn, xác suất chắn chặnđược bóng ứng với xác suất chặnđược bóng thấp khiđốiđầu với cầu thủ Sưso sánh xác suất thấp cho phép chọn lựa thủ mơn phù hợp tiêu chí: người có xác suất thấp cao Nói cách khác, toán "lớn - nhỏnhất" (Max-Min) bao gồm hai tốn tối

ưu, lời giải toán (xác suất thấp nhất)được dùng ràng buộc phục vụcho tốn cịn lại (xác suất thấp cao nhất).Đây ví dụ đơn giản toán tối

ưu hai lớp (bi-level optimisation problem)

Bài toán tốiưu hai lớp khởi nguồn từtrò chơi Stackelberg [1,2] nhằm nghiên cứu sựphụthuộc quyếtđịnh kinh tếthịtrường [3] Trong nghiên cứuđó, q trình lên kế

hoạch kinh tếliên quanđến sựtương tác tác nhânởhai cấpđộriêng biệt: vài cá nhân (được gọi chung làlãnhđạo- leader) chỉthịcho tác nhân lại (gọi làngườiđi theo -followers) [4] Các toán bắtđầuđược nghiên cứu loạt hai tác giảBracken McGill [5, 6,7], thuật ngữ "hai lớp" (bi-level) bắtđầu xuất báo cáo Candler Norton [8] Trong thực tế, toán xuất rộng rãi nhiều vấn đề

thuộc lĩnh vựcđa ngành Phần sẽgiới thiệu thêm vềbài toán lĩnh vực thực tế

Bài toán tối ưu hai lớp xem xét hai biến địnhx y cho hai mục tiêu tối ưu phụ thuộc

(50)

hai lớp tốn tốiưu mà trongđó, phần ràng buộcđượcđịnh nghĩa toán tốiưu thứhai Theo [3], tốn tốiưu thứhai đượcđịnh nghĩa ( 1):

min

x {f(x, y) :g(x, y)≤0} (1)

Bài toán nàyđược gọi làbài toán theo(follower’s problem) haybài toán dưới(lower problem) GọiΦ(y)là tập nghiệm toán ( 1) vàx(y)là phần tửnàođó tậpΦ(y) Nhiệm vụ toán ( 2),bài toán dẫn(leader’s problem) haybài toán trên(upper problem) xácđịnh lời giải tốt nhấty∗ vàx(y∗)thỏa mãn ràng buộcG(x(y), y)≤0vàđemđến giá trịtốt cho

F(x(y), y):

min

y {F(x(y), y) :G(x(y), y)≤0, x(y)∈Φ(y)} (2)

Bài tốn tốiưu hai lớp giải nhiều phương pháp khác cách biểnđổi thành tốn tốiưu thơng thường (một lớp) ( [3], trang 3) Nếu toán cho lời giải nhất, toán hai lớp làổnđịnh (stable) Tuy nhiên, toán hai lớpđược biếnđổi thành tốn lớp, giải trường hợp nghiệm tốn khơng

Trởlại tốnởgiải Penalty Challenge 2019, biếnxvà biếnylần lượt biểu diễn cho cầu thủ

và thủmôn GọiP(x, y)là xác suất chặn bóng thủmơny người sút bóng cầu

thủx GọiQ(y)là phongđộhiện thủmơny

Bài tốn (lower problem) ( 1)được viết lại nhưsau:

x {f(x, y) : g(x, y)≤0}

f(x, y) = P(x, y) (3)

g(x, y) = (4)

Bài toán (upper problem) ( 2)được viết lại nhưsau:

y {F(x(y), y) : G(x(y), y)≤0, x(y)∈Φ(y)}

F(x(y), y) = −P(x(y), y)×Q(y) (5)

G(x(y), y) = (6)

2 K

thu

t gi

i

Bài toán tốiưu hai lớp, trường hợpđơn giản nhất, bao gồm hai tốn tuyến tính tốn vớiđộkhóN P −Hard [9] Vì vậy, có thểnói tốn tốiưu hai lớp

bài tốn khó vàđểgiảiđược nó, cần thuật tốn cóđộphức tạp thuộc lớpN P Trong

(51)

2.1 K

thu

t s

d

ng

đ

i

u ki

n KKT

Điều kiện KKT [11,12] (KKT conditions)được dùngđểbiếnđổi toán theo thành mộtđiều kiện ràng buộc tốn dẫn, quađó biến tốn tốiưu hai lớp thành toán tốiưu lớp

Bài toán tốiưu hai lớpđược viết lại thành:

x,y F(x, y) (7)

Thỏa mãn hệ điều kiện:

G(x, y) ≤ (8)

g(x, y) ≤ (9)

λ ≤ (10)

λT ×g(x, y) = (11)

∂L(x, y,λ) = (12) Trong L(x, y,λ) = f(x, y) + λT × g(x, y) Nếu L(x, y,λ) là một hàm lồi (convex) thì

∂L(x, y,λ) =∇L(x, y,λ) Trong trường hợp tổng quát,điều kiện KKT làđiều kiệnđủnên bảo

đảm lời giải nghiệm tốn (nếu có, khơng đảm bảo tính đầy đủcủa tập nghiệm)

Ngồi ra, tốn theo khơng có nghiệm nhất, tốn tốiưu hai lớp trởnên phức tạp với trường hợp lạc quan (optimistic position) trường hợp bi quan (pessimistic position) nhưtính tồn cục hay cục bộcủa lời giải tốiưu

2.2 Công c

YALMIP

Bộcông cụ(toolbox) YALMIP dành cho Matlab [13,14]đóng vai trị giao diện phổ

quát (generic interface) nhằm giúp người dùng giải tốn tốiưu khác nhau,

đó có toán tối ưu hai lớp cách dễ dàng YALMIP cho phép người dùng đích danh cơng cụgiải (solver) kèm theo tùy chọn Nói cách khác, YALMIPđơn giản hóa việc thực thi (implementation) giải tốn tốiưu vớioptimizekèm theo sựchỉ định cơng cụgiải nhưlinproghayquadprog Các tốn tốiưu hai lớpđơn giản giải trực tiếp cách sửdụngsolvebilevelcủa YALMIP

3 M

t s

áp d

ng

Như đềcậpởphía trên, tốn tốiưu hai lớp xuất rộng rãi thực tế ởnhiều lĩnh vực

đa ngành Phần dướiđây giới thiệu hai toán thực tế mà toán tốiưu hai lớp sử

(52)

3.1 Bài toán thu phí c

u

đườ

ng

Ởnhiều quốc gia, tuyến giao thông huyết mạchđược xây dựng theo hai nhánh song song: nhánh miễn phí nhánh có thu phí (chất lượng tốt hơn, tốcđộcho phép cao hơn) Ngườiđiều khiển phương tiện giao thơng có thểchủ động chọn nhánhđường tùy theo nhu cầu mình.Ở

gócđộ đơn vịquản lýcầuđường, họmuốn thu thật nhiều tiền Sốtiền họthuđược phụthuộc vào giá tiềnđưa sốlượng xe chấp nhận trảsốtiềnấy Ngược lại,ởgócđộngườiđiều khiển xe, người sẽcân nhắc lựa chọnđườngđi với việc chấp nhận trảmức phíđó hay khơng: nhanh thuận tiện thếnào, yêu cầu thời gian chuyếnđi sao, nhu cầuđi nhanh hay chậm người ngồi xe

Ứng với mức phíđượcấnđịnh, tốn theo liên quanđến chi phí tối thiểu cho xe, tức chi phí tốn sử dụng tuyếnđường có thu phí (bao gồm phí cầuđường) chi phí sửdụng tuyếnđường miễn phí Nghiệm tốn theo câu trảlời cho sốlượng xe sửdụng tuyếnđường có thu phíứng với mức phíđãấnđịnh Bài tốn dẫn sẽquyếtđịnh mức phí

ấnđịnhđểtích mức phí sốlượng xe trảphí lớn

3.2 Bài tốn giá l

ướ

i

đ

i

n thơng minh

Khác với lướiđiện truyền thống, người sửdụngđiện lướiđiện thông minh (Smart-Grids) có khảnăng tùy chỉnh mức sửdụngđiện nhưdễdàng thayđổi nhà cung cấp Trong dựán liên quanđến việc sửdụng bìnhắc-quy lướiđiện thơng minh tạiĐại học Kỹthuật Vienna (TU Wien - Áo), mơ hình đơn vịkinh doanh điện (trung gian nhà cung cấpđiện người tiêu dùng)được áp dụng với vai trò thúcđẩyđảm bảo sản lượng tiêu thụ điện theo cam kết Trong dự án này, sách giá linh hoạt theo ấnđịnh đơn vị

kinh doanhđãđược nghiên cứu nhằmđịnh hướng hành vi sửdụngđiện khách hàng Tương tự tốn thu phí cầuđường, ảnh hưởng qua lại sách giá hành vi khách hàngđãđược nghiên cứu tốn tốiưu hai lớp [15]

Ởgócđộchi tiết hơn, việc sửdụng lượng tái tạođòi hỏi sựtuân thủchặt chẽvềcông suất tiêu thụ điện (tải) nhằm tránh q tải Trong khiđó, hộgiađìnhở Đứcđã sử dụngđến 79 TWhđiện để đun nước nóng vào năm 2015 [16], bao gồm xấp xỉ 12 TWh cho máy nước nóng giađình [17] Mức tiêu thụ điện chiếm 9.3% lượngđiện tiêu thụcủa hộgiađình 2.0% tộng lượng tiêu thụ điện tồn nướcĐức Các sốnày cho thấy tốn sửdụng

điện hiệu quảcho máy nước nóng giađình quan trọng TrườngĐại học Kỹthuật Hamburg (TUHH -Đức)đã thực nghiên cứu vềbài toán với việc áp dụng toán tốiưu hai lớp Ứng với mức giáđiện cho trước, hộgiađình có sách sử dụng máy nước nóng gia đình cho hóa đơnđiện phải trảlà thấp (bài tốn theo) Chính sách sử dụng máy nước nóng giađình hộdânứng với mức giáđiện sẽcho rađường tiêu thụ điện tương ứng - thứ mà người ta mong muốn giống vớiđường tiêu thụ điện định mức (căn

vào công suấtđiệnđược cung cấp vào thờiđiểm) Vì vậy, tốn dẫn, người ta mong muốn tìm mức giáđiện cho sựkhác biệt đường tiêu thụ điện (ứng với mức giá)

(53)

4 K

ế

t lu

n

Bài toán tối ưu hai lớp (bi-level optimisation problem) tốn tối ưu có

phụthuộc kiểu dẫn-theo hai biến quyếtđịnh Bài toán loại tốn khó u cầu thuật tốn với độphức tạp thuộc lớpN P đểgiải Cách thường sửdụngđểgiải toán

cốgắng biếnđổi toán gốc thành tốn tốiưu thơng thường lớp Có nhiều kỹthuậtđể

giải viết này, kỹthuật sửdụngđiều kiện Karush-Kuhn-Tucker (KKT) đãđược giới thiệu Ngồi ra, cơng cụYALMIP mơi trường Matlab cũngđược giới thiệuđểgiải tốn tốiưu, trongđó có tốn tốiưu hai lớp

Bài toán tốiưu hai lớp xuất rộng rãi thực tế,đặc biệt lĩnh vực liên ngành Trong viết này, vịdụliên quan tới bóng đá hay tính phí cầuđường điều chỉnh giá lướiđiện thơng minh (Smart-Grids) đãđược giới thiệu mơ hình hóa cách sử

dụng toán tốiưu hai lớp Bài toán vẫnđang thu hútđược nhiều nhà nghiên cứu nhằm sửdụng chúngđểmơ hình hóa tốn thực tế, nhưnhằm tìm cáchđểgiải chúng cách hiệu quảhơn

Tài li

u

[1] H Von Stackelberg,The Theory of the Market Economy London: Oxford University Press, 1952 [2] H Von Stackelberg,Market Structure and Equilibrium Berlin, Heidelberg: Springer Berlin

Hei-delberg, 2011

[3] S Dempe, Nonconvex Optimization and Its Applications - Foundations of Bilevel Programming Dordrecht: Kluwer Academic Publisher, 2002

[4] B Colson, P Marcotte, and G Savard, “An overview of bilevel optimization,”Annals of Operations Research, vol 153, no 1, pp 235–256, 2007

[5] J Bracken and J T McGill, “Mathematical Programs with Optimization Problems in the Con-straints,”Operations Research, vol 21, pp 37–44, feb 1973

[6] J Bracken and J T McGill, “Defense Applications of Mathematical Programs with Optimization Problems in the Constraints,”Operations Research, vol 22, pp 1086–1096, oct 1974

[7] J Bracken and J T McGill, “Production and marketing decisions with multiple objectives in a competitive environment,”Journal of Optimization Theory and Applications, vol 24, pp 449–458, mar 1978

[8] W Candler and R Norton, “Multilevel programming,” tech rep., World Bank Development Re-search Center, Washington D.C, 1977

[9] R G Jeroslow, “The polynomial hierarchy and a simple model for competitive analysis,” Mathe-matical Programming, vol 32, pp 146–164, jun 1985

[10] MathWorks, “Matlab,” 2016

(54)

[12] H W Kuhn and A W Tucker, “Nonlinear programming,” inProceedings of 2nd Berkeley Sympo-sium, (Berkeley, CA), pp 481–492, University of California Press, 1951

[13] J Lăofberg, YALMIP : A Toolbox for Modeling and Optimization in MATLAB,” inProceedings of the CACSD Conference, (Taipei, Taiwan), 2004

[14] J Lăofberg, YALMIP, 2017

[15] R M Kovacevic, N V Vo, and J Haunschmied, “Bilevel approaches for distributed DSM using internal individualized prices,” in2017 IEEE International Conference on Smart Grid Communi-cations (SmartGridComm), pp 521–526, IEEE, oct 2017

[16] Destatis, “Destatis—statistisches bundesamt,” 2017

[17] G Stryi-Hipp, Grosolstudie zu groòen solarwăarmean- lagen, tech rep., Bundesverband-Solarwirtschaft e.V, Berlin, 2007

(55)

T

ÍNH

C

H

T

P

HI

A

RCHIMEDEAN

C

A

Đ

NH

G

P-A

DIC

Nguy

n Song Minh

G

IỚI THIỆU

Khái niệm định giá p-adic (hay gọi sốmũ đúng), khái niệm quan trọng SốHọc giúp chuyển hóa vấnđềSốhọc sang ngơn ngữGiải Tíchđểtìm cách giải Bài viết này, có nội dung bànđến tính chất củađịnh giá p-adic,đó tính phi Archimedean Phầnđầu viết, trình bày lại khái niệm, quy tắc tính chất Phần cịn lại, tốn áp dụng

Các quy

ướ

c k

ý

hi

u

Trong viết này, sửdụng kýhiệu vớiýnghĩađược quyước thống nhưsau:

! gcd.a; b/WƯớc sốchung lớn củaa; b 2Z

! m 6 j aWSốnguyênakhông chia ht cho snguyờnm Ô0:

! bxc WSnguyờn ln không vượt sốthựcx(phần nguyên củax)

! '.m/WPhi hàm Euler

! vp.x/WHàmđịnh giá p-adic

! PWTập hợp chứa tất cảcác sốnguyên tố

1 M

ở đầ

u

Với hai sốnguyên dươngavàm, theođịnh lýcơbản Sốhọc, ta có biểu diễn sauđây aDY

p2P

pvp.a/; m DY

p2P

pvp.m/:

Ở đó, bộsốtựnhiên!vp.a/"p2P và!vp.m/"p2P xácđịnh nhất, theo nghĩa nếua Dm

(56)

Việc kiểm tra quan hệmja, quy vềxem xét tínhđúng sai củađánh giá vp.a/ !vp.m/; 8p 2P:

Việc xem xét quan hệa"b mod m/vớia > b vàa; b2 N!, quy vềkiểm tra bấtđẳng thức vp.a#b/!vp.m/; 8p 2P:

Như vậy, nhiều định tính cơbản Sốhọc kiểm soát qua đánh giáđịnh lượng vớivp Lýdođó, cho ta thấy cần quan tâmđếnđịnh nghĩa sau

Định nghĩa (Định giá p-adic trênN!) Nếu sốngun dươngmcó phân tích thừa sốngun

tố ởdạng

m DY p2P

pvp.m/;

trongđóvp.m/2N, với sốnguyên tốpcố địnhvp.m/được gọi làđịnh giá p-adic

củam:

Ví dụv2.2016/D5; v3.2016/D2vàv7.2016/D1cịn

v11.2016/ Dv17.2016/ Dv2017.2016/ D0:

Với sốnguyên tốpvà sốnguyên dươngmcho trước, tađặt Y

q2Pnfpg

qvq.m/ Dm0; vp.m/Dk; thìp 6 j m0và cóđượcmDpkm0:

Nhưvậy, vềbản chất thìkD vp.m/là sốlớn sốtựnhiênt thỏa mãnpt j m Vì pt j0vớit lớn thỏaý, nên ta mởrộngđược khái niệmđịnh giá p-adic lênNvới quyước

vp.0/D C1; 8p 2P:

Nếu quyước thêm rằngvp.a/ D vp.#a/với a ZnN, ta mởrộngđược khái niệmđịnh giá

p-adic trênZ, với chấtvp.m/Dk, nếum Dpkm0vớik2 N; m02Zvàp j m0:

Bây giờ, với sốhữu tr Ô0bt k, ta thy luụn tn ti nht sốnguyênkvàa; b 2Z!

thỏap j abvàr Dpk ab:Nhờvậy, ta mởrộngđược khái niệm lênQ, nhưsauđây

Định nghĩa (Định giá p-adic Q) Cho trước sốnguyên tốp và hữu tỷr.

i Nếur D0, thìđịnh giáp#ad i ccar lvp.0/ D C1.

ii Nur Ô0thỡnh giỏp#ad i c củar kýhiệu làvp.r/và ta cóvp.r/ Dkkhi chỉkhi r Dpk ab;trongđó,k2 Za; b 2Z!vớip 6 j ab.

Ởmột góc nhìn khác, sốhữu tỷr viếtđược dạng phân sốlàr D mn vớim; n Zthì

ta sẽcóđược

(57)

2 Các tính ch

t c

ơ

b

n

Vớiplà sốnguyên tố,x; ylà sốhữu tỷbất kỳvàa; b; ml cỏc snguyờn vimÔ0

Khiú, chỳng ta cú tính chất quy tắc cơbản nhưsau

1 vp.xy/Dvp.x/Cvp.y/vàvp.xn/Dnvp.x/vớin2N!

2 vp#yx$Dvp.x/#vp.y/ ; vp.1/D0:

3 vp.gcd.a; b//Dmin˚vp.a/; vp.b/%:

4 vp.lcm.a; b//Dmax˚vp.a/; vp.b/%:

5 x 2Zkhi chỉkhivp.x/!0; 8p 2 P:

6 ajb chỉkhivp.a/ "vp.b/; 8p 2P

7 a"b modm/khi chỉkhivp.a#b/!vp.m/; 8p P

8 vp.xCy/!min˚vp.x/; vp.y/%và khivp.x/ < vp.y/thìvp.x Cy/Dvp.x/:

Các quy tắc tính chất nàyđều chứng minh rấtđơn giản, qua việc trực tiếp sửdụngđịnh nghĩa định giá p-adic trênQ Trong tính chấtđã nêu, bốn tính chấtđầu tiên cho ta quy tắc tiện lợiđểtính tốn định giá Ba tính chất kếtiếp, thực chất cácđiều kiện tươngđương, chúng giúp ta chuyển hóa cácđịnh tính Sốhọc thành định lượng theo vp Riêng tính

chất cuối cùng, cịnđược gọi tính chất phi Archimedean, nhờtính chất ta hình thànhđược chuẩn p-adic trênQbởi cơng thức sauđâykxkp Dp"vp.x/:Khi ta có chuẩn p-adic, ta cóđược khơng gian siêu metric với metric d.x; y/ D kx#ykp Các khơng gian siêu metric, có

một tính chất thú vịlà hình cầu mở(hoặc hình cầuđóng) vừa tập mởvừa tập

đóng hai hình cầu cứcóđiểm chung sẽcó hình cầu chứa hình cầu cịn lại Những tư

tưởng hiệnđại này, tảngđểchúng ta soi xét nhiều toán Olympiad

3 M

t s

bài toán áp d

ng

Bài toán Cho trước sốnguyên tốp, vàn(vớin > 1) phân sốtối giản a1

b1,

a2

b2; : : : ;

an

bn Biết

rằng tổngnphân số sốnguyên,đồng thời tồn sốnguyên dươngmvà chỉsốj thỏa pm jbj Chng minh rng, tn ti chskÔj sao chopmjbk:

Li gii.Đặt

bi Dri giảsửkhông tồn chskÔj chop

m jbk, khiú vp!rj""#m < vp.rk/ ; 8kÔj:

T õy theo tớnh cht phi Archimedean, ta có

vp X 1"i"n

ri !

(58)

Vậy không thểxảyđến việc P

1"i"n

ri 2Z, trái với giảthiết banđầu

Bài toán Cho trước sốnguyên dươnga n lớn hơn1, giảsửrằng với sốnguyên

dươngmđều tồn sốnguyên dươngrmsao cho a"rmn mod m/:

Chứng minh rằng, tồn sốnguyên dương˛ sao choaD˛n.

Lời giải Từgiả thiết toán, ta thấy sẽtồn tạir N! " rn mod a2/:Như

vậy với sốnguyên tốp, ta sẽcó

2vp.a/ Dvp!a2""vp.a#rn/ :

Ta xét hai trng hp sau

1 Nuvp.a/Ôvp.rn/, khiú theo tớnh cht phi Archimedean ta có

2vp.a/"vp.a#rn/"min˚vp.a/ ; vp.rn/%"vp.a/ : Điềuđó dẫnđến làvp.a/D0,đểcónjvp.a/

2 Nếuvp.a/Dvp.rn/, thếthìvp.a/Dvp.rn/Dnvp.r/ ;đểlại cónj vp.a/

Tóm lại, ta ln cónjvp.a/với sốngun tốp, thế˛ D Q p2P

p1nvp.a/ N!và ta cóđiều cần chứng minh từ đẳng thức

aD @Y

p2P

p1nvp.a/

1 A n

:

Chứng minh hồn tất

Bài tốn Tìm tất cảcácđa thứcP x/hệsốnguyên thỏa mãn 2njP 3n/ ; 8n2N:

Lời giải Cố định sốnguyên dươngk, khiđó doP x/2 ZŒx!nên vớia; b 2Zta có v2.a#b/"v2.P a/#P b// :

Kéo theođánh giá sauđây với sốnguyên dươngn

v2.32n#1/Dv2.32nCk #3k/"v2.P 32nCk/#P 3k//:

Theođịnh lýEuler thì2nj!32n#1", có

(59)

Bõy ginuP 3k/Ô0, ta chnn > c Dv2.P 3k//, thếthì theo bấtđẳng thức Bernoulli ta có 1Cn"2nvà kết hợp giảthiết mà suy

c < 1Cn < 1CnCk < 2nCk"v2.P 32nCk//:

Theo tính chất phi Archimedean vàđánh giá (1), ta có mâu thuẫn

n"v2.P 32nCk/#P 3k//Dv2.P 3k// Dc:

Tổng kết suy luận cho ta thấyP !3k" D với số ngun dươngk, từ đóđa thức P x/có vơ sốnghiệm nên phải làđa thức0

Bài tốn Xét sốhữu tỉdươngx1; x2; : : : ; xnthỏa mãnxi C p1

i là sốnguyên dương

vớipi D x1x2###xn

xi ; 8i D1; n:

i Chứng minh rằngx1x2$ $ $xnD1:

ii Có bộsố.x1; x2; : : : ; xn/thỏa mãnđềbài.

Lời giải Đặtp1p2$ $ $pnDa, từgiảthiết ta cóđánh giá sau với sốnguyên tốp 0"vp

&

xi C pi

'

Dvp#xi C xi a

$

Dvp.xi/Cvp.1Ca/#vp.a/ : (2)

Lấy tổng lại với lưuýlàvp.a/D P 1"i"n

vp.xi/, ta có

0"nvp.1Ca/#.n#1/ vp.a/ : (3)

i Nếuvp.a/ > 0, theo tính chất phi Archimedean ta cóvp.1Ca/D vp.1/ D0:Vậy nên

từ(3), có mâu thuẫn với tình huốngđang xét là0"#.n#1/ vp.a/ :

Cịn nếuvp.a/ < 0, theo tính chất phi Archimedean lại có vp.1Ca/ D vp.a/ :Và lại

từ(3), có mâu thuẫn với tình huốngđang xét

0"nvp.1Ca/#.n#1/ vp.a/Dvp.a/:

Tóm lại ln phải cóvp.a/D0với mọip2 P, kết hợpa > 0ta cóđượcaD1

ii TừaD1, với sốnguyên tốplẻvà chỉsối theo (2) có 0"vp.xi/ ; 0"1Cvp.xi/ :

Vậy, sốhữu tỷ có dạngxi D2"1Cki vớiki 2Nvà từaD1ta có ràng buộc

k1Ck2C$ $ $CknDn:

Mỗi bộ.k/D.k1; k2; : : : ; kn/2 Nnsẽtươngứng với một bộsốhữu tỷdương thỏa u

cầu, thếtheo tốn chia kẹo Euler ta có sốbộcần tìm làN D!2nn""11":

(60)

Bài toán Choa1; a2; : : :là dãy vô hạn sốnguyên dương Giảsửtồn sốnguyên

dươngN sao cho

a1 a2 C

a2

a3 C$ $ $C an"1

an C an

a1 Z; 8n!N:

Chứng minh tồn sốnguyên dươngM sao choamC1 Dam; 8m !M.

Lời giải Từgiảthiết, ta có an

anC1 C

anC1"an

a1 Z; 8n !N Vì thế, với sốnguyên tố

p sốnguyên dươngn!N ta có vp

& an

anC1 C

anC1#an a1

'

!0; 8n!N: (4)

Giảsửtồn sốnguyên tốp chovp.an/ < vp.anC1/, vớin!N Dovp# an

anC1

$

< 0nên

từtính chất củađịnh giá phi Archimedean (4) ta có

vp

& an

anC1

'

Dvp

&an

C1#an a1

'

< 0:

Từ có

0 > vp.an/#vp.anC1/Dvp.anC1#an/#vp.a1/Dvp.an/#vp.a1/ :

Tức

vp.an/ < vp.anC1/Dvp.a1/ :

Nếu p số nguyên tố thỏa mãn vp.an/ > vp.anC1/, với n ! N Từ (4) tính chất phi

Archimedean có

0"vp

&

anC1#an a1

'

Dvp.anC1#an/#vp.a1/Dvp.anC1/#vp.a1/ :

Nhưvậy lại cóđánh giá

vp.a1/"vp.anC1/ < vp.an/ :

Tóm lại vớin!N nếuvp.an/ < vp.anC1/sẽcó

vp.an/ < vp.anC1/Dvp.a1/ : (5)

Cịn nếuvp.an/ > vp.anC1/vàn!N, sẽcó

vp.a1/"vp.anC1/ < vp.an/ : (6)

Với sốnguyên tốp, ta xét hai trường hợp

1 Nếu tồn tạik !N chovp.ak/ < vp.akC1/thếthì theo (5) cóvp.akC1/Dvp.a1/

Từ đâyvp.akC2/Dvp.a1/, nếuvp.akC2/ > vp.a1/Dvp.akC1/sẽmâu thuẫn với

(61)

2 Nếu không tồn tạik !N chovp.ak/ < vp.akC1/thì có nghĩa vp.an/!vp.anC1/ ; 8n!N:

Những suy luận cho thấy với sốnguyên tốpbất kỳ, sẽtồn tạiNp đểcó vp.anC1/"vp.an/ ; 8n!Np:

Nhưng giá trịcủavp.an/là sốtựnhiên, nên việc không xảy dấu ởhữu hạn

chỉsố Từ đó, với sốnguyên tốp,đều tồn tạiMp đủlớn cho vp.anC1/Dvp.an/ ; 8n!Mp:

Cũng từ(5) (6) ta lại thấy với sốnguyên tốp

vp.anC1/"max˚vp.an/ ; vp.a1/% "max˚vp.aN/ ; vp.a1/%; 8n!N:

Từ thấy dãyfangn2N! chỉcó hữu hạnước nguyên tố ChọnM Dmax ˚

Mp%vớipchạy khắp

tập cácước nguyên tốcủafangn2N!, ta cóđiều cần chứng minh

Bài toán Chứng minh rằng, tồn vô sốcác bộba sốhữu tỷ.a; b; c/thỏa mãn aCbCc DabcD6:

Lời giải Giảsửbộba sốhữu tỷ.a; b; c/thỏa yêu cầu, khiú cúc Ô0tc D#m6 v cú aCb D6C

m; ab D#mÔ0:

Vy nờn theo Viettố,a; b l nghiệm củađa thứcP x/ Dmx2#.6mC6/ x#m2,đa thức

cần có hai nghiệm hữu tỷcho nên phải tồn tạir Qsao cho "0P D9.mC1/2Cm3 Dr2:

Đếnđây thếmC3Dx, ta quy tốn chứng minh có vơ số.x; y/2Q2thỏa x3#9xC9Dy2:

Xétđường cong elliptic có phương trình.E/W y2 Dx3#9xC9, ta cần chứng tỏtrên.E/có

(62)

x y

Mn(xn, yn)

Mn+1(xn+1, yn+1)

O

GiảsửMn.xn; yn/là mộtđiểm hữu tỷtrên.E/, khiđó tiếp tuyến tạiMncó phương trình

.Tn/W y D

&3x2 n#9 2yn

'

.x #xn/Cyn:

Hồnhđộgiaođiểm tiếp tuyếnđó với.E/là nghiệm phương trình

(&3x2 n#9 2yn

'

.x#xn/Cyn

)2

Dx3#9xC9:

Sau biếnđổi, phương trình trởthành

.x#xn/2 "

x#

&

3xn2#9 2yn

'2

C2xn #

D0:

Vì thế, tiếp tuyến vàđường cong sẽcịn có mộtđiểm chung

MnC1.xnC1; ynC1/D

!

3xn2#9"2 4y2

n

#2xn;

&3x2 n#9 2yn

'

.xnC1#xn/Cyn !

:

Doyn2Dxn3#9xnC9, nên cóxnC1 Df xn/với f x/ D

!

3x2#9"2

4 x3#9xC9/#2x D

(63)

Ta cóx1 Df 0/ D 94, nênv2.x1/D#2 Bây giờgiảsửx Qvàv2.x/"#2khiđó rõ ràng

có cácđánh giá sau

4v2.x/Dv2!x4"< 1C2v2.x/Dv2!18x2"; 4v2.x/Dv2!x4"< 3Cv2.x/Dv2.72x/ ; 4v2.x/Dv2!x4"< 0Dv2.81/ ;

3v2.x/Dv2!x3"< v2.x/Dv2.9x/ ; 3v2.x/Dv2!x3"< 0Dv2.9/ :

Từ theo tính chất phi Archimedean ta có

v2.f x// Dv2

&

x4C18x2#72xC81 x3#9xC9/

'

Dv2!x4C18x2#72xC81"#v2!x3#9xC9"#2

Dv2!x4"#v2!x3"#2

Dv2.x/#2:

Nhưvậy, vớix1 D 94 vàxnC1 Df xn/vớin2N, ta cóđược

#2Dv2.x1/ > v2.x2/ >$ $ $> v2.xn/ >$ $ $

Vậy nên cácđiểmMnlàđơi phân biệt, ta cóđiều phải chứng minh

4 Bài t

p

Sauđây toán,đểtựluyện tập thêm

Bài tập Cho sốnguyên dươnga; b; c thỏa mãn gcd.a; b; c/D1và ajbc; b jca; c jab:

Chứng minh bc

a sốchính phương

Bài tập Cho sốnguyên dươnga; b; mlớn hơn1và sốnguyên tốp thỏa mãn pmDabC1:

Chứng minh rằngp.a#1/.bC1/khơng sốchính phương

Bài tập Cho sốnguyên dươngalớn hơn1, tìm tất cảcác sốnguyên tốp vàqthỏa mãn pq j.ap"qC1/ :

Bài tập Cho sốhữu tỷavàb phân biệt, giảsửtồn vô sốsốnguyên dươngnthỏa mãn an#bn2Z:

(64)

Bài tập Tìm sốnguyên dương a; b; c choab#c; bc#avàca#b số

ngun dương khơng cóước ngun tốlẻ

Bài tập Chof W Z ! Zlà hàm sốkhác hàm thỏa mãn điều kiện với số

nguyên phân biệtavàbbất kỳthì

.a#b/j.f a/#f b// :

Chứng minh tồn tập vô hạnS chứa sốnguyên tố, cho với mỗip S

tồn tạim2 Zsao chop jf m/

Bài tập Với sốnguyên dươngn, kýhiệu

f n/ D

&2v2.n/

n

'v2.n2/

:

Chứng minh nếum sốnguyên dương, vàalà sốngun dương lẻkhơng vượt q mbất kỳthì

aj Y 1"k"m

f k/ :

Bài tập Cho trước sốnguyên dươnga; b nguyên tốcùng nhau, dãyfxngn2ZC xácđịnh

bởix1 Da; x2 Dbvà

xnC2 D

xn2C1Cxn2 xnC1Cxn

; 8n2N!:

Chứng minh vớixn… Z; 8n!3

Bài tập Với số thựcx, ta ký hiệukxklà khoảng cách từ x đến số nguyên gầnx

Chứng minh với sốnguyên dươnga; bluôn tồn sốnguyên tốpvà sốnguyên dương kthoảmãn **

** a pk

**

**C****pbk

**

**C****apCkb

** **D1:

Bài tập 10 Chứng minh vớinlà sốnguyên dương

lcm n ! ; n !

; : : : ; n n

!!

D lcm.1; 2; : : : ; n; nn C1/ C1 :

Bài tập 11 Cho số nguyên dương n ! 2; với s % f1; 2; : : : ; ng s ¤ ; ta ký hiệu #.s/ DQe2se Chứng minh vớiklà sốnguyên dương nhỏhơnnthì

n Y jDk

lcm

&

1; 2; : : : ;

+

n j

,'

(65)

Tài li

u

[1] www.mathscope.org

[2] www.artofproblemsolving.com

[3] www.mathoverflow.net

[4] www.math.stackexchange.com

(66)

P

H

ƯƠ

NG

P

HÁP

T

HÊM

B

I

N

T

RONG

G

I

I

P

H

ƯƠ

NG

T

RÌNH

H

ÀM

Võ Quốc Bá Cẩn

(Archimedes Academy)

G

IỚI THIỆU

Đơi khi, q trình xửlýcác tốn phương trình hàm, ta có thểthêm vài biến phụvàođểphép thếtrởnên linh hoạt hơn, từ phát hiệnđược nhiều tính chất thú vịcủa hàm giúp ích cho việc giải tốn Bài viết này, xin giới thiệu bạnđọc sốbài tốnđược xửlýbằng phương pháp

Bài tốn Tìm tất cảcác hàm số f :R→Rthỏa mãn

f!y+f(x)"= f(x)f(y) + f!f(x)"+f(y)−xy, ∀x,y∈R

Lời giải Thayybởiy+f(z)vào phương trình hàmđã cho khai triển vếphải, tađược

f!y+f(x) + f(z)"=#f(x) +1$f!y+f(z)"+f!f(x)"−x#y+f(z)$

=#f(x) +1$%#f(z) +1$f(y) + f!f(z)"−yz&+f!f(x)"−xyx f(z) =A+f(x)f!f(z)"−yz f(x)−x f(z),

trong A=#f(x) +1$#f(z) +1$f(y) + f!f(x)"+ f!f(z)"−y(x+z) Đảo vị trí xz

trong phương trình với chúýgiá trịcủa biểu thức f!y+f(x) +f(z)" vàAvẫn không

đổi, tađược

f(x)f!f(z)"−yz f(x)−x f(z) = f(z)!f(x)"−yx f(z)−z f(x)

Xem hai vế ởphương trình haiđa thứcẩny.Haiđa thức có giá trị với mọiy

nênđồng với nhau, từ ta có

z f(x) =x f(z), ∀x,z∈R

Suy f(x) =kx (k số thực đó) với mọix∈R Tiếp theo, ta sẽtìm giá trịcủak

Thay f(x) =kxtrởlại phương trình hàmđã cho, tađược

k(y+kx) =k2xy+k2x+kyxy, ∀x,y∈R,

hay

xy(k2−1) =0, ∀x,y∈R

(67)

Bài toán (IMO Shortlist, 2007) Tìm tất cảcác hàm số f :R+→R+ thỏa mãn

f!x+f(y)"= f(x+y) +f(y), ∀x,y>0

Lời giải Thayybởiy+f(z)vào phương trìnhđã cho khai triển hai vế, tađược

f!x+f!y+f(z)""= f!x+y+f(z)"+f!y+f(z)" = f(x+y+z) +f(y+z) +2f(z)

Mặt khác, ta lại có

f!x+f!y+f(z)""= f!x+f(y+z) +f(z)" = f!x+z+f(y+z)"+f(z) = f!x+y+2z) + f(y+z) + f(z)

Đối chiếu hai phép khai triển trên, tađược

f(x+y+2z) = f(x+y+z) +f(z), ∀x,y,z>0

Một cách tươngđương, ta có

f(x+y) = f(x) + f(y), ∀x>y>0

Từ đây, với mọix,y>0vàz>x+y,tađều có

f(z+x+y) = f(z) + f(x+y),

f(z+x+y) = f!(z+x) +y"= f(z+x) + f(y) = f(z) + f(x) + f(y)

Dođó

f(x+y) = f(x) +f(y), ∀x,y>0

Nhưthế, f hàm cộng tính từR+ vào R+.Suy ra f(x) =kx (k là hằng sốdương nàođó) với

mọix>0.Thay trởlại phương trìnhđã cho, tađược

k!x+ky) =k(x+y) +ky, ∀x,y>0,

hay

k(k−2)y=0, ∀y>0

Dođók=2.Vậy có hàm sốthỏa mãn yêu cầu f(x) =2x

Bài tốn (IMO Shortlist, 2011) Tìm tất cảcác cặp hàm số f,g:R→Rthỏa mãn g!f(x+y)"= f(x) + (2x+y)g(y), ∀x,y∈R

(68)

g!f(y)"=b+yg(y), ∀y∈R

Từ suy

f(x) =xg(x) +b−2ax, ∀x∈R

Bây giờ, thayxbởix+zvào phương trình hàmđã cho, tađược

g!f(x+y+z)"= f(x+z) + (2x+2z+y)g(y)

= (x+z)g(x+z) +b−2a(x+z) + (2x+2z+y)g(y)

Thayybởiy+zvào phương trình hàmđã cho, ta có

g!f(x+y+z)"= f(x) + (2x+y+z)g(y+z)

=xg(x) +b−2ax+ (2x+y+z)g(y+z)

Đối chiếu hai kết quảtrên, tađược

xg(x) +b−2ax+ (2x+y+z)g(y+z) = (x+z)g(x+z) +b−2a(x+z) + (2x+2z+y)g(y)

Trong phương trình này, chox=yvà rút gọn thành

xg(x+z) = (x+z)g(x)−az, ∀x,z∈R

Trong phương trình này, thayx=1vàz=x−1,tađược

g(x) =xg(1)−a(x−1) =kx+a, ∀x∈R,

trongđók=g(1)−a.Suy f(x) =xg(x) +b−2ax=kx2−ax+bvới mọix∈R.Thay trởlại

phương trìnhđã cho, tađược

k#k(x+y)2−a(x+y) +b$+a=kx2−ax+b+ (2x+y)(ky+b), ∀x,y∈R

So sánh hệsốcủax2ởhai vế, tađượck2=k.Suy rak=0hoặck=1

• Vớik=0,ta cóa=−ax+b+ (2x+y)bvới mọix,y∈R.So sánh hệsốcủayởhai vế,

tađượcb=0.Từ đó, so sánh hệsốcủaxởhai vế, ta cóa=0.Vậy trường hợp này, ta có f(x)≡0vàg(x)≡0.Thửlại thỏa mãn

• Vớik=1,ta có

(x+y)2−a(x+y) +b+a=x2−ax+b+ (2x+y)(y+b), ∀x,y∈R,

hay

ay+a=2xb+yb, ∀x,y∈R

So sánh hệsốcủaxởhai vế, tađượcb=0.Từ đó, so sánh hệsốcủayởhai vế, ta

a=0.Vậy trường hợp này, ta có f(x) =x2vàg(x) =x.Thửlại thỏa mãn

(69)

Bài tốn Tìm tất cảcác hàm số f :R+→R+ thỏa mãn

f

' x

xy

(

= f!x f(x)"−f!x f(y)", ∀x>y>0

Lời giải Nếu cóa>b>0sao cho f(a) = f(b)thì ta có

f

' a

ab

(

= f!a f(a)"−f!a f(b)"=0, mâu thuẫn Dođó f đơn ánh Từgiảthiết, ta suy

f

' x

xy

(

+f!x f(y)"= f ' x

xz

(

+f!x f(z)"= f!x f(x)"

với mọix>max{y,z}>0.Khơng tính tổng qt, ta chỉcần xéty!z>0.Trong phương

trình trên, chọnx=z+ f(1y) ta cóx f(y) = xxz.Suy f

' x

xy

(

= f!x f(z)"

Do f đơn ánh nên ta có xxy=x f(z),hayx=y+ f(1z).Từ suy z+

f(y) =y+

1

f(z), ∀z!y>0

Nhưthế, ta có f(x) = x+1c (clà sốthực nàođó) với mọix>0.Vì f(x)>0với mọix>0

nênc!0.Thay trởlại phương trình hàm banđầu, tađược

xy x+c(xy) =

x+c

x+c(x+c)−

y+c

x+c(y+c), ∀x>y>0 Chox=y+1,tađược

1 y+1+c =

y+1+c

y+1+c(y+1+c)−

y+c

y+1+c(y+c), ∀y>0 Trong phương trình trên, choy→0+,tađược

1 1+c =

1+c

1+c+c2−

c 1+c2

Giải phương trình này, tađượcc=0.Từ suy f(x) = x1 với mọix>0.Thửlại ta thấy thỏa

mãn Vậy có hàm sốthỏa mãn yêu cầu f(x) = 1x

Bài tốn (IMC, 1999) Tìm tất cảcác hàm số f :R+→R+thỏa mãn

(70)

Lời giải Thayybởiy+zvào phương trìnhđã cho, tađược

f(x+y+z) = f(x)f!(y+z)f(x)", ∀x,y,z>0

Thayxbởix+zvào phương trìnhđã cho, tađược

f(x+y+z) = f(x+z)f!y f(x+z)", ∀x,y,z>0

Từhai kết quảtrên, ta suy

f(x+z)f!y f(x+z)"= f(x)f!(y+z)f(x)", ∀x,y,z>0

Giả sửtồn tạix0,z0>0 cho f(x0+z0)> f(x0).Trong phương trình trên, ta thayx=x0,

z=z0 vày= f(x0z+0zf0()x−0)f(x0) cóy f(x0+z0) = (y+z0)f(x0).Suy f(x0+z0) = f(x0),mâu

thuẫn Dođó f(x+z)" f(x)với mọix,z>0,tức f khơng tăng Xét trường hợp sau:

• Trường hợp 1: f giảm ngặt.Thayybởi f(yx) vào phương trìnhđã cho, tađược

f

'

x+ y

f(x) (

= f(x)f(y), ∀x,y>0

Đảo vịtrí củaxytrong phương trình với chúý f giảm ngặt, tađược

x+ y

f(x) =y+

x

f(y), ∀x,y>0

Thay y=1 vào phương trình trên, ta đượcx+ f(1x) =1+ f(x1),hay f(x) = kx1+1 với

x>0,trongđók= f(11)−1.Do f giảm ngặt từR+ vàoR+nên dễthấyk>0.Thửlại, ta

thấy hàm số f(x) = kx1+1 thỏa mãn u cầu tốn

• Trường hợp 2:Tồn tại0<a<bsao cho f(a) = f(b).Lần lượt thayx=ax=bvào phương trìnhđã cho, tađược

f(y+a) = f(a)f!y f(a)"= f(b)!y f(b)"= f(y+b), ∀y>0

Từ suy f(y) = f(y+ba) với y>a.Do f không giảm nên từ ta suy f(x) =C(Clà sốdương nàođó) với mọix>a.Bây giờ, phương trình hàmđã

cho, ta cố địnhxvà choy>max)a, f(ax)*thì có

C= f(x+y) = f(x)f!y f(x)"=C f(x),

suy f(x) =1với mọix>0.Hàm thỏa mãn yêu cầu tốn

Tóm lại, hàm sốthỏa mãn yêu cầu có dạng f(x) = kx1+1 vớik!0là sốnàođó

Bài tốn (Germany TST, 2007) Tìm tất cảcác hàm số f :R+→R+ thỏa mãn

f

' f (x)

y f(x) +1 (

= x

(71)

Lời giải Từgiảthiết, dễthấy f mộtđơn ánh Thayxbởi z f(fx()+x)1 vào phương trìnhđã cho

khai triển hai vế, tađược

f

+ x

x f(z)+1

xy x f(z)+1+1

, =

f(x)

z f(x)+1

f(x)f(y)

z f(x)+1+1

,

hay

f

' x

xy+x f(z) +1 (

= f(x)

f(x)f(y) +z f(x) +1, ∀x,y,z>0 Thayybởi f(y)vào phương trình trên, tađược

f

+

x

x#f(y) +f(z)$+1 ,

= f(x)

f(x)#f!f(y)"+z$+1, ∀x,y,z>0 (1)

Đảo vị trí yztrong phương trình với chúý giá trị biểu thức f-x[f(y)+xf(z)]+1

vẫn khôngđổi, tađược

f!f(y)"+z= f!f(z)"+y, ∀y,z>0

Từ suy f!f(x)"=x+c(clà sốthực nàođó) với mọix>0.Từ đó, phương trình

(1) viết lại thành

f

+

x

x#f(y) + f(z)$+1 ,

= f(x)

f(x)(y+z+c) +1 =

f(x)

f(x)f!f(y+z)"+1 Thayxbởi f(x)vàybởi f(y+z)vào phương trìnhđã cho, tađược

f

' x

+c (x+c)f(y+z) +1

(

= f(x)

f(x)f!f(y+z)"+1, ∀x,y,z>0 Kết hợp với kết quảtrên, tađược

f

+

x

x#f(y) + f(z)$+1 ,

= f

' x

+c (x+c)f(y+z) +1

(

, ∀x,y,z>0

Do f đơn ánh nên ta có

x

x#f(y) + f(z)$+1=

x+c

(x+c)f(y+z) +1, ∀x,y,z>0, hay

x#(x+c)f(y+z) +1$= (x+c)%x#f(y) +f(z)$+1&, ∀x,y,z>0

Xem hai vếcủa phương trình cácđa thứcẩnx.Haiđa thức có giá trịbằng với

mọix>0nênđồng với nhau, từ cách so sánh hệsốcủax2ởhai vế, ta suy f(y+z) = f(y) +f(z), ∀y,z>0

Kết quảnày chứng tỏ f hàm cộng tính từR+ vào R+.Suy ra f(x) =kx(klà hằng sốdương

nàođó) Thay trởlại phương trìnhđã cho, tađược

k2x

kxy+1 =

x

kxy+1, ∀x,y>0

(72)

Bài tốn Tìm tất cảcác hàm số f :R+→R+ thỏa mãn

f!x f(x) + f(y)"=y+f2(x), ∀x,y>0

Lời giải Từgiảthiết, dễthấy f đơn ánh.Đặt f(1) =a,ta có f!f(y) +a"=y+a2, ∀y>0

Suy hàm f có thểnhận giá trịtrên(a2,+∞).Thayybởi f2(y)vào phương trình hàmđã cho, tađược

f!x f(x) + f!f2(y)""= f2(y) + f2(x), ∀x,y>0

Đảo vịtrí củaxytrong phương trình với chúý f đơn ánh, tađược

x f(x) + f!f2(y)"=y f(y) +f!f2(x)", ∀x,y>0

Từ suy rax f(x) = f!f2(x)"+c (clà số thực nàođó) với x>0.Từ đây, kết

hợp với (1) giảthiết, ta có

f!y+f2(x) +a"= f!f!x f(x) +f(y)"+a" =x f(x) +f(y) +a2

= f!f2(x)"+f(y) +a2+c

Do f(x)có thểnhận giá trịtrên(a2,+∞)nên ta có

f(x+y+a) = f(x) + f(y) +a2+c, ∀x>a4,y>0

Từ đây, với mọix,y>0và với mọiz>a4,ta có

f(z+x+y+2a) = f!(z+x+a) +y+a" = f(z+x+a) + f(y) +a2+c = f(z) + f(x) + f(y) +2(a2+c)

f(z+x+y+2z) = f!z+ (x+y+a) +a" = f(z) + f(x+y+a) +a2+c

Kết hợp hai kết quảtrên lại, tađược

f(x+y+a) = f(x) +f(y) +a2+c, ∀x,y>0

Trong phương trình trên, ta thayx,ybởi x+2y,x+2yf(x+y+a) =2f

'x +y

2

(

+a2+c, ∀x,y>0

Từ suy

2f

'x +y

2

(

(73)

Với mọix,y,z>0,ta có 4f

'x

+y+z

2

(

=2#f(z+x) +f(y)$= f(2z) + f(2x) +2f(y)

Đảo vịtrí củaxytrong phương trình trên, tađược

f(2x) +2f(y) = f(2y) +2f(x), ∀x,y>0

Từ suy f(2x) =2f(x) +m(mlà sốthực nàođó) với mọix>0.Đặtg(x) = f(x) +m ta cóg(2x) =2g(x)và

g(x) +g(y) =2f 'x

+y

2

(

+2m=2g 'x

+y

2

(

=g(x+y), ∀x,y>0

Từphương trình trên, quy nạp ta chứng minhđượcg(nx) =ng(x)với mọin∈Z+ và với

mọix>0.Dog(nx) = f(nx) +m>mnên ta có g(x)> m

n, ∀x>0,n∈Z+

Chon→+∞,tađượcg(x)!0 với mọix>0.Từ đó, ta suy rag hàm cộng tính từ R+ vào R!0.Theo tính chất hàm cộng tính, ta cóg(x) =kx(klà sốkhơng âm nàođó) với

x>0.Suy f(x) =kxmvới mọix>0.Thay kết quảnày trởlại phương trìnhđã cho, tađược k(kx2−mx+kym)−m=y+ (kxm)2, ∀x,y>0

So sánh hệsốcủayởhai vế, tađượck2=1,hayk=1(dok!0) Từ đó, ta có

x2−mx−2m= (xm)2, ∀x>0

So sánh hệsốcủaxởhai vế, tađượcm=0.Nhưvậy, ta có f(x) =xvới mọix>0.Thửlại, ta

thấy thỏa mãn Vậy có hàm sốthỏa mãn yêu cầu f(x) =x

Bài tốn Tìm tất cảcác hàm số f :Z+→Z+ thỏa mãn

f(a+b) = f(a) + f(b) +f(c) +f(d) với mọia,b,c,d nguyên dương choc2+d2=2ab

Ý tưởng cho lời giải tốn tìm số ngun dương a,b,c,d,e,g (trong

{c,d}̸={e, f}) thỏa mãn2ab=c2+d2=e2+g2.Khiđó, ta sẽcó f(c) +f(d) = f(e) +f(g)

Ngồi ra, chọnc,d,e,g biểu thức có dạng tuyến tính bậc theo nthì ta

hoàn tất lời giải phương pháp quy nạp theon

Ýtưởng biểu diễn sốthành tổng hai bình phương theo hai cách khác gợi ta nghĩ đến

đồng thức Lagrange nhưsau: Với sốnguyênm,n,p,q,ta có

(m2+n2)(p2+q2) = (mp+nq)2+ (mqnp)2= (mpnq)2+ (mq+np)2

Chọnm=p=2vàq=4,tađược

(4n+4)2+ (2n−8)2= (4n−4)2+ (2n+8)2=20(n2+4)

(74)

Lời giải Đặt f(1) =k.Thaya=b=c=d=nvào phương trìnhđã cho, tađược

f(2n) =4f(n), ∀n∈Z+. (1)

Thaya=10,b=n2+4,c=4n+4vàd=2n−8(n>4), tađược

f(n2+14) = f(10) +f(n2+4) + f(4n+4) + f(2n−8) = f(10) +f(n2+4) +16f(n+1) +4f(n−4)

Mặt khác, thaya=10,b=n2+4,d=4n−4vàd=2n+8,ta có

f(n2+14) = f(10) +f(n2+4) + f(4n−4) + f(2n+8) = f(10) +f(n2+4) +16f(n−1) +4f(n+4)

Kết hợp hai kết quảtrên, tađược

f(n+4) +4f(n−1) = f(n−4) +4f(n+1), ∀n>4 (2) Bây giờ, ta sẽtính giá trịcủa f(2), f(3), f(4), , f(8).Từ(1), dễthấy f(2) =4k, f(4) =16k f(8) =64k.Do2·5·1=32+12nên ta có

4f(3) = f(6) = f(5+1) = f(5) +f(1) +f(3) +f(1) = f(5) +f(3) +2a

Mặt khác, ta có2·4·1=22+22nên

f(5) = f(4+1) = f(4) + f(1) +f(2) +f(2) =25k

Kết hợp với kết ởtrên, tađược f(3) =9k.Suy f(6) =4f(3) =36k

Ta có72+12=52+52=2·25·1nên

f(26) = f(25) +f(1) +f(5) + f(5) = f(25) + f(1) + f(7) + f(1),

suy f(7) =2f(5)−f(1) =49k.Nhưvậy, ta chứng minhđược f(n) =kn2vớin∈{1,2, ,8}

Kết hợp với (2), ta dễdàng quy nạpđược f(n) =kn2với mọinnguyên dương Thửlại, ta thấy hàm thỏa mãn u cầu tốn Vậy có hàm sốthỏa mãn yêu cầu

f(n) =kn2(klà sốngun dương nàođó)

Bài tốn Tìm tất cảcác cặp hàm số f,g:R→Rthỏa mãn

f(x3+2y) + f(x+y) =g(x+2y), ∀x,y∈R (1)

Lời giải Từgiảthiết, ta có

f(z3+2t) + f(z+t) =g(z+2t), ∀z,t∈R (2) Ta sẽchứng minh rằng, với a,b∈R, hệ phương trình sau ln có nghiệm (x,y,z,t)với

x,y,z,t∈R: ⎧

⎪ ⎪ ⎪ ⎪ ⎨ ⎪ ⎪ ⎪ ⎪ ⎩

(75)

Từcác phương trình thứnhất thứhai, ta cóx=a−2yz=b−2t.Thay vào phương

trình thứba, tađược(a−2y)3+2y=bt,suy

t=b−2y−(a−2y)3

Thayx=a−2y,z=b−2t,t=b−2y−(a−2y)3vào phương trình thứtưcủa hệ, tađược

ay= (b−2t)3+2t=%b−2#b−2y−(a−2y)3$&3+2#b−2y−(a−2y)3$

Đây phương trình bậc9ẩnynên ln có nghiệm thựcy0.Từ suy hệln có

nhất nghiệm thực(x0,y0,z0,t0)vớix0=a−2y0,z0=b−2t0vàt0=b−2y0−(a−2y0)3

Khẳngđịnhđược chứng minh

Từkhẳngđịnh vừa chứng minh phương trình (1), (2), ta dễdàng suy rag(a) =g(b)với mọia,b∈R.Dođóg(x)≡C.Thay trởlại (1), tađược

f(x3+2y) +f(x+y) =C, ∀x,y∈R

Thayy=xx3vào phương trình trên, tađược

f(2xx3) =C

2, ∀x∈R

Do2xx3có thểnhận giá trịtrênRnên từ đây, ta có f(x) =C2 với mọix∈R.Thửlại, ta

thấy f(x) = C2 g(x) =C thỏa mãn yêu cầu Vậy có cặp hàm số f,gthỏa mãn

yêu cầu f(x) =C2 vàg(x) =C(Clà sốthực nàođó)

Bài tốn 10 Tìm tất cảcác hàm số f :R→Rthỏa mãnđồng thời cácđiều kiện sau với bộ

sốthực(a,b,c):

a) Nếua+b+c!0thì f(a3) + f(b3) +f(c3)!3f(abc)

b) Nếua+b+c"0thì f(a3) + f(b3) +f(c3)"3f(abc)

Lời giải Để ýrằng, hàm số f thỏa mãn yêu cầu hàm sốgvớig(x) = f(x)−C(Clà sốnàođó) thỏa mãn u cầu Dođó, khơng tính tổng qt, ta chỉcần xét trường hợp

f(0) =0.Từgiảthiết, ta suy

f(a3) + f(b3) +f(c3) =3f(abc) (1) với mọia,b,c∈Rmàa+b+c=0.Thayc=0vào (1), tađược

f(−a3) = f(b3) =−f(a3),

suy f hàm lẻ Từ đây, thayc=−(a+b)vào (1), tađược

f!(a+b)3"= f(a3) + f(b3) +3f!ab(a+b)", ∀a,b>0 (2) Thaybbởib+cvào phương trình khai triển vếphải, tađược

f!(a+b+c)3"= f(a3) + f!(b+c)3"+3f!a(b+c)(a+b+c)"

(76)

Đảo vịtrí củaactrong dãyđẳng thức trên, tađược

f!bc(b+c)"+f!a(b+c)(a+b+c)"= f!ab(a+b)"+f!c(a+b)(a+b+c)" (3) Tiếp theo, ta sẽchứng minh với mọi0<x<y,hệphương trình sau ln có nghiệm(a,b,c) vớia,b,c>0: ⎧

⎪ ⎨ ⎪ ⎩

bc(b+c) =x,

a(b+c)(a+b+c) =y,

ab(a+b) =c(a+b)(a+b+c)

(4) Hệphương trình tươngđương với

⎧ ⎪ ⎨ ⎪ ⎩

bc(b+c) =x,

a(b+c)(a+b+c) =y, ab=c(a+b+c)

Từphương trình thứhai phương trình thứba, ta cóa2b(b+c) =cy.Kết hợp với phương trình

thứ nhất, tađượca2x=c2y.Suy ra c=ka vớik=3x

y.Ngồi ra, từphương trình thứnhất

phương trình thứhai, ta cóxa(a+b+c) =ybc.Suy rak(a+b+ka) =b,hayb=ka1(1−+kk)

Thay c=ka b= ka1(1−+kk) vào phương trình thứ hệ, ta giải đượca=33 2xk(3k(−k+1)12)

Nhưthế, hệ(4) ln có nghiệm(a,b,c)vớia,b,c>0.Kết hợp kết quảnày với (3) chúý ab(a+b) +c(a+b)(a+b+c) =bc(b+c) +a(b+c)(a+b+c) =x+y,

tađược

f(x) + f(y) =2f 'x

+y

2

(

, ∀x,y>0,x<y

Dếthấy f(x) +f(x) =2f!x+2x"nên ta có

f(x) + f(y) =2f 'x

+y

2

(

, ∀x,y>0 (5)

Từ đây, với mọix,y,z>0,ta có 4f

'x

+y+z

2

(

=2#f(x+z) +f(y)$= f(2x) +f(2z) +2f(y)

Đảo vị trí x y đẳng thức trên, ta f(2x) +2f(y) = f(2y) +2f(x) với

x,y>0.Suy f(2x) =2f(x) +M(Mlà sốthực nàođó) với mọix>0.Dođó f(8x) =2f(4x) +M=4f(2x) +3M=8f(x) +7M, ∀x>0

Mặt khác, từ(2), ta có f(8x) =2f(x) +3f(2x).Suy 8f(x) +7M=2f(x) +3#2f(x) +M$,

(77)

Mặt khác, từ điều kiện a) toán f(0) =0,ta dễdàng suy f(a)!0với mọia!0 Nhưvậy, f hàm cộng tính trênR+ và f(x)!0với mọix>0nên ta có f(x) =ℓx(ℓlà hằng

sốkhơng âm nàođó) với mọix>0.Mà f lẻnên f(x) =ℓx, ∀x∈R.Thửlại, ta thấy thỏa mãn

Vậy hàm sốthỏa mãn yêu cầu có dạng f(x) =ℓx+C(ℓ,Clà sốthực,ℓ!0)

Bài tốn 11 Tìm tất cảcác hàm số f :R→Rthỏa mãn

f!x+ f(y)"= f(y2+3) +2x f(y) +f(x)−3, ∀x,y∈R

Lời giải Dễ thấy f(x)≡0 khơng thỏa mãn phương trình tồn y0 ∈R cho

f(y0)̸=0.Thayy=y0vào phương trìnhđã cho, tađược

f!x+ f(y0)"−f(x) =2x f(y0) + f(y20+3)−3, ∀x∈R

Vếphải phương trình hàm bậc nhấtẩnxnên có thểnhận giá trịtrênR

Từ suy hiệu f(u)−f(v)có thểnhận giá trịtrênR

Bây giờ, thayxbởixf(y)vào phương trìnhđã cho, tađược

f!xf(y)"= f(x)−f(y2+3)−2!xf(y)"f(y) +3, ∀x,y∈R

Trong phương trình trên, ta thayxbởix+ f(z)thìđược

f!x+f(z)−f(y)"= f!x+f(z)"−f(y2+3)−2#x+f(z)−f(y)$f(y) +3

= f(x) +f(z2+3) +2x f(z)−f(y2+3)−2#x+f(z)−f(y)$f(y) = f(x) +2x#f(z)−f(y)$+ f(z2+3)−f(y2+3) +2f(y)#f(y)−f(z)$

Đảo vịtrí củay ztrong phương trình cộng phương trình thuđược phương trình lại theo vế, tađược

f!x+f(z)−f(y)"+f!x+f(y)−f(z)"=2f(x) +2#f(y)−f(z)$2

Do hiệu f(y)−f(z)có thểnhận giá trịtrênRnên từ ta có

f(x+y) + f(xy) =2f(x) +2y2, ∀x,y∈R

Đặt f(0) =ag(x) = f(x)−x2−athì ta cóg(0) =0và

g(x+y) +g(xy) =2g(x), ∀x,y∈R

Thayx=yvào phương trình này, tađượcg(2x) =2g(x)với mọix∈Rnên ta có

g(x+y) +g(xy) =g(2x), ∀x,y∈R

Lần lượt thayx,ybởi x+2y, x−2y vào phương trình trên, ta suy raglà hàm cộng tính

Tiếp theo, thayx=0vào phương trìnhđã cho, tađược

(78)

Thay f(y) =g(y) +y2+avào rút gọn với chúýgcộng tính, tađược

g!g(y)"+g2(y) +2(y2+a)g(y) +2(a−3)(y2+a+2) +g(a)−g(3) =0 Thayy=0vào hương trình trên, tađược

2(a−3)(a+2) +g(a)−g(3) =0 Từ suy

g!g(y)"+g2(y) +2(y2+a)g(y) +2(a−3)y2=0, ∀y∈R

Thayybởiny(n∈Z)vào phương trình trên, tađược

ng!g(y)"+n2g2(y) +2n(n2y2+a)g(y) +2n2(a−3)y2=0, ∀y∈R,n∈Z

Ta xem vế trái đa thức ẩn n Đa thức có giá trị vô hạn giá trị n

nên phải đồng bằng0 Từ suy hệsố củan3 phải bằng 0,hay ta có 2y2g(y) =0

với y∈R Kết hợp với g(0) =0, ta suy g(y) =0 với y∈R Từ với ý 2(a−3)(a+2) +g(a)−g(3) =0, ta tínhđược a=3hoặc a=−2 Suy f(x) =x2+3với mọix∈Rhoặc f(x) =x2−2với mọix∈R

Thửlại, ta thấy chỉcó hàm f(x) =x2+3thỏa mãn yêu cầu Vậy có hàm sốthỏa mãn yêu cầu f(x) =x2+3

Trênđây, chúng tôiđã giới thiệu bạnđọc sốbài toán hay giảiđược phương pháp thêm biến Rất mong sẽnhậnđược traođổiđóng góp xây dựng từbạnđọc gần xa Dướiđây sốbài tập khác có thểgiải phương pháp này:

Bài tập Tìm tất cảcác hàm số f :R→Rthỏa mãn

f(x+y) = f(x)f(y)f(xy), ∀x,y∈R

Bài tập (IMO Shortlist, 2005) Tìm tất cảcác hàm số f :R+→R+ thỏa mãn

f(x)f(y) =2f!x+y f(x)", ∀x,y>0

Bài tập Tìm tất cảcác hàm số f :R+→R+ thỏa mãn

f!x+f(y)"= f(x) + f(x+y)−x, ∀x,y>0

Bài tập Tìm tất cảcác hàm số f liên tục, f :R→Rthỏa mãn

f(x+y) +f(xy) = f(x) +f(y) +f(xy+1), ∀x,y∈R

Bài tập Tìm tất cảcác hàm số f :Z+→Z+ thỏa mãn

f!f2(m) +2f2(n)"=m2+2n2, ∀m,n∈Z+.

Bài tập Cho hàm số f :R→Rthỏa mãn f(0) =0và

f(x) +f(y) = f(x+yxy) +f(xy), ∀x,y∈R

(79)

S

ÁNG

TẠ

O

- L

ÀM

C

H

T

Ngô V

ă

n Thái, Thái Bình

Một tính chất quan trọng bấtđẳng thức tốn học mà học sinhđược biết từkhi cịnđang học tiểu học,đó tính chất bắc cầu: “NếuA!B; B !C thìA!C:”

Tính chất nàyđược sửdụng làm phép suy luận logicởhầu khắp toán chứng minh bất đẳng thức Thếnhưng đứng trước toán chứng minh bấtđẳng thứcA ! C; khơng

phải tốn người rađềcũngđểngười giải nhìn ngayA!B; B !C:Nếu tốnđề

ra mà người giải nhìn thấy ngayA!B; B !C tốnđó q tầm thường Cịn với

bài tốn bấtđẳng thức hay khó, người giải phải sửdụng kiến thức cơbản phù hợp, cách giải phải tường minh, linh hoạt ngắn gọnđể đượcA ! B B ! C;thì người giảiđã làm tốt

được hai việc Việc thứnhất làđã hồn thành chứng minh tốnđềra, việc thứhai làđã sáng tạo rađược toán bấtđẳng thức chặt hơn, khó (có thểhay hơn) tốnđã cho Để thấyđược vẻ đẹp muôn màu sáng tạo bất đẳng thức tốn học, tơi xin giới thiệu viết: “Sáng tạo bấtđẳng thức cách làm chặt số bất đẳng thức tiếng” dựa vào ý tưởng từ“Tính chất bắc cầu bấtđẳng thức” Nội dung viết gồm năm phần sauđây:

! Làm chặt bấtđẳng thức AM-GM

! Làm chặt bấtđẳng thức Cauchy-Schwarz

! Làm chặt toán bấtđẳng thứcđăng báo Toán học Tuổi trẻ ! Làm chặt toán bấtđẳng thức thi Toán học Quốc Tế

! Giới thiệu toán mớiđược làm chặt từnhững toánđã biết

1 Làm ch

t b

t

đẳ

ng th

c AM-GM

Bấtđẳng thức AM-GM bấtđẳng thức cơbản kinhđiển quan trọng tốn học sơ cấp, nóđã có nhiều cách chứng minhđượcđưa ra, hàng chục mởrộng, hàng chục kết chặt hơnđăng diễnđàn toán học Phần xin giới thiệu kết quảchặt bất đẳng thức AM-GM khácđược suy từchính cách chứng minh bấtđẳng thức AM-GM Bất đẳng thức AM-GM.Với sốthực không âma1; a2; : : : ; an.n!2/ta có

(80)

Lời giải Nếu a1a2" " "an D (1)đúng Xét a1a2" " "an > 0; dễ thấy với X !

.1#X / ; !n#"1CXCX2C" " "CXn!1#$cùng dấu Ta có thểgiảsử 1#X /!n#"1CXCX2C" " "CXn!1#$!0;

hay

n 1#X /#.1#Xn/!0;

.n#1/CXn!nX:

Bây đặtX D qn an n!p1a

1a2"""an!1;thì

.n#1/C an

n!p1 a

1a2" " "an!1 !

nn

r a

n

n!p1 a

1a2" " "an!1

;

.n#1/ n!p1 a

1a2" " "an!1Can!npn a1a2" " "an; (2) Dođó

.n#2/ n!p2a

1a2" " "an!2Can!1 !.n#1/ n!p1 a1a2" " "an!1; " " " " " "

2pa1a2Ca3 !3p3 a1a2a3;

a1Ca2 !2pa1a2: Cộng vếvới vếcủan#1ởtrên rút gọn

a1Ca2C" " "Can !npna1a2" " "an:

Đẳng thức xảy chỉkhia1 Da2 D" " "Dan:Ta cóđiều phải chứng minh

Như từ bất đẳng thức (2) có thêm cách chứng minh bất đẳng thức AM-GM gọn, sửdụng bất đẳng thức (2) lại dễdàng rút rađược kết quảchặt bấtđẳng thức AM-GM sauđây

Với sốtựnhiênn!k!p !2vànsốthực khơng âm, khiđó

a1Ca2C" " "Can!ppp a1a2" " "apCapC1C" " "Can

!kpk a

1a2" " "ak CakC1C" " "Can

!npn a

1a2" " "an:

2 Làm ch

t b

t

đẳ

ng th

c Cauchy-Schwarz

(81)

phần lớn tốn chứng minh bấtđẳng thức Ngồi sốhệquảcủa cặp bấtđẳng thức có thểvận dụngđểgiải hàng loạt toán thú vịvềcựcđại cực tiểu

Bất đẳng thức Cauchy-Schwarz.Cho sốthựca1; a2; : : : ; an b1; b2; : : : ; bn; n ! 2/

khiđó "

a12Ca

2C" " "Ca n

# "

b12Cb

2C" " "Cb n

#

!.a1b1Ca2b2C" " "Canbn/2: (3) Lời giải Hiện bấtđẳng thức Cauchy-Schwarz có nhiều cách chứng khác nhau, tất cảcác cách chứng minh đóđều ngắn gọnđặc sắc, xin giới thiệu cách chứng minh số cách chứng minhđã có nhưsau

Nếua21Ca22C" " "Ca2

nD0tứca1 Da2 D" " "DanD0thì (3)đúng Nếua21Ca22C" " "Ca2n> 0;xét

f x/D

n

X

iD1

a2i

!

x2C2

n

X

iD1

aibi

!

xC

n

X

iD1

bi2 D

n

X

iD1

.aixCbi/2 !0; 8x2 R: Dođó biệt thức

!0 D

n

X

iD1

aibi

!2 #

n

X

iD1

a2i

! n X

iD1

bi2

!

"0:

Cho nên

"

a12Ca

2C" " "Ca n

# "

b12Cb

2C" " "Cb n

#

!.a1b1Ca2b2C" " "Canbn/2:

Đẳng thức xảy chỉkhi a1

b1 D

a2

b2 D " " " D

an

bn:

1 Bấtđẳng thức Cauchy-Schwarzđược

chứng minh

Từbấtđẳng thức Cauchy-Schwarz suy hai hệquả đểsửdụng viết này: Hệquả Với sốthựcai; bi; i D1; nthì

"

a2i Ca iC1

# "

bi2Cb iC1

#

!.aibiCaiC1biC1/2: (4) Hệquả Với hai dãy hữu hạn sốthựca1; a2; : : : ; anb1; b2; : : : ; bn; n!2/khiđó

n

X

iD1

q

a2i Cb2i

!2

!

n

X

iD1

ai

!2

C

n

X

iD1

bi

!2

: (5)

Lời giải Xét biểu thức n

X

iD1

q

ai2Cbi2

!2 #

n

X

iD1

ai

!2

D

n

X

iD1

q

a2i Cbi2C

n

X

iD1

ai

! n X

iD1

q

a2i Cbi2#

n

X

iD1

ai

!

D

" n X

iD1

%q

a2i Cb2i Cai

&# "Xn iD1

%q

a2i Cbi2#ai

&#

:

(82)

Theo bấtđẳng thức Cauchy-Schwarz

"Xn iD1

%q

ai2Cbi2Cai

&# "Xn iD1

%q

a2i Cbi2#ai

&#

!

"Xn iD1

s%q

a2i Cb2i Cai

& %q

ai2Cbi2#ai

&#2

D

n

X

iD1

q b2 i !2 D n X

iD1

jbij

!2

! ˇˇˇˇˇ

n

X

iD1

bi ˇˇ ˇˇ ˇ !2 D n X

iD1

bi !2 : Suy n X

iD1

q

a2i Cb2i

!2

!

n

X

iD1

ai

!2

C

n

X

iD1

bi

!2

:

Vậy ta cóđiều phải chứng minh.Đẳng thức xảy chỉkhi a1

b1 D

a2

b2 D" " "D

an

bn:

Bây giờta sửdụng2hệquảtrênđểlàm chặt bấtđẳng thức Cauchy-Schwarz

Dạng Với hai dãy hữu hạn sốthựca1; a2; : : : ; anb1; b2; : : : ; bn; n!2/khiđó n

X

iD1

a2i

! n X

iD1

bi2

!

!

n

X

iD1

aibi

!2

C

4

n

X

iD1

jaibiC1#aiC1bij

!2

; (6)

ở quyướcanC1 Da1; bnC1Db1:

Lời giải Theo (4) với˛i !0; i D1; n;thì

"

a2i Cai2C1# "b2i Cbi2C1#!.aibi CaiC1biC1/2;

suy "

a2i Ca2iC1# "bi2Cbi2C1# D.aibi CaiC1biC1/2C.˛i/2; tươngđương với

˛i D jaibiC1#aiC1bij: Theo bấtđẳng thức Cauchy-Schwarz

4

n

X

iD1

a2i

! Xn iD1

bi2

!

D

"Xn iD1

"

ai2Ca2iC1#

# "Xn iD1

"

bi2Cbi2C1#

#

!

"Xn iD1

q"

a2i Cai2C1# "bi2Cbi2C1#

#2

:

Dođó

4

n

X

iD1

a2i

! Xn

iD1

bi2

!

!

"Xn

iD1

q"

a2i Ca2iC1

# "

bi2Cb2iC1

##2

D

"Xn

iD1

q

.aibi CaiC1biC1/2C.˛i/2

#2

:

Mặt khác theo (5) ta có

" n X

iD1

q

.aibi CaiC1biC1/2C.˛i/2

#2

!

" n X

iD1

.aibi CaiC1biC1/

#2

C

n

X

iD1

˛i

!2

(83)

Cho nên

4

n

X

iD1

a2i

! Xn iD1

bi2

!

!

"Xn iD1

.aibi CaiC1biC1/

#2

C

n

X

iD1

˛i !2 ; hay n X

iD1

a2i

! n X

iD1

bi2

!

!4

n

X

iD1

aibi

!2

C

n

X

iD1

jaibiC1#aiC1bij

!2

;

suy

n

X

iD1

a2i

! n X

iD1

bi2

!

!

n

X

iD1

aibi

!2

C 14

n

X

iD1

jaibiC1#aiC1bij

!2

:

Đẳng thức xảy chỉkhi a1

b1 D

a2

b2 D" " "D

an

bn:

Từ(6) lại có hệquả đẹp sau

Với sốthực dươnga1; a2; : : : ; an; n!2/;thì n

X

iD1

ai

! n X

iD1

1

!

!n2C

n

X

iD1

ˇˇ ˇˇr

aiC1 #

ra

iC1

ai

ˇˇ ˇˇ!

2

;

với quyướcanC1 Da1:

Dạng Với hai dãy hữu hạn sốthựca1; a2; : : : ; anb1; b2; : : : ; bn; n!2/khiđó n

X

iD1

a2i

! Xn iD1

bi2

!

!

"Xn iD1

"

ai2Cbi2#

# Xn iD1

a2ibi2

ai2Cb2i

!

!

n

X

iD1

aibi

!2

:

Lời giải Ta chứng minh vếtrái n

X

iD1

a2i

! Xn iD1

bi2

!

!

"Xn iD1

"

ai2Cb2i#

# Xn iD1

ai2bi2

a2i Cbi2

!

:

Bấtđẳng thức tươngđương với bấtđẳng thức bên

# n

X

iD1

ai2b2i a2i Cbi2

!

!#

%Pn iD1

ai2

& %Pn iD1

b2i

&

n

P

iD1

"

ai2Cb2i# ;

n

X

iD1

a2i# n

X

iD1

ai2b2i a2i Cbi2

!

!

n

X

iD1

a2i#

%Pn

iD1

ai2

& %Pn

iD1

bi2

&

n

P

iD1

"

(84)

n

X

iD1

%

a2i # a

2 ibi2

ai2Cbi2

&

!

n

X

iD1

a2i#

n

P

iD1

ai2 n

P

iD1

bi2 n

P

iD1

ai2C n

P

iD1

b2i

;

n

X

iD1

a4i a2i Cbi2

!

%Pn iD1

a2i

&2 n

P

iD1

ai2C

n

P

iD1

b2i ;

" n X

iD1

a4 i

"

ai2Cbi2#

# " n X

iD1

"

ai2Cbi2#

#

!

n

X

iD1

a2i

!2

:

Hiển nhiên theo bất đẳng thức Cauchy-Schwarz Mặt khác theo bất đẳng thức Cauchy-Schwarz

" n X

iD1

"

a2i Cb i

## Xn iD1

a2ibi2 ai2Cbi2

!

!

n

X

iD1

q

ai2bi2

!2

D

n

X

iD1

jaibij

!2

!

n

X

iD1

aibi

!2

:

Vếphải cũngđược chứng minh.Đẳng thức xảy chỉkhi a1

b1 D

a2

b2 D" " "D

an

bn:

3 Làm ch

t toán b

t

đẳ

ng th

c

đă

ng báo Toán

h

c Tu

i tr

.

Trong “Tuyển tập năm tạp chí Tốn học Tuổi trẻ” cóđăng “Mởrộng bấtđẳng thức” tác giảTạHồng Quảng, mở đầu báo tác giảcó viết: Trên báo Gazeta Matematica Romania số9; 1989trang335tác giảGheoghe Marghescuđưa bấtđẳng thức sau

Bài toán Cho sốthựca1; a2; : : : ; anb1; b2; : : : ; bn; n!2/chứng minh rằng n

X

iD1

q

ai2Cb2i

! n X

iD1

1 Cbi

!

!npn: (7)

Sauđó tác giảTạHồng Quảngđãđưa toán bấtđẳng thức chặt (7) nhưsau Bài toán Cho sốthựca1; a2; : : : ; anb1; b2; : : : ; bn n!2/chứng minh rằng

n

X

iD1

q

ai2Cbi2

! Xn iD1

1 Cbi

!

! pn2

2: (8)

(85)

Bài toán Cho sốthựca1; a2; : : : ; anb1; b2; : : : ; bn; n!2/chứng minh rằng B B @ n X

iD1

v u u

tqa2i Cb2i Cbi

1 C C A

2

! pn2

2: (9)

Lời giải Dễthấy vớiai; bi > 0; 8i D1; nthì

q

a2i Cbi2

ai Cbi !

1

p

2;

suy v

u u

tqa2i Cbi2

ai Cbi !

1 p 2: Cho nên n X

iD1

v u u

tqa2i Cbi2

ai Cbi !

n

4

p

2;

tươngđương 0

B B @

n

X

iD1

v u u

tqa2i Cb2i Cbi

1 C C A

2

! pn2

2:

Đẳng thức xảy chỉkhiai Dbi;8i D1; n:

Quảthật theođánh giá chủquan (9) tốn hay, tốn nàyđược tạo thành túy chỉgồm phép cộng vếvới vếcủanbấtđẳng thứcđơn giản chiều, sau

đóđem bình phương cảhai vếcủa bấtđẳng thứcđó lại mà Thếnhưng (9) lại chặt (8) vàđương nhiên chặt (7), theo bấtđẳng thức Cauchy-Schwarz ta có

n

X

iD1

q

a2i Cb2i

! n X

iD1

1 Cbi

! ! B B @ n X

iD1

v u u

tqai2Cbi2 Cbi

1 C C A

2

! pn2

2:

Bài toán Cho sốthựca1; a2; : : : ; anb1; b2; : : : ; bn; n!2/:Chứng minh rằng n

X

iD1

q

ai2Cbi2

! n X

iD1

1 aiCbi

!

! pn2

2 C B B @ n X

iD1

v u u

tqa2i Cb2i

ai Cbi #

(86)

Lời giải Vớiai; bi > 0; 8i D1; nthì

q

a2 i Cbi2

ai Cbi !

1

p

2;

dođó tồn tại˛i !0để q

a2i Cbi2

ai Cbi D

% p &2

C.˛i/2 (10)

Theo bấtđẳng thức Cauchy-Schwarz n

X

iD1

q

a2 i Cbi2

! Xn iD1

1 Cbi

! ! B B @ n X

iD1

v u u

tqai2Cbi2

ai Cbi

1 C C A D 4Xn

iD1

s% p &2

C.˛i/2

3

2

:

(11) Mặt khác theo (5), vếphải (11) tađược

2 4Xn

iD1

s% p &2

C.˛i/2

3 ! n X

iD1

1 p !2 C n X

iD1

˛k !2 D n p C n X

iD1

˛i

!2

;

suy tiếp 2

4Xn

iD1

s% p &2

C.˛i/2

3

2

! pn2

2 C

n

X

iD1

˛i

!2

:

Từ(10) tađược

˛i D

v u u

tqa2i Cbi2

ai Cbi #

1

p

2; 8i D1; n;

n

X

iD1

q

ai2Cbi2

! Xn iD1

1 aiCbi

!

! pn2

2 C B B @ n X

iD1

v u u

tqa2i Cb2i

ai Cbi #

1 p C C A :

Đẳng thức xảy chỉkhia1 D a2 D " " " D an D b1 D b2 D " " " D bn:Bài toán chứng minh

4 Làm ch

t m

t toán b

t

đẳ

ng th

c thi Toán h

c

Qu

c T

ế

(87)

Bài toán.Cho sốthực dươngx; y; z:Chứng minh rằng x

p

x2C8yz C

y

p

y2C8zx C

z

p

z2C8xy !1:

Lời giải Với ba sốthực dươnga; b; c áp dụng bấtđẳng thức AM-GM tađược

.a4Cb4Cc4/2 !.a4C2b2c2/2 Da8C4a4b2c2C4b4c4 !a8C8a2b3c3; (12)

suy

1

a6C8b3c3 !

a2

.a4Cb4Cc4/2;

hay s

a6

a6C8b3c3 !

a4

a4Cb4Cc4:

Tương tự s

b6

b6C8c3a3 !

b4 a4Cb4Cc4;

s

c6

c6C8a3b3 !

c4 a4Cb4Cc4: Dođó

a3

p

a6C8b3c3 C

b3

p

b6C8c3a3 C

c3

p

c6C8a3b3 !

a4Cb4Cc4 a4Cb4Cc4 D1:

Cuối thayx Da3; y D b3; z Dc3 ta sẽcóđiều phải chứng minh Dấuđẳng thức xảy ra chỉkhix Dy Dz:

Đếnđâyđộc giả thấy toán bấtđẳng thức thi IMO2001được chứng minh thông qua (12)

Bây giờta làm chặt (12) lặp lại cách làm tương tự, chuỗi bấtđẳng thức chặt bấtđẳng thức IMO2001sauđây

Bài toán.Cho sốthực dươngx; y; z:Chứng minh rằng

X x

p

x2C8yz !

X x

q

x2C.y23 Cz 3/

3 !

X x

q

x2C2.yCz/2

!Xq x

x2C2y2C2z2C2y34z23 C2y23z43

!1:

(88)

tađược

"

a4Cb4Cc4#2Da8C"a4b4Cb8#C"a4c4Cc8#C"a4b4Cb4c4#C"a4c4Cb4c4#

!a8C2a2b6C2a2c6C2a2b4c2C2a2b2c4 !a8C2a2b6C2a2c6C4a2b3c3

Da2ha6C2"b3Cc3#2i!a2ha6C"b2Cc2#3i !a2!a6C8b3c3$:

Suy

1

a6C8b3c3 !

1

a6C.b2Cc2/3 !

1

a6C2.b3Cc3/2

!

a6C2b6C2c6C2b4c2C2b2c4 !

a2

.a4Cb4Cc4/2; tươngđương với

s

a6

a6C8b3c3 !

s

a6

a6C.b2Cc2/3 !

s

a6

a6C2.b3Cc3/2

!

s

a6

a6C2b6C2c6C2b4c2C2b2c4 !

a4 a4Cb4Cc4/

(13)

Tương tựthì

s

b6

b6C8c3a3 !

s

b6

b6C.c2Ca2/3 !

s

b6

b6C2.c3Ca3/2

!

s

b6

b6C2c6C2a6C2c4a2C2c2a4 !

b4

.a4Cb4Cc4/:

(14)

s

c6

c6C8a3b3 !

s

c6

c6C.a2Cb2/3 !

s

c6

c6C2.a3Cb3/2

!

s

c6

c6C2a6C2b6C2a4b2C2a2b4 !

c4

.a4Cb4Cc4/:

(15)

Cộng tươngứng vếcủa (13), (14), (15) rút gọn vàđặtx Da3; y Db3; z Dc3tađược

X x

p

x2C8yz !

X x

q

x2C.y23 Cz23/3

!Xq x

x2C2.yCz/2

!Xq x

x2C2y2C2z2C2y43z C2y

2 3z

4

!1:

(89)

5 Gi

i thi

u nh

ng toán m

i

đượ

c làm ch

t t

nh

ng

bài toán

đ

ã bi

ế

t

Bài toán Choa1; a2; : : : ; anlà sốthực dương Chứng minh rằng

a1Ca2C" " "Can! pn a1a2" " "an

%

n

r

a1

a2 C

n

r

a2

a3 C" " "C

n r an a1 & :

Bài toán Choa1; a2; : : : ; anlà sốthực không âm Chứng minh rằng

%

a1Ca2C" " "Can

n

&n

!an

%

a1Ca2C" " "Can!1

n#1

&n!1

:

Bài toán Cho sốthựca1; a2; : : : ; anb1; b2; : : : ; bn; n!2/khiđó n

X

iD1

a2i

! n X

iD1

bi2

!

!

" n X

iD1

"

ai2Cbi2#

# " n X

iD1

a2 ibi2

ai2Cbi2

#

!

n

X

iD1

aibi

!2

C 14

2 4Xn

iD1

ˇˇ ˇˇ ˇˇ

s"

ai2Cbi2# "a2iC1bi2C1# a2iC1Cbi2C1 #

s"

a2iC1Cbi2C1# "ai2bi2# ai2Cb2i

ˇˇ ˇˇ ˇˇ ! n X

iD1

aibi

!2

;

quyướcanC1 Da1; bnC1 Db1:

Bài toán Cho ba sốthực dương a; b; c thỏa mãnabc D 1:Chứng minh với số

nguyên dươngk!3ta ln có

ak.bCc/ C

1

bk.cCa/ C

1

ck.aCb/ !

3 2:

Bài toán Cho ba sốthực dươnga; b; c và hai sốnguyên dươngp !2q:Chứng minh rằng a

p

p

.a2C8bc/q C

b

p

p

.b2C8ca/q C

c

p

p

.c2C8ab/q !.aCbCc/

p!2q p :

Bài toán 10 Với sốthựca1; a2; : : : ; anb1; b2; : : : ; bn; n!2/ta có n

X

iD1

q

a2i Cb2i

! Xn

iD1

1 Cbi

!

!

"Xn

iD1

%q

a2i Cbi2C

1 Cbi

&#26

n

X

iD1

%q

a2i Cbi2& 'a

iCbi

( %q

a2i Cbi2C a

iCbi

& 7 ! B B @ n X

iD1

v u u

tqa2i Cbi2 Cbi

1 C C A

2

! pn2

2C 6 n X

iD1

v u u

tqa2i Cbi2

ai Cbi #

1 p 7

! pn2

2 !n

p

(90)

Bài toán 11 Cho số thực a1; a2; : : : ; an b1; b2; : : : ; bn; n ! 1/ khi đó giả sử

c1; c2; : : : ; cnlà hốn vịnàođó sốa1; a2; : : : ; an:Chứng minh rằng n

X

iD1

q

a2i Cbi2

! n X

iD1

1 ci Cbi

!

! pn2

2:

Bài toán 12 Ta chia sốthựca1; a2; : : : ; an b1; b2; : : : ; bn; n !1/làmnnhóm cho

trong nhóm có số Gọi M1; M2; : : : ; Mn lần lượt tổng số trong mỗi

nhóm Chứng minh rằng

n

X

iD1

q

ai2Cb2i

! n X

iD1

1 Mi

!

! pn2

2:

Bài toán 13 Choai; bi > 0; 8i D1; n:Chứng minh rằng n

X

iD1

q

a2i Cbi2

! n X

iD1

1 Cbi

!

! p1

2

2 Xn

iD1

r a

i

ai Cbi

!2

C

0 @Xn

iD1

s

bi

ai Cbi

1 A

23

5:

Bài toán 14 Choai; bi; ci > 0; 8i D1; n:Chứng minh rằng n

X

iD1

q

a2i Cbi2Cc2i

! n X

iD1

1 Cbi Cci

!

! pn2

3:

Bài toán 15 Choai; bi; ci > 0; 8i D1; n:Chứng minh rằng

n

X

iD1

q

a2i Cbi2Cci2

! n X

iD1

1 Cbi Cci

!

! pn2

3 C B B @ n X

iD1

v u u

tqa2i Cbi2Cci2 Cbi Cci #

1 p C C A :

Bài toán 16 Choai; bi; ci > 0; 8i D1; n.x1; x2; : : : ; xn/ ; y1; y2; : : : ; yn/ ; z1; z2; : : : ; zn/

lần lượt tươngứng hoán vịtùyýcủa.a1; a2; : : : ; an/ ; b1; b2; : : : ; bn/ ; c1; c2; : : : ; cn/ :

Chứng minh rằng

n

X

iD1

q

a2i Cbi2Cci2

! n X

iD1

1 xi Cyi Czi

!

! pn2

3:

Bài toán 17 Choai; bi > 0; 8i D1; n:Chứng minh rằng

"Xn iD1

"

ami Cbim#

# "Xn iD1

1 Cbi/m

#

! n2

2m!1: Bài toán 18 Choai; bi > 0; 8i D1; n:Chứng minh rằng

n

X

iD1

m

p

ami Cbim

ai Cbi !

n

m

p

(91)

Bài toán 19 Choai; bi > 0; 8i D1; n:Chứng minh rằng

" n X

iD1

.ai Cbi/

# " n X

iD1

1

m

pa

i C mpbi

#m

! nmC1

2m!1: Bài toán 20 Choai; bi > 0; 8i D1; n:Chứng minh rằng

n

X

iD1

m

q

ami Cbim!

npm 2m!1

2 Xn

iD1

pa i !2 C n X

iD1

p

bi

!23

5:

Bài toán 21 Choai; bi > 0; 8i D1; n:Chứng minh rằng n

X

iD1

m

q

ami Cbim!

.npm 2/m!1

" n X

iD1

m p !m C n X

iD1

m

p

bi

!m#

:

Bài toán 22 Cho sốthực dươnga1; a2; : : : ; an; n!1/:Chứng minh rằng n

Y

iD1

"

ainC2#ai2Cn# !

n

X

iD1

ai

!n

:

Bài toán 23 Cho sốthực dươnga1; a2; : : : ; an; n!1; k!2/:Chứng minh rằng

ak1C1 ak2

C a

kC1

ak3

C" " "C a

kC1 n

a1k

! a12

a2 C

a22

a3 C" " "C

a2 n

a1

:

Bài toán 24 Cho số thực dương x1; x2; : : : ; xn; n ! 1; k ! 2/có tổng bằng 1: Với

˛; ˇ;"!0thỏa mãn˛2Cˇ2C"2 > 0:Chứng minh rằng xk

1

˛x1Cˇx2C"x3 C

xk

˛x2Cˇx3C"x4 C" " "C

xk n

˛xnCˇx1C"x2 !

1

nk!2.˛CˇC"/: Bài toán 25 Cho ba sốthực dươnga; b; c và sốnguyênk !3:Chứng minh rằng

a4C2b2c2

.a2C2bc/k C

b4C2c2a2

.b2C2ca/k C

c4C2a2b2

.c2C2ab/k !

3k!2

.aCbCc/2k!4:

Bài toán 26 Với sáu sốthực dươnga; b; c; x; y; zvà sốnguyên dươngk !2thì ak x C bk y C ck z !

.aCbCc/k 3k!2.xCyCz/:

Bài toán 27 Cho ba sốthực dươnga; b; c và sốnguyênn!2:Chứng minh rằng anC1

bCc C

bnC1

cCa C

cnC1

aCb !

.aCbCc/n!1 2"3n!2 "

n

r

anCbnCcn

3 :

Bài toán 28 Cho ba sốthực dươnga; b; c plà sốnguyên dương Chứng minh rằng a

p

p

bC2c C

b

p

p

cC2a C

c

p

p

aC2b !.aCbCc/

(92)

Bài toán 29 Cho ba sốthực dươnga; b; c:Chứng minh rằng

r

a4

b2c2 C2C

r

b4

c2a2 C2C

r

c4

a2b2 C2!3

p

3:

Bài toán 30 Vớiklà sốtựnhiên ba sốthực dươngxk; yk; zk; ˛; ˇthỏa mãn n

X

kD1

xk D

n

X

kD1

yk D

n

X

kD1

zk D1:

Chứng minh với sốnguyênp !2ta có

n

X

kD1

.1Cxk/p

˛yk Cˇzk !

2p np!2.˛Cˇ/:

Bài toán 31 Choak; bk > 0; k D1; nvà sốnguyên dươngm:Chứng minh rằng n

X

kD1

m

q

amk Cbmk

! Xn kD1

1 akCbk

!

! pm n2

2m!1:

Tài li

u

[1] Nguyễn Văn Mậu, Bấtđẳng thứcđịnh lí áp dụng, NXB Giáo dục,2006:

[2] Phạm Kim Hùng, Sáng tạo bấtđẳng thức, NXB Tri thức,2006:

[3] Phạm Văn Thuận, Lê Vĩ, Bấtđẳng thức suy luận khám phá, NXBĐHQG Hà Nội,2007:

(93)

B

Y TR

C

T THÔNG THÁI C

A

T

H

NG KÊ H

C

Nguy

n V

ă

n Tu

n

G

IỚI THIỆU

Chuyên mụcđiểm sách kỳnày Epsilon vinh dựnhậnđược viết từgiáo sư Nguyễn Văn Tuấn, với điểm "Bảy trụcột thông thái thống kê học" ("The Seven Pillars of Statistical Wisdom") tác giảStephen M Stigler.Đây sách mang lại góc nhìn lýgiải vềnhững phương pháp thống kê đãđược sửdụng xuyên suốt chiều dài lịch sửcủa thống kê học Chúng chủquan cho tác phẩm sẽgiúp cho ngườiđọc hiểu sâu vềcác phương pháp thống kê, qua đó, giúp khơng chỉdùng thống kê nhưnhững cơng cụ, mà cịn hiểuđượcđó cảmột hệthống tưduy Xin mờiđộc giả đến với bàiđiểm sách giáo sưNguyễn Văn Tuấn

Một sách khoa học mà tơi thích vài tháng gần "The Seven Pillars of Statistical Wisdom" tác giảStephen M Stigler (1).Đây sách nhỏ(200 trang) cung cấp cho lí giải lí thú vềkhoa học thống kê lịch sử đằng sau phương pháp mà sửdụng suy luận khoa học Nhưtựađềcuốn sách, tác giảStigler tập trung vào giải thích trụcột thơng thái thống kê học, tơi thửtóm lược theo cách hiểu tơi dướiđây

Nhưng trước giải thích, tơi thấy cần phải dành vài chữgiải thíchýnghĩa chữwisdom, mà tơi thấy khó dịch sang tiếng Việt Ởmứcđộcđơn giản nhất, wisdomlà thơng thái, khơn ngoan Nhưng có kinh nghiệm cọsát với xã hội nói tiếng Anh hình nhưchữ"thơng thái" "khơn ngoan" có vẻkhơng tươngđương với wisdom Trong ngữ cảnh tựađềcuốn sách này, hiểuwisdomnhưlà tri thức trải nghiệmđượcđúc kết qua trải nghiệm thực tế, giống nhưnhững câu ca dao tinh tuývề ứng xử đời mà cha ông chúng tađãđúc kết truyền lại

Đểhiểu khái niệm sách, cần phải phân biệt dữliệu (data) thơng tin (information) Dữ liệu thu thập từ nghiên cứu Đểchuyển hố dữliệu thành thơng tin, phải áp dụng phương pháp phân tích thống kê Nói cách khác, thống kê học cơng cụ đểchúng ta thu nạp thông tin từdữliệu Dĩnhiên, từthông tin, có thểbiến thành kiến thức (knowledge) qua dùng phương pháp qui nạp khoa học

(94)

Tr

c

t - aggregation: Qui lu

t lo

i b

d

li

u

để

thu

n

p thông tin

Trong phần này, Stigler lí giải vàđưa nhận xét làm ngạc nhiên:đó thu nạp kiến thức cách loại bỏthông tin! Chẳng hạn đối phó với dãy sốliệu chiều cao, cần tính số trung bình, dùng thông tin để kiến tạo tri thức Cịn tất cảnhững số đểtạo nên sốtrung bình bịloại bỏ, khôngđượcđềcập đến Mỗi ngày, chúng tađọc nghe sốtrung bình, từthịtrường chứng khốn, sách kinh tế, đến nghiên cứu y khoa, tất dùng sốtrung bìnhđể điđến quyếtđịnh phức tạp

Lịch sửvà sựrađời sốtrung bình cũngđược tác giảdiễn giải tường tận Thống kê học, hay khái niệm thống kê học,đãđược sửdụng thiên văn học từ thếkỉ18, phải đợi đến thếkỉ19 thịnh hành Lí vấnđề đo lường liên quanđến giá trịtrung bình Tác giảStigler chỉra đo lường [chẳng hạn như] Sao Mộc, biết rõđó thực thể, nóở vị trí ước tính sai số Nhưng chúng tađo lường tuổi thọhay mứcđộlạm phát kinh tế, khơng cóđược "xa xỉ" đo lường Sao Mộc, biến sốnhư tuổi thọ xuất phát từ mẫu mà cóđược không biếtđược giá trịthật quần thể Người có cơngđầu việc phát kiến trịsốtrung bình Nhà khoa học người BỉAdolphe Quetelet (người sáng tạo chỉsố body mass index) Vào năm 1831, Quetelet "sáng chế" mà ơng gọi "L’homme Moyen" (người trung bình) Người trung bình cá nhân hưcấu, với giá trị trung bình mà có thểsửdụng để đại diện nhóm người Dođó, Quetelet tính chiều cao trọng lượng trung bình nhóm lính Pháp, xem người lính tiêu biểu Nhưng Quetelet hiểu trịsốtrung bình sẽdao động nhóm lính, ơng bàn độ xác nhưcách tính Từ đó, khoa học thống kê có giá trịmà sau trởthành phổbiến vàđược áp dụng hầu nhưbất cứlĩnh vực xã hội Cái bất ngờmà tác giảStigler chỉra có giá trịtiêu biểu cách loại bỏdữliệu!

Tr

c

t - information: Qui lu

t gi

m l

ượ

ng thông tin

Giảdụnhưnếu chúng taước tính sốtrung bình quần thểdựa 100đối tượng (và gọi x1), sốtrung bình dựa 200đối tượng (x2), câu hỏiđặt giá trịcủa thông tin x2 cao gấp lần so với giá trịthông tin x1? Câu trảlời không Trong thực tế, tăng lượng dữliệu gấp lần giá trịthông tin chỉtăng khoảng 1.4 lần Nếu tăng lượng liệu gấp lần lượng thơng tin chỉtăng 1.7 lần

Từ đâu mà có số đó? Tác giảchỉra sựthật hiển nhiên từcơng thức tính sai sốchuẩn (standard error) Sai sốchuẩn bằngđộlệch chuẩn chia cho sốbậc sốcỡmẫu; hay nói cách khác,độlệch chuẩn sai sốchuẩn nhân cho sốbậc lượng dữliệu Chẳng có mớiở đây, De Moivređã chỉra từ1738, vàđó lí thuyếtđằng sauĐịnh lí giới hạn trung tâm (Central Limit Theorem)

(95)

người tin vào Dữliệu Lớn nghĩrằng cách tăng lượng dữliệu sẽcó thơng tin xác hơn,đáng tin cậy hơn.Đúng chưađủ, lượng thơng tin khơng phải hàm sốtuyến tính lượng dữliệu

Tr

c

t - likelihood: Thu n

p thông tin t

tình tr

ng

b

t

đị

nh

Trong chương này, tác giảStigler bàn vềlí thuyết khảdĩ(Likelihood) Trong phần này, tác giả Stigler lí giải thu nạp thơng tin từdữ liệu qua phương pháp kiểm định thống kê (test of significance) trịsố P mà Ronald Fisher đềxướng từ năm 1925, với phương pháp sau nhưkhoảng tin cậy 95% Các phương pháp cũngđã giúp giảm bất định sống Kiểm định thống kê mà Ronald Fisher đề xướng làýtưởng mới, phương cách nàyđãđược John Arbuthnot áp dụng trướcđóđểtính tốn xem tượng sinh trai nhiều gái doýcủa Thượngđếhay ngẫu nhiên!

Tr

c

t - intercomparison: So sánh

Bất cứai làm nghiên cứu khoa học cần so sánh Thường so sánh hai nhóm xem có khác cách có hệthống hay khác biệt chỉlà yếu tốngẫu nhiên Phương pháp kiểmđịnh t (do William Gosset đề xướng) phương pháp quen thuộc Một phương pháp so sánh khác hay áp dụng phân tích phương sai (ANOVA hay analysis of variance) Ronald Fisher phát kiến Trong ngành khác, người ta so sánh với chuẩn vàng (gold standard), khoa học thống kê so sánh thông tin dữliệu, nghiên cứu Những đọc sách sửthống kê học biết William Gosset làm việc cho hãng bia Guiness, cơng việc ơng lúcđó kiểm nghiệm chất lượng bia Trong nhiệm vụ đó, ơng phải làm nghiên cứu thường dựa sốmẫu nhỏ, "cái khó ló khơn", ơngđã sáng chếra phương pháp so sánh khác biệt dựa vào cỡmẫu nhỏ Trong dịp nghỉhè (sabbatical) ông thăm labo Karl Pearson University College London, viết báo tiếng Khi báo gửi cho tập san thống kê học, ơng khơng kí tên thật (vì nhân viên Guiness), nên phải kí bút danh "Student" Từ đó, khoa học có phương pháp kiểmđịnh gọi "Student’s test"

Tr

c

t - regression: Thu n

p thông tin t

qui lu

t h

i

qui v

s

trung bình

(96)

trứ danh Francis Galton đề xướng từ cuối kỉ 19 Lúc đó, Galton nghiên cứu ảnh hưởng di truyền đến trí thông minh, ông dùng chiều cao marker Ông quan sát cặp cha mẹcó chiều cao thấp trung bình thường sinh có chiều cao cao cha mẹ; ngược lại, cặp vợchồng có chiều cao cao thường sinh có chiều cao thấp họ Đây tượng hồi qui sốtrung bình, hay thuật ngữ tiếng Anh "regression to the mean"

Đây chương hay sách Tác giả bắt đầu với câu chuyện Charles Darwin, người em họcủa Francis Galton Darwin người khơng thích tốn, khơng phải ơng khảnăng vềtốn, mà ơng cho tốn khơng giúp cho khoa học! Darwinđề raýtưởng gọi "The Rule of Three" hay "Qui luật tam suất" Ơng nói biết a/b = c/d biết số, có thểxác định số thứ Nhưng thực tế, cần nhiều tập hợp giá trị để ước tính tham sốcủa mơ hình hồi qui tuyến tính Nhưngýtưởng vềphân loại chủng vật Darwin sau lại nhờcác phương pháp phân tích đa biến giúpđỡrất nhiều Tất phương pháp phân tíchđa biếnđều xuất phát từ mơ hình hồi qui tuyến tính

Tr

c

t - design: Thu n

p thông tin t

s

ng

u nhiên

Ngạc nhiên thay, thu nạp thông tin cơchế ngẫu nhiên hố.Ýnghĩa trụcột cần vài lời giải thích Thiết kế phương pháp thu nạp thông tin có hệthống Nhưng thiết kếlà phải dùngđến cơchếngẫu nhiên hố (randomization) Chẳng hạn muốn đánh giá hiệu thuốcđiều trịbệnh, phải chia nhóm bệnh nhân cách ngẫu nhiênđể đảm bảo nhómđều có yếu tốnhiễu giống

Thật ra, có thểxem thiết kếnghiên cứu cách tốiưu hoá Trong chương này, tác giảStigler dìu dắt qua lịch sử thiết kế nghiên cứu thú vị Ý tưởng thiết kế nghiên cứu để thu nạp thông tin xuất phát từ sổ sốbên Pháp vào năm 1757 Sổ số lúcđó đóng góp 4% cho ngân sách Pháp (có lẽ giống nhưsổsốtràn lan nayở Việt Nam) Sau Ronald Fisher làm việc cho Trạm thí nghiệm Rothamsted, ơng nghĩ phương pháp chia nhóm ngẫu nhiên (randomization) Trong diễn thuyết Hội nghịThống kê họcẤn Độnăm 1938, Fisher tuyên bốrằng"To consult the statistician after an experiment is finished is often merely to ask him to conduct a post mortem examination He can perhaps say what the experiment died of"(Tưvấn nhà thống kê học sau thí nghiệmđã làm xong có thểví von hỏi nhà thống kê học làm giảo nghiệm tửthi Nhà thống kê học có thểnói thí nghiệm chết lí gì.)Ýcủa Fisher muốn thí nghiệm có kết quảtốt muốn thu nạp thơng tinđáng tin cậy phải tưvấn nhà thống kê học trước làm thí nghiệm – lời khuyên cịnýnghĩa thời sựngày hơm Nhưng việc nàyđịi hỏi nhà thống kê phải hiểu vấnđềkhoa học hiểu qui trình suy luận khoa học

Tr

c

t - residual: Thu n

p thông tin t

sai s

(97)

thường từnhững dữliệu bất bình thường Cái bất bình thườngở residuals, sai số từ mơ hình Chẳng hạn đểphát gen có liên quanđến bệnh lí, mơ hình phân bốcủa gen dựa vào qui luật sinh học, dữliệu nằm hay lệch so với giá trịkì vọng genđáng quan tâm Nhưvậy, phát cơchếsinh học từnhững dữliệu thông tin bất thường

Điều cóýnghĩa quan trọng cho nhà thống kê học.Đối với nhà thống kê họcđược huấn luyện cácđại học mà khơng có tương tác với khoa học, phân tích daođộng dưlàđể kiểmđịnh tính hợp lí mơ hình họ Nhưngđối với khoa học,điềuđó chẳng quan trọng; điều quan trọng dữliệu mà mơ hình khơng giải thíchđược

Cuốn sách viết với văn phong khoa học hấp dẫn với người ngồi khoa học Nhưng thỉnh thoảng, tác giảcó vẻgiả định ngườiđọc phải hiểu sốkhái niệm thống kê học Chẳng hạn nhưngườiđọc phải "động não"đểhiểu

L(Θ) = L(Θ)|XandCov(L, W) =E{Cov(L, W|S)}+Cov(E{L|S}, E{W|S})

Nhưng may mắn thay, công thức loại chỉxuất vài lần sách, ngườiđọc không cần hiểu chúng mà nắmđượcýnghĩađằng sau mơ hình thống kê

Tóm lại, sách"The Seven Pillars of Statistical Wisdom"là tác phẩm hay vàđáng đọc Tác phẩm tác giả đãđemđến cho nhìn tươi, với lí giải vềnhững phương pháp thống kê cổ điển Chỉtrong 200 trang sách mà tác giả lược qua điểm (7điểm) suốt chiều dài lịch sử chuyên ngành khoa học thống kê Cuốn sách giúp cho chúng ta, người làm nghiên cứu thực nghiệm, hiểu sâu phương pháp thống kê, qua giúp suy nghĩmột cách thống kê, suy nghĩnhưlà công cụ Xin nhấn mạnh: xem thống kê học cách suy nghĩ Thay tập trung vào chi tiết tính tốn, cần phải hiểu ýnghĩađằng sau phương pháp mơ hình thống kê Nếu bạn nhà thống kê học, nhà khoa học thực nghiệm, sách"The Seven Pillars of Statistical Wisdom"phải có tủsách bạn Ghi thêm:

(98)

M

T S

BÀI TỐN TRÊN TÂM

ĐƯỜ

NG TRỊN

E

ULER

Tr

n Quang Hùng

GI

ỚI THIỆU

Bài viết xoay quanh sốbài tốn liên quan tới tâmđường trịn Euler Các toán kết quảcủa tác giảtrong trìnhđi tập huấn số đội tuyển thi HSG cảnước Thời gian gầnđây, nhiều tốn thú vịtrên tâmđường trịn Euler

đường trịn Euler liên tụcđược phát hiện, trongđó thân tác giảcũng tìm rađược sốbài tốn vềchủ đềnày Mặt khác quãng thời gian tác giảgiảng dạy toán cách thờiđiểm cũngđã lâu, nhiên sốbài tốn có nhiều giá trị

Đặc biệt tốnđó sử dụng tốt việc bồi dưỡng em học sinh yêu hình học Dođó tơi xin trân trọng giới thiệu với bạnđọc viết tổng hợp

1 M

t s

bài tốn ví d

Đường trịn Euler hay tên quốc tế thường gọi đường tròn 9điểm [7], đường tròn qua trung điểm ba cạnh, chân bađường cao trung điểm ba đoạn thẳng nối trực tâm ba đỉnh tam giác.Đây kết quảrất tiếng vàđược khai thác nhiều toán khác Tâmđường tròn mộtđềtài thú vịcủa nghiên cứu hình học sơcấp

đềtài hay dành cho thi học sinh giỏi toán nước quốc tế Phần Tơi xin trình bày lại số tốn chủ yếu tơiđề xuất liên quanđến tâm đường tròn thú vị khoảng thời gian từnăm 2013-2014

Kỳthi học sinh giỏi tốn quốc gia năm 2013 có tốn nhưsau [1]

Bài toán 1 (VMO 2013). Cho tam giác ABC, đường tròn nội tiếp (I)tiếp xúc CA, AB lần

lượt tại E, F. G, H lần lượt làđối xứng của E, F qua I. Đường thẳng GH giao IB, IC lần

lượt tạiP, Q Giảsử B, C cố định, Athayđổi cho tỷsố AB

AC = k khôngđổi Chứng minh

rằng trung trựcP Qluônđi qua mộtđiểm cố định.

Lời giải. GọiIB, IC cắtEF tạiK, L Chúýtam giácAEF cân tạiAnên∠KEC =

∠AEF = 180◦2−∠A = 180◦−(90◦ − ∠A

2 ) = 180◦ −∠BIC = ∠KIC Từ tứgiácKEIC nội tiếp suy ra∠IKC =∠IEC = 90◦ Tương tự∠ILB−90◦ Từ đó nếu gọiM là trungđiểm

BC,J trungđiểmKLđễcó tam tam giácKLM cân nênM J ⊥EF (1)

DoG, H làđối xứng củaE, F quaI nênđường thẳngGH đối xứngđường thẳngEF

(99)

A

B D C

I

E

G F

H

P Q

N

K

L J

M R

QL Vậy haiđoạnKLvàP Qđối xứng quaI Từ gọiRlà trungđiểmP Qthì trung

điểmJcủaKLvàRđối xứng quaI hayI trungđiểmRJ

Gọi trung trựcP QcắtBCtạiN, ta thấyRN vuông gócP Q,P Qsong songEF (2)

Từ(1)và(2)suy raRN song songJM GọiIAcắtBCtạiD, dễcóID≡IAvng gócEF

nênIDcũng song song vớiRN, JM Từ hình thangRJM N cóI trungđiểmRJ

nênIDlàđường trung bình, vậyDlà trungđiểmM N

Theo tính chấtđường phân giác DB DC =

AB

AC =kkhôngđổi nênDcố định.M trungđiểmBC

cố định nênN đối xứngM quaDcố định Vậy trung trựcP Qđi quaN cố định

Nhận xét.Việc dựng thêm cácđiểm phụL, Ktrong lời giảiđóng vai trị quan trọng, cho

phép ta sửdụng phép vịtự để chuyển tính chấtđường thẳngRN JM cho Trong

q trình tìm hiểu tốn tơi nhận thấy thực chấtK, L, N chânđường cao từC, B, I

của tam giácIBC Nhưvậyđường tròn ngoại tiếp tam giácN KLlàđường tròn Euler tam

giácIBC nên cũngđi quaM tâmđường trịn Euler tam giácIBC nằm

trung trựcKLcũng làđường thẳngJM Vậy từtâmđường tròn Euler tam giácIBC

mà ta vẽ đường thẳng song song với IA trung trực KL màđi qua trung

điểmBC Ta dễthấy đường thẳng qua trungđiểm BC, CA, AB mà song song

vớiIA, IB, IC thìđồng quy Dođó tađềxuất tốn nhưsau

Bài toán 2. Cho tam giácABCtâm nội tiếpI GọiNa, Nb, Nclần lượt tâmđường tròn Euler

của tam giácIBC, ICA, IAB thì cácđường thẳng quaNa, Nb, Nc lần lượt song song với

IA, IB, IC đồng quy.

Qua tìm hiểu khai thác tơi nhận tốn nàyđúng khơng chỉvới tâm nội tiếpI mà

thực chất nóđúng với mọiđiểmP bất kỳtrong mặt phẳng Dođó tơiđềxuất tốn sau

Bài tốn 3. Cho tam giác ABC P là một điểm Gọi Na, Nb, Nc lần lượt tâm

(100)

A

B C

P

Oa Ob Oc

Ga

Na K

G N

Ma

Lời giải. GọiOa, Ob, Oc tâmđường tròn ngoại tiếp tam giácP BC, P CA, P AB

Ta dễthấyđường thẳng quaOa song songP Achính làđường cao từOa tam giácOaObOc

do cácđường thẳng quaOa, Ob, Oc song song với P A, P B, P C đồng quy trực

tâmKcủa tam giácOaObOc

GọiGa, Gb, Gc, Glần lượt trọng tâm tam giácP BC, P CA, P AB, ABC ta dễthấyP Gavà

AGđi qua trungđiểmMacủaBC, từ dễthấyGaGsong songP Anói cách khác cácđường

thẳng qua Ga, Gb, Gc song song vớiP A, P B, P C đồng quy trọng tâm Gcủa tam

giácABC

Đến lại ý Na tâmđường trịn Euler tam giácP BC nên dễcó 2−−−→GaNa +

−−−→

GaOa =−→0 Từ sửdụng phép chiếu song song phươngP Axuốngđường thẳngKG, gọiN

là hình chiếu song song phươngP AcủaNaxuốngKGta dễsuy ra2−−→GN+−−→GK =−→0 nói cách

khác đường thẳng qua Na song song P A quaN xác định Tương tự đường thẳng qua

Nb, Nc song song vớiP B, P C cũngđi quaN ta cóđiều phải chứng minh

Nhận xét.Bài toán trường hợp tâm nội tiếp tốn cóýnghĩa, việc tổng qt tốn vớiđiểmP bất kỳrồi sauđóđặc biệt hóa choP di chuyển trùng sốtâmđặc biệtđểtạo

bài toán việc làm thú vị Qua toán ta dễrút tâm đường tròn Euler chỉlà trường hợpđặc biệt Tương tư nhưcách chứng minh tốn với trực tâm tổng quát điểm trênđường thẳng Euler chia đoạn nối trực tâm, trọng tâm tỷsốcố định Ta có tốn khác nhưsau

Bài toán 4. Cho tam giácABC P là mộtđiểm GọiHa, Hb, Hc lần lượt trực tâm

của tam giác P BC, P CA, P AB Chứng minh các đường thẳng qua Ha, Hb, Hc lần

lượt song song vớiP A, P B, P Cđồng quy.

Bài toán 5. Cho tam giác ABC P là một điểm Gọi Ha, Hb, Hc lần lượt trực

tâm tam giácP BC, P CA, P AB GọiGa, Gb, Gc lần lượt trọng tâm tam giác

P BC, P CA, P AB GọiLa, Lb, Lc lần lượt chia các đoạnHaGa, HbGb, HcGc cùng tỷsố.

Chứng minh các đường thẳng qua La, Lb, Lc lần lượt song song với P A, P B, P C đồng

(101)

Một khai thác tương tự tác giả đềnghịtrong thi giải toán mathley [3], bạn làm nhưbài luyện tập

Bài toán 6. Cho tam giácABCDEF nội tiếpđường tròn(O) GọiNa, Nb, Nclà tâmđường

tròn Euler tam giácDBC, ECA, F AB Chứng minh cácđường thẳng quaNa, Nb, Nc

lần lượt song songAD, BE, CF đồng quy.

Sau đề xuất tốn 3, tơiđã mạnh dạn nghĩtới kết quảthay đường thẳng song song

đường thẳng vuông góc thật tuyệt vời tốn vng gócđúng, toán phát biểu

sau

Bài toán 7 (Kiểm tra đội tuyển THPT chuyên KHTN năm 2014). Cho tam giác ABC P là một điểm Gọi Na, Nb, Nc lần lượt tâm đường tròn Euler tam giác

P BC, P CA, P AB Chứng minh cácđường thẳng quaNa, Nb, Nc lần lượt vng góc với

P A, P B, P C đồng quy.

Bài toán theođánh giá tơi tốn hay Tơiđã sửsụng toán trongđợt kiểm tra

đội tuyển VMO trường THPT chun KHTN thật đáng tiếc khơng có em giải toán

Lời giảiđầu tiên tơi cóđược cho tốn chưađẹp, sauđây tơi trình bày lời giải thứhai tham khảtưởng từhọc trịTạHà Ngun, học sinh lớp 12A1 Tốn, khóa 48, trường THPT chuyên KHTN Trong suốt toán ta kýhiệu(XY Z)chỉ đường trịn ngoại tiếp tam giác

XY Z Cơng cụsửdụng phần gócđịnh hướng haiđường thẳng

A

B C

P

S X

F

Y Z

D E

Lời giải. GọiD, E, F, X, Y, Zlần lượt trungđiểm củaBC, CA, AB, P A, P B, P C,

đường tròn(DY Z),(EZX),(F XY),(DEF)lần lượt cácđường tròn Euler tam giác

P BC, P CA, P AB, ABC Gọiđường tròn(DY Z)vàđường tròn(DEF)cắt tạiS khác D Ta có biếnđổi góc có hướng nhưsau

(SF, SY) = (SF, SD) + (SD, SY)

= (EF, ED) + (ZD, ZY)(DoS thuộc cácđường tròn(DY Z),(DEF)) = (BD, BF) + (BY, BD)(DoEF ∥BD, ED ∥AC, Y Z ∥BD, ZD∥BY)

(102)

= (XF, XY)(DoXY ∥BF, XF ∥BY)

DođóSthuộc(F XY) Tương tựS thuộc(EZX) Ta cóđiều phải chứng minh

Theo tốn cácđường thẳng quaNa, Nb, Nc song songP A, P B, P C đồng quy

tạiQ Ta chỉcần chứng minh rằngQnằm trênđường trịn(NaNbNc)thì hiển nhiênđường thẳng

qua Na, Nb, Nc vng góc với P A, P B, P C đồng quy điểmđối tâmQ Vậy ta

biếnđổi góc(QNb, QNc) = (P B, P C) = (DZ, DY) = (SZ, SY) = (NaNb, NaNc) Ta chúý

đẳng thức cuối có dođường trịn(Na)và (Nb)có dây cung chung làSZ nênNaNb ⊥ SZ,

tương tựNaNc ⊥SY Từ ta cóđiều phải chứng minh

Nhận xét.Bài toán choP trùng với số điểmđặc biệt sẽdẫn tới nhiều hệquảquan

trọng

Ngoài khai thác tốn xoay quanh tâmđường trịn Euler, nhậnđược nhiều tốn lạmắt Tơi xin giới thiệu tốn mà tơi tình cờtìm giải toán liên quan tới vấnđềvề điểm cố định tâmđường trịn Euler

Bài tốn 8. Cho lục giác ABCDEF nội tiếp đường tròn (O) Gọi K, L, N lần lượt tâm

đường tròn Euler tam giácDEC, BCA, F AE GọiX, Y, Z lần lượt hình chiếu của K, L, N theo thứtựlênAD, BE, CF Chứng minh trung trực củaAX, EY, CZđồng quy.

Trước hết ta có bổ đềsau

Bổ đề. Cho tam giácABC nội tiếpđường tròn(O).P là mộtđiểm trên(O). K là tâmđường

tròn Euler tam giácP BC.

a) Chứng minh rằngđường thẳng quaK vng góc vớiP Alnđi quađiểm cố định khiP di

chuyển.

b) GọiHlà hình chiếu củaK lênP A Chứng minh trung trực củaAH luônđi quađiểm

cố định khiP di chuyển.

A

B C

O

P

K N

L

I

(103)

Chứng minh. a) Gọi N tâm đường tròn Euler tam giác ABC Gọi L trung điểm

của ON Gọi I đối xứng A qua L, gọi M trung điểm P A Ta biết kết quen thuộc

−−→

KN = 12−→P A = M A−−→ suy ra−−→AN = −−→M K Do I đối xứngAquaL nênAN−−→ = −→OI Vậy từ

−−→

OM =−→KI suy raKI ∥OM ⊥P A Vậyđường thẳng quaK vng gócP Ađi quaI cố định

Ta cóđiều phải chứng minh

b) Dễthấy trung trực củaAH qua trungđiểmLcủaAI trungđiểmON cố định Ta

cóđiều phải chứng minh

O F

A

B

C D

E

K

L N

Z

Y

X

T

Lời giải toán. Bài toán hệquảcủa bổ đềtrên ta dễthấy trung trực cácđoạnAX, EY, CZ

đồng quy trungđiểmOT vớiT tâmđường tròn Euler tam giácAEC Ta cóđiều phải

chứng minh

Nhận xét.Bổ đềlà toánđi quađiểm cố định quan trọng Chúng ta hồn tồn

dựa vào toánđi quađiểm cố địnhđể đềxuất thành toán chứng minhđồng quy làm toán Bài toán từngđược tác giả đềnghịtrong kỳthi chọnđội tuyển thi học sinh giỏi quốc gia trường THPT chuyên KHTN năm 2013 xem [4] Sauđó tốn xuất [5] không ghi nguồn gốc, viết này, xin giới thiệu lại rõ nguồn gốc với bạnđọc nhưtrên

Sauđây tốn tác giảcũng tình cờtìm Bài tốn nàyđược tác giảtạo khiđang tìm hiểu vềcác hệthức hàngđiểmđiều hịa, bắt nguồn từ giảng cho lớp học toán online EgoGreen Bài toán cũngđã xuất [6] không ghi nguồn gốc, xin giới thiệu lại rõ nguồn gốc với bạnđọc nhưtrên

Bài toán 9. Cho tam giácABC đường caoBE, CF vàđường tròn Euler là(O9),D, Gthuộc

BC sao cho (BC, DG) = −1. ED, F D lần lượt cắt (O9) lần lượt tại M, N khác E, F.

(104)

G qua trung điểm P Q Gọi T đối xứng của D qua trung điểm M N Chứng minh rằng R, S, T thẳng hàng.

A

B C

O9

D G

F

E P

Q

M N

T

S

R Y

X

I

0 Lời giải. Gọi I trungđiểm BC theo hệ thức Newton ta dễ cóIE2 = IF2 = IB2 =

IC2 =ID.IG.

Từ dễ có ∠IF D = ∠IGF,∠IED = ∠IGE GọiP S cắt (O9)tại X Suy ∠EP X =

∠EGF = ∠IGE −∠IGF = ∠IED−∠IF D = ∠IED−∠IEN = ∠M EN Từ dễ

suy raEM ∥N X suy raN T quaX Tương tựgọiSQcắt(O9)tạiY thìM T quaY Áp dụngđịnh lýPascal cho

!

P N Y Q M X

"

ta thuđượcR, S, T thẳng hàng Ta cóđiều phải chứng

minh

Sauđây ba toán hay liên quan tới tâmđường tròn Euler tâmđường tròn nội tiếp tơiđềxuất Các tốn nàyđãđược hồn thiện lời giải học trị xuất sắc tơi Nguyễn Ngọc Chi Lanhọc sinh lớp 12A1 Tốn, khóa 48, trường THPT chuyên KHTN Bài toán 10. Cho tam giác ABC có tâmđường trịn nội tiếp làI.D, E, F là hình chiếu của AlênBC, IC, IB GọiK là tâm ngoại tiếp tam giácDEF Chứng minh rằngKI đi qua tâm

đường tròn Euler tam giácIBC.

B A

C I

D

E F

N M

Z

Y H

X

K J

(105)

Lời giải. GọiM, N giaođiểmAE, AF vớiBC Dễdàng chứng minhE, F

là trung điểm AM, AN Suy K tâm ngoại tiếp tam giác DEF tâm đường tròn

Euler tam giác AM N Cũng chứng minhđược I tâm ngoại tiếp tam giácAM N Gọi IX, BY, CZ đường cao tam giácIBC vàH trực tâm tam giác Dễthấy AM ∥ HB, AN ∥ HC GọiP giaođiểmAH, BC Phép vịtựtâmP biến tam giácAM N

thành tam giácHBC Lại cóIKlàđường thằng Euler tam giácAM N GọiJlà tâmđường

trịn Euler tam giácHBC, khiđóIJ làđường thẳng Euler tam giácHBC VậyI, J, K

thẳng hàng.J tâm ngoại tiếp(XY Z), suy raJ tâm Euler tam giácIBC Suy

điều phải chứng minh

Bài tốn 11. Cho tam giác ABC có tâmđường trịn nội tiếp làI.D, E, F là hình chiếu của A lên BC, IC, IB Gọi AI cắtđường tròn ngoại tiếp(O) của tam giácABC tại Gkhác A.

Chứng minh rằngđường thẳng Euler tam giácDEF đi qua trungđiểmAG.

J A

B C

I O

G E F

N M

D K H

L P

Q

Lời giải. GọiM, N giaođiểmAE, AF với BC H, K trực tâm tâm

ngoại tiếp tam giácDEF.AHcắt(AM N)tạiL.Jlàđối xứng củaAquaKvàP hình chiếu

của I lênBC Ta có kết quen thuộc: Llàđối xứng củaA quaH, I tâm ngoại tiếp

tam giácAM N vàJlàđối xứng củaIquaBC Nhưvậy, dễthấyHK ∥JLhayJLsong song

vớiđường thẳng Euler tam giácDEF Do∠F AH = ∠F CD =

2∠ABC = ∠ACF =

∠ADF =∠F EH suy raA, E, F, H thuộc mộtđường trịn Lại cóAE ⊥IE, AF ⊥IF

suy raA, E, F, I thuộc mộtđường trịn, suy raHthuộcđường trịnđường kínhAI, suy IH ⊥ ADsuy raIHDP hình chữnhật Ta chứng minhH, P, Gthẳng hàng GọiQlà giao

điểmAG, BC Lại cóGlà tâm ngoại tiếp tam giácIBC nên dễcóQG.QA=QI(QG+GI)

suy raQG.IA =QI.GI suy QGIG = IQIA suy P DP Q = P QHI = QGIG, suy raH, P, G thẳng hàng

Suy raL, J, Gthẳng hàng Suy raHK ∥LGsuy raHKđi qua trungđiểmAG

Bài toán 12. Cho tam giácABC nội tiếpđường tròn(O). P di chuyển trên(O).Đường thẳng

quaP song songBC cắtCAtạiE GọiK là tâmđường tròn ngoại tiếp tam giácP CE L

(106)

A

B C

O

P E

M K

G

L N

J Q H

Lời giải. GọiH, N trực tâm tâmđường tròn Euler tam giácABC GọiM

trung điểmBC, G làđối xứng củaP quaL Kết quảquen thuộc, ta có Glà đối xứng O quaBC GọiQlà giaođiểmAH với(O) DoP E ∥ BC suy ra∠CEP = ∠ACB, suy

∠CP K = 90◦ −∠CEP = 90◦ −∠ACB = ∠CAQ = ∠CP Qsuy P, K, Qthẳng hàng

Đường thẳng quaLsong songP K cắtGQtạiJ Suy raJ trungđiểmLQ TứgiácOHQG

là hình thang cân,N J làđường trung bình hình thang Suy raJ đối xứngN quaBC Suy

rađường thẳng quaLsong songP K luônđi quađiểm cố định làđiểmđối xứng với tâm Euler

(107)

2 M

t s

bài toán luy

n t

p

Các toán (trừbài toán cuối cùng)đềuđược tác giảsáng tác vàđã gửi lên diễnđàn AoPS sốnăm gầnđây

Bài toán 13. Cho tam giácABCnội tiếp(O)và cóđiểm LemoineL.LA,LB,LCcắt lại(O)

tạiA′,B′,C′.

1) Chứng minh rằngABC A′B′C′ có chung haiđiểm BrocardΩ1Ω2.

2) GọiP là trungđiểm củađoạn thẳngΩ1Ω2.N N′ là tâmđường tròn Euler của tam

giácABCA′B′C′ Chứng minh rằngP N =P N′.

Bài tốn 14. Cho tam giácABC có trực tâmH,điểm LemoineLvà tâmđường tròn EulerN.

Gọi A′B′C′ là tam giác anti-Ceva củaN Chứng minh rằng tâm ngoại tiếp tam giácA′B′C′

nằm trênHL.

A

B C

N

A'

B'

C'

H

O' L

Bài tốn 15. Cho tam giácABCcó tâm ngoại tiếpO tâmđường tròn Euler làN Tam giác

Napoleon làDEF.

(108)

2) Chứng minh rằngAD, BE, CF đồng quy tạiQ.

3) Chứng minh rằngđường thẳngP Qchiađơiđoạn thẳngON.

Bài tốn 16(Tham khảo [9]). Cho tam giácABC N,Na,Nb, vàNc lần lượt tâmđường

tròn Euler tam giác ABC, N BC, N CA, vàN AB Chứng minh rằngANa, BNb, và

CNc đồng quy.

Tài li

u

[1] T Q Hùng, Mởrộng tốn hình học kỳthi học sinh giỏi quốc gia năm 2013, tạp chí tốn học tuổi trẻsố429 tháng năm 2013

[2] T Q Hùng, P V Thuận, Cuộc thi giải toán mathley,

http://www.hexagon.edu.vn/mathley.html

[3] T Q Hùng, Tuyển tập toán hình học chọnđội tuyển KHTN, năm 2013 [4] T Q Hùng, Tỷsốkép, phép chiếu xuyên tâm, hàngđiểmđiều hòa, chùmđiều hòa,

https://analgeomatica.blogspot.com/2014/01/bai-giang-ty-so-kep-phep-chieu-xuyen.html

[5] DonaldLove (Nick name người Việt Nam), nice concurrent,

https://artofproblemsolving.com/community/c6h577336

[6] lambosama (Nick name người Việt Nam), Very hard concurrent problem,

https://artofproblemsolving.com/community/c6h560895

[7] E W Weisstein, Nine-Point Circle, from MathWorld–A Wolfram Web Resource,

http://mathworld.wolfram.com/Nine-PointCircle.html

[8] T Q Hùng, Diễnđàn tốn hình học nằm AoPS,

https://artofproblemsolving.com/community/c374081

[9] C Kimberling, Encyclopedia of Triangle Centers,

(109)

ĐIỂM HUMPTY - DUMPTY TRONG TAM GIÁC

ỨNG DỤNG

Nguy

n Tr

ườ

ng S

ơ

n

(GV THPT Chuyên L

ươ

ng V

ă

n T

y, Ninh Bình)

LỜ

I BAN BIÊN TẬP

Gầnđây, nhiều mơ hìnhđược khai thácđể"tăng tốc" việc tiếp cận tốn hình học khó Ngồi mơ hình kinhđiển nhưtứgiác tồn phần, tứ giácđiều hịa, ta cịn có cácđiểmđặc biệt với nhiều tính chất tam giác, tứgiác Trong viết này, tác giảsẽ giới thiệu vềhaiđiểm Humpty, Dumpty tam giác với cácứng dụng chúng đểgiải toán

1.

Đ

i

m Humpty - Dumpty m

t s

tính ch

t liên quan.

1.1.

Đị

nh ngh

ĩ

a.

1 Điểm Humpty. Cho tam giác ABC Điểm PA nằm tam giác ABC thỏa mãn

!

PACB =P!AAC,P!ABC =P!AAB gọi làđiểmA−Humpty Định nghĩa tương tựvới cácđiểmB−Humpty vàC−Humpty

C A

B

(110)

2 Điểm Dumpty. Cho tam giác ABC Điểm QA nằm tam giác ABC thỏa mãn

!

QACA=Q!AAB,Q!ABA=Q!AACđược gọi làđiểmA−Dumpty Định nghĩa tương tựvới cácđiểmB−Dumpty vàC−Dumpty

C A

B

I J

O QA

1.2 Tính ch

t.

Tính chất 1.Cho tam giácABCcó trực tâmH Giaođiểm thứhai củađường trịnđường kính AH đường tròn ngoại tiếp tam giácBHC nằm trênđường trung tuyếnAM Giao điểmđó

chính làđiểmA−Humpty

Chứng minh. GọiA′làđiểm thỏa mãnABA′Clà hình bình hành.

VìBAC" = BA!′C = π−BHC.! Suy raA′ thuộcđường trịn(BHC).VìHBA!′ = HBC!+ !

A′BC = π

2 −ACB" +ACB" =

π

2,vì vậyHA′làđường kính của(BHC)

Khiđó:AP!AH =

π

2 =π−HP!AA′,suy raPAnằm trênđường trung tuyếnAM

A

B

O PA

D

E F

M H

A′

(111)

Nhận xét Đường trung tuyến AM cắt đường tròn (BHC) tại điểm A′ khác điểm P

A thì

ABA′Clà hình bình hành.

Tính chất 2.Gọi(I)làđường trịn quaA, Btiếp xúc vớiBC,(J)làđường trịn quaA, Cvà tiếp

xúc vớiBC Khiđó giaođiểm khácAcủa haiđường trịn(I),(J)chính làđiểmA−Humpty

Chứng minh. GọiP giaođiểm thứhai haiđường trịn(I),(J)

VìAP B" =π−CBA" vàAP C" =π−ACB" nên

!

BP C= 2π−AP B" −AP C" =ACB" +CBA" =π−BAC"

Suy raP nằm trênđường trịn(BHC) Vì theo tính chất1, P nằm trênđường trung tuyến

của tam giácABC Suy raP ≡PA

Tính chất 3. Cho tam giácABC nội tiếp(O).ADlà đường đối trung tam giácABC, D

thuộc(O).GọiP trungđiểmAD KhiđóP, PAliên hợpđẳng giác

Chứng minh. Do ADlà đườngđối trung tam giác ABC, D thuộc đường tròn (O)nên tứ

giácABDC tứgiácđiều hịa DođóBC làđườngđối trung tam giácBAD Mặt khác, BP làđường trung tuyến tam giácBDA.DođóBC, BP đẳng giác

Suy P BA" = CBD! = !DAC = P!AAB = P!ABC.Vậy BP, BPA hai đường đẳng giác

Suy raP, PAlà haiđiểmđẳng giác liên hợp

A

B

O PA

D K′ M H

(112)

Tính chất 4.Cho tam giácABC nội tiếpđường trịn(O).ADlàđườngđối trung tam giác ABC, Dthuộcđường trịn(O).KhiđóD, PAđối xứng với quaBC

Chứng minh. Bằng biếnđổi góc, suy hai tam giác BPAC vàBDC nên dễdàng

suy raD, PAđối xứng quaBC.Từ đây, ta suy kết quả:PAnằm trênđường trònA−

Apollonius tam giácABC

Tính chất 5. Gọi (I, R1) làđường tròn quaA, B tiếp xúc với BC,(J, R2) làđường tròn qua

A, C tiếp xúc với BC Gọi R, R3 bán kính đường trịn ngoại tiếp tam giác ∆ABC,∆PABC KhiđóR.R3 =R1.R2

Chứng minh. Theo tính chất 4, suy raR = R3.GọiK tâmđường trịn ngoại tiếp tam giác

BPAC Dễthấy,IK, JK tươngứng trung trực củaBPA, CPA,ta có

"

BJK =

2BJP!A=P!ABC =

1

2P!AKC =CKI"

Tương tự,BKJ" =CIK," dođó∆BJK ∼∆CKI.Suy JB

KC = KB

IC ⇔R

2

3 =R1 ·R2 ⇒R·R3 =R1·R2

A

B

PA

J

K

I

(113)

Nhận xét Từ chứng minh dẫn đến ∆PABC ∼ BJK ⇒

PAB

PAC

= R1

R3

Tương tự, AB

AC = R1

R3

Dođó phân giác hai gócBAC," BP!ACcắt mộtđiểm thuộc BC.

Tính chất 6.GiảsửADlàđườngđối trung tam giácABC, D∈BC.Đường thẳng quaD

và vuông góc vớiBC cắtđường trung tuyếnAM tạiS Đường thẳng quaS song song với BC cắtAB, AC theo thứtựtạiF, E KhiđóPA≡BE∩CF

Chứng minh. Ta sẽchứng minh toán sau: Cho tam giácABC, PAlàđiểmA−Humpty

M trungđiểm củađoạnBC GọiE =BPA∩AC, F =CPA∩AB, S =EF ∩AM GọiD

là hình chiếu củaSlênBC KhiđóADlàđườngđối trung tam giácABC

Thật vậy, ta có:

M B2 =M A·M PA=M C2 ⇒

M B M PA

= M A

M B, M C M PA

= M A

M C

Suy cặp tam giác(M BPA, M AB),(M CPA, M AC)đồng dạng

Dođó: BPA

AB = M B M A,

CPA

AC = M C M A nên

BPA

AB = M C

M A (1)

LấyđiểmLtrênADsao choDS ∥PAL Từ đó, chúýrằngDA, DPA, DS, DM chùmđiều

hòa, suy raDM qua trungđiểm củaLPA

A

B

O PA

D M

H

C S

F E

K

L

Từ đó, chúýrằngLPA⊥DM, suy raL, PAđối xứng quaBC (2)

Từ(1),(2)suy LB

AB = LC

CA Hơn nữa,BAC" +BLC" = BAC" +BP!AC =π Dođó, tứgiác ABLC nội tiếp, cũngđiều hịa DođóADlàđườngđối trung tam giácABC

Tính chất 7.Trực tâmKcủa tam giácDEF, PAvà tâmđường tròn ngoại tiếpOtam giácABC

(114)

Chứng minh. Từchứng minhởtính chất 6, ta có

!

DPAC =DLC" =!F BC,F AE" +F P!AE =π

Dođó tứgiácF PAEA, BF PADnội tiếp Vì F PAEA nội tiếp vàBC tiếp xúc với đường

tròn(APAB)nênP!AEF =P!AAF =P!ABC Suy raBC ∥EF

Vì tứgiácF PAEA, BF PADnội tiếp vàOlà tâmđường tròn ngoại tiếp tam giácABC nên

!

F DB =BP!AF =BAC" =

1

2!BOC =

π

2 −!OBC Suy raBO⊥F D

Từ đó, chúýrằng,EK⊥F D,suy raBO∥EK Tương tự,CO ∥F K

Từ đó, ta suy OK, BE, CF đồng quy PA Điều có nghĩa K, PA, O thẳng

hàng

Ở nhiều tính chất (từdễ đến khó) củađiểmHumpty Sauđây, chúng tơi sẽliệt kê sốtính chất củađiểmDumpty,bạnđọc thửtự đưa chứng minh, xem nhưbài tập

Tính chất 8.Cho tam giácABC cóQAlàđiểmA−Dumpty Khiđó:

1 QAnằm trênđườngA−đối trung tam giácABC

2 QAlà tâm vịtựquay biến∆AQACthành∆CQAB biếnAC thànhBA

3 BốnđiểmB, QA, O, Cđồng viên trongđóOlà tâmđường trịn ngoại tiếp tam giácABC

4 OQA⊥AQA

5 Giaođiểm củađường trònđi quaA, B tiếp xúc vớiACvàđường trònđi quaA, Ctiếp

xúc vớiAB làđiểmA−Dumpty

2 Bài t

p áp d

ng.

Bài toán (ELMO shortlist 2013) Cho tam giác ABC Gọi D điểm di động trênđường thẳngBC.Đường tròn ngoại tiếp tam giácABDcắtAC tạiđiểm thứhaiE,đường tròn ngoại tiếp tam giác ADC cắt AB tại điểm thứ haiF Khiđó đường trịn ngoại tiếp tam giác AEF lnđi qua mộtđiểm cố định khácA, khiDdiđộng.

Lời giải. Cách 1.GọiKlà giaođiểm củaBEvàCF Áp dụngđịnh lí Miquel cho tứgiác tồn

phầnAEKF BCta cóDthuộcđường trịn ngoại tiếp tam giácBF K

Do đó:(KF, KB) ≡ (DF, DB) ≡ (AF, AC) (mod π).Vậy tứ giácAEKF nội tiếp đường

tròn, suy

(115)

Vậy tứgiácBKPACnội tiếpđường tròn, suy

(KPA, KE)≡(CPA, CB)≡(APA, AC)≡(APA, AE) (mod π)

Vậy tứgiácAKPAE nội tiếpđường trịn, hayđường trịn ngoại tiếp tam giácAEF lnđi qua điểmA−Humpty

A

B

PA

F

D

E

C K

Cách 2.Ta phát biểu không chứng minh bổ đề sau:Cho tam giác ABC M là trungđiểm BC.CQ, BP là các đường cao tam giác ABC KhiđóM P, M Q đường thẳng quaA song song vớiBC là tiếp tuyến củađường trịn ngoại tiếp tam giácAP Q

Trởlại tốn, gọiHlà trực tâm tam giácABC.P, Qtheo thứtựlà hình chiếu củaH lên AC, AB

Phép nghịchđảoIAP.AC

A biếnđiểmB thànhQ, C thànhP,đường thẳngBC thànhđường tròn

ngoại tiếp tam giác AP Q, biến PA thành trung điểm M BC, biếnD thành D′ nằm đường tròn(AP Q).Đường tròn(ABD)biến thànhđường thẳngD′Q,biếnđường trịn(ACD)

thànhđường thẳngD′P Vì vậyE′ =QD′∩AC, F′ =P D′∩AB

Suy rađường tròn(AEF)biến thànhđường thẳngE′F′ Ta sẽchỉrađường thẳngE′F′ đi qua

trungđiểmM củaBC

DễthấyE′F′ làđườngđối cực củađiểmAD′ ∩P Q Áp dụng bổ đềtrên ta có,P Qlà đường

(116)

A

B

PA

F

D

E

C P

H Q

D′

E′

F′

M

Bài tốn (Tp chí tốn hc tui tr, s490, năm 2018).Cho tam giácABC cóđường trịn

(K) đi quaA, C tiếp xúc với AB;đường tròn (L) đi qua A, B tiếp xúc với AC.(K)cắt (L)

tạiDkhácA.AK, AL lần lượt cắtBD, DC tạiE, F.GọiM, N theo thứtự là trungđiểm của BE, CF.P điểmA− Humpty tam giácABC Chứng minh bốn điểmA, P, M, N thẳng hàng.

Lời giải. Cách 1.Từgiảthiết củađềbài ta thấyABtiếp xúc vớiđường trịn(K)tạiA, ACtiếp

xúc vớiđường trịn(L)tạiA.Trong tam giác vngF ACcó: N A=N C = CF

2 ⇒!N AC =N CA,! (1)

Mặt khác, doAB tiếp xúc vớiđường tròn(K)tạiA,nên

!

N CA=!DCA=!BAD(cùng chắn cung AD củađường tròn (K)) (2)

Từ(1) (2) suy ra:!BAD=!N AC,hayADvàAN haiđườngđẳng giácđối vớiBAC" (3)

Tương tự, ta chứng minhđượcBAM! =!CAD⇒BAD!=CAM! hayADvàAM hai

đườngđẳng giácđối với gócBAC."

(117)

C A

B

O P D

K L

E

F M

N

Do ∆ABD ∼ ∆CAD(g.g)suy d(D, AB)

d(D, AC) =

AB

AC nên suy raAD làđườngđối trung

tam giác ABC Do AM đường trung tuyến tam giác ABC Do A, P, M thẳng

hàng Suy rađiều phải chứng minh

Cách 2.

C A

B

O P D

K L

E

F M

N J

U

R

Q

Giảsử đường trịn(L)cắtBCtạiđiểm thứhaiU.U DcắtACtạiđiểmQthìQU C" =DAB!= !

(118)

GọiJlàđiểmđối xứng củaBquaAvàRlàđiểmđối xứng củaAquaU Do hai tam giácCU A

và CAB đồng dạng, nên hai tam giácCRAvà CJB đồng dạng Từ ýtam giác JBE

cân tạiE nên

"

BJC =ARC" =AU Q" =!ABD=BJE."

Suy raC, E, J thẳng hàng, màAlà trungđiểmBJ dođóAM ∥CE Tương tựAN ∥BF

Lại cóCF A" = 90◦ −ACF" = 900−!BAD = !DAE; tương tựthìAED! = !DAF Từ đó hai

tam giácAF DvàEADđồng dạng (g.g)

Kết hợp với tam giácBADvàACDđồng dạng (g.g) ta suy DF ·DE =DA2 =DB·DC

Từ ta thấy DB

DE = DF

DC nênBF ∥CE suy raA, M, N thẳng hàng

Do ∆ABD ∼ ∆CAD(g.g)suy d(D, AB)

d(D, AC) =

AB

AC nên suy raAD làđườngđối trung

tam giác ABC Do AM đường trung tuyến tam giác ABC Do A, P, M thẳng

hàng Suy rađiều phải chứng minh

Bài toán (IMO Shorlist 2008, G4).Cho tam giác nhọnABC BE, CF là haiđường cao của tam giác Haiđường tròn cùngđi qua haiđiểm A E và tiếp xúc với đường thẳng BC tạiP Qsao choB nằm giữaC Q Chứng minh haiđường thẳngP EQF sẽcắt nhau giaođiểm haiđường thẳng nằm trênđường tròn ngoại tiếp tam giácAEF Lời giải. Cách 1.

P C A

Q

F H

B D E S

Theo tính chất củađiểm Humpty ta cóB trungđiểm củaP Q

Vẽ đường caoADvà gọiH trực tâm tam giácABC

(119)

Từ suy raBP2 =BE·BH ⇒ BP

BH = BE

BP hay tam giácBP HvàBEP đồng dạng Suy

ra!BP E =BHP! (1)

ĐiểmP nằm giữaDvàC, lại cóBP2 =BC·BDnên

DP ·DQ= (BP −BD)·(BD+BQ) = BP2−BD2

=BC ·BD−BD2 =BD·(BC−BD) = BD·DC

Vì hai tam giác vuôngBDH vàADC đồng dạng nênBD·DC =AD·DH.Kết hợp vớiđẳng

thức tađượcDP ·DQ=AD·DH ⇒ DA DQ =

DP DH

Từ hai tam giác vngDAQvàDP H đồng dạng nênHP D! =!QAD.Có thểviết lạiđẳng

thức làBP H! =BAD!+BAQ."

VìBQlà tiếp tuyến của(AF Q)nên!BQF =BAQ" =BP H!−!BAD (2)

Từ(1)và(2)suy !

BP E+!BQF =BHP!+BP H!−BAD!= (1800 −P BH!)−!BAD

= (900+BCA")−(900−ABC") =BCA" +ABC" = 1800−CAB"

Dođó!BP E+!BQF <1800,vì vậy hai tiaP EvàQF phải cắt nhau.

GọiSlà giaođiểm củaP EvàQF.Khiđó:

"

P SQ = 1800−(!BP E+!BQF) = CAB" =EAF "

NếuS nằm giữaP, EthìP SQ" = 1800−ESF";cịn nếuEnằm giữaP, S thìwidehatP SQ= "

ESF

Từ ta nhậnđượcSnằm trênđường tròn ngoại tiếp tam giácAEF

Cách 2.

P C A

Q

F H

B D

E S

U V

(120)

Gọi U, V, S giaođiểm thứ hai P A, P H, P E vớiđường tròn đường kínhAH Tam giác AHP có cácđường caoAV, BC, HU chúngđồng quy tạiQ′

Áp dụngđịnh lí Pascal cho sáuđiểmAESF HV ta có bađiểm C =AE∩F H, P =ES∩HV, SF ∩V A

thẳng hàng, suy Q′ ∈ F S. Giả sử đoạn thẳng AD theo thứ tự cắt đường trịn đường kính

(BC),(P Q′)tạiK, L.VìHlà trực tâm tam giácABC,và∆BKC vuông nên ∆ABD ∼∆ADC ⇒DA·DH =DB·DC =DK2

VìH trực tâm tam giácAQ′P ta cóDL2 =DA·DH Vì vậyDK = DL ⇒ K ≡ L Kết

hợp∆BKC vuông và∆ABD∼∆CBF, suy

BD·BC =BA·BF =BK2

Mặt khác,BA·BF =BP2 =BQ2, suy raBK =BP =BQ.Điều suy raBlà tâmđường

trịn đường kínhP Q′ Dođó Q ≡ Q′.Dođó giao điểm củaP E vàQF làS nằm đường

trịn(AEF)

Nhận xét 1 Bài tốn ta nhận thấy, F chính làđiểm A-Humpty tam giácAP Q

Đồng thời,H vừa trực tâm tam giácABC vừa trực tâm tam giácAP Q

2 Nếu ta áp dụngđịnh lí Pascal với sáuđiểmAF T EHV ta thấyET đi quaQ′ Như

vậy:P F QE cũng cắt mộtđiểm trênđường tròn(AEF) 3 Bài tập tương tựnhưsau:

Cho tam giácABC nhọn, có cácđường caoBE, CF.GọiP trên cạnhBC, Qthuộc tia

đối tiaBC sao choBQ2 =BP2 =BF ·BA GọiS =QF ∩P E Chứng minh bốn

điểmE, S, A, F đồng viên.

Một tập có mơ hình tương tự, xinđược chia sẻtới bạnđọc

Bài tốn Cho tam giác ABC khơng cân khơng vng tại A Cácđường caoBE, CF Cácđường trịn(O1),(O2)cùngđi quaAvà theo thứ tựtiếp xúc vớiBC Dlà giaođiểm thứ

hai của (O1) (O2) M, N theo thứ tự là giao điểm thứ hai củaBA, BD (O2).P, Q theo thứ tựlà giaođiểm thứhai củaCA, CD với(O1) R là giaođiểm củaADvớiEF S là giao

điểm củaM P N Q Chứng minh rằngRS⊥BC

Lời giải. Ta cần có bổ đề: Cho tam giácABC Cácđường caoBE, CF.Cácđường tròn

(O1),(O2)cùngđi quaAvà theo thứtự tiếp xúc vớiBC D giaođiểm thứ hai của(O1)và (O2) Rlà giaođiểm củaADvớiEF X giaođiểm củaBDvàAC Y giaođiểm củaCD

vàAB Khiđó:

a RthuộcXY

(121)

Chú ý D điểm A− Humpty tam giác ABC nên bổ đề khơng khó, xin dành

chứng minh cho bạnđọc

A

B

D E F

I H

R

W

C O1

O2

X Y

Trởlại toán,

A

B

D E F

R

C O1

O2

X Y

M

N

P Q

S

(122)

Dễthấy:

(P B, P Q)≡(DB, DC)≡(DB, BC) + (BC, DC)≡(AD, AB) + (AC, AD)

≡(AC, AB)≡(AP, AB)≡(QP, QB)≡ −(QB, QP) (mod π)

Dođó, tam giácBP Qcân tạiB.Kết hợp vớiBC tiếp xúc với(O1),suy raP Q∥BC

Tương tự, tam giácCN M cân tạiCvàN M ∥BC

Dễthấy

(P B, CN)≡(P B, AP) + (AC, N C)≡(BD, AD) + (AD, N D)

≡(BD, N D)≡0 (mod π)

Dođó:BP ∥CN

Tương tựBQ ∥CM

VậyP Q∥BC ∥N M, BP ∥CN, BQ∥CM

Dođó tam giácCM N, BP Qhoặc làảnh qua phép vịtựhoặc làảnh

qua phép tịnh tiến Vậy nên

XP XC =

BP N C =−

P B N C =−

P Q N M =

QP N M =

SP SM

DođóSX ∥ CM.Tương tự, chúý rằngBP ∥ CN,ta cóSY ∥ BP ∥ CN.Theo bổ đềtrên,

chúýrằngM N ∥BC,ta cóXY ∥BC ∥M N

Vậy tam giácSXY, CM Nđồng dạng hướng Từ đó, chúýrằng tam giácCM Ncân C,suy tam giácSXY cân tạiS Lại theo bổ đềtrên thìRX =RY nên suy raRS⊥BC

Bài toán (ELMO 2014.)Cho tam giácABC vớiO, H lần lượt tâm ngoại tiếp tam giác, trực tâm tam giácABC Gọi ω1,ω2 đường tròn ngoại tiếp tam giác BOC, BHC.Giả sử

đường trònđường kínhAOcắtω1 tạiđiểm thứhaiM,đường thẳngAM cắt lạiđường trịnω1

tạiđiểmN.Tương tự, giảsử đường trịnđường kínhAHcắtđường trịnω2tạiđiểm thứhaiN

vàđường thẳngAN cắt lạiđường trònω2 tạiY Chứng minh rằngM N ∥XY

Lời giải. DoOM⊥AXnênOX làđường kính củađường trịnω1 Suy raXC⊥OC, XB⊥OB

hayXC, XBlà tiếp tuyến củađường tròn(O).Suy raAX làđườngđối trung tam giác ABC Ta có

!

BM X =BCX! =BAC" ⇒M BA! =M AC.!

Tương tự ta suy M CA! = M AB.! Ta cóN giao điểm thứ hai hai đường trịn

(AH),(BHC), nên theo tính chất N điểm A− Humpty Suy ra,N nằm đường

trung tuyến dựng từAcủa tam giácABC DoABY C hình bình hành vàAX, AY đẳng giác

nên

!

M AC =N AB! =AY C" ⇒M BA! =AY C."

Suy ra∆AM B ∼∆ACY ⇒ AM AC =

AB

AY ⇒AM ·AY =AC·AB (1)

Lại có:M BA! =M AC! =N AB! =N BC.!

Suy ra∆AN B ∼∆ACX ⇒ AN AC =

AB

AX ⇒AN ·AX =AC·AB (2)

Từ(1),(2)suy raAM ·AY =AN ·AX ⇒ AM AX =

AN

(123)

A

B

O H

C

X

Y M

N

Bài toán (USA TST 2005.)Cho tam giácABC nội tiếpđường tròn(O).ĐiểmP nằm trong tam giácABC thỏa mãn!P CB =P AC," !P BC =P AB Trung tr" ực củađoạnAP cắtBC tại Q Chứng minh rằngAQP" = 2!OQB

Lời giải. Khơng tính tổng qt, giảsửAB < AC GọiMlà trungđiểm củađoạnBC.D, K

lần lượt giaođiểm củađường phân giác trong, phân giác ứng với gócA tam giác ABC.X giaođiểm hai tiếp tuyến tạiB, Ccủađường tròn(O).Y giaođiểm thứhai AK với(O).Suy raY, O, M, X thẳng hàng

Theo giảthiết, suy raP mộtđiểmA−Humpty tam giácABC.Theo tính chất

điểmA−Humpty thìP nằm trênđường trịnA−Apollonius,đường trịnđường kínhKD.Suy

raQchính trungđiểm củaKD Khiđó,AQP" = 2!AKP

GọiF giaođiểm củaOQvớiAX, T giaođiểm thứhai củaAX vớiđường tròn(O) Theo

tính chất 4, ta suy raT thuộcđường trònA−Apollonius VậyOQ⊥AX.Suy raQF M X

là tứgiác nội tiếp Suy raF QM! =F XM ! DoBX tiếp tuyến củađường tròn(O)nên "

(124)

C A

B D

O P

M

X Y

K

Q

F

T

Xét hai tam giác:∆AP C,∆ABX cóP AC" =BAX,! ACP" =!AXB Suy

∆AP C ∼∆ABX ⇒AP ·AX =AB·AC

Xét hai tam giác:∆ABK,∆AY C cóY AC" =BAK,! ABK! =AY C" Suy

∆ABK ∼∆AY C ⇒AK·AY =AB ·AC

Từ đó, suy raAP ·AX =AK·AY.Kết hợp vớiđiều kiện!P AK =Y AX,! ta suy

∆AP K ∼∆AY X ⇒!AKP =!AXY

Mặt khác,!OQB = OQM! = F QM! = F XM! = !AKP, kết hợp vớiAQP" = 2AKP ,! suy

điều phải chứng minh

Bài tốn (Brazil National Olympiad 2015.)Cho tam giácABC khơngđều cóH, Glần lượt là trực tâm, trọng tâm tam giác GọiX, Y, Z là cácđiểm theo thứtự nằm cácđường thẳngBC, CA, AB thỏa mãn !AXB = !BY C = CZA Hai" đường tròn (BXZ),(CXY) cắt

nhau tạiđiểm thứhaiP.Chứng minh rằngP nằm trênđường trịnđường kínhHG

Lời giải. GọiM làđiểmB−Humpty,N làđiểmC−Humpty Dễthấy:M ∈BG, N ∈ CG

vàM, N thuộcđường trịnđường kínhHG

Do BXA! +BZC" = π nên giao điểm T củađường thẳng AX CZ nằm đường trịn

(BXZ).Lại có

(125)

nên bốnđiểmA, T, M, C đồng viên Suy ra:

(T X, T M)≡(CA, CM)≡(BC, BM)≡(BX, BM) (mod π)

Dođó:B, M, T, Z đồng viên Vậy nênM thuộc vàođường tròn(BXZ).Chứng minh tương tự N thuộc vàođường tròn(CXY)

Áp dụngđịnh lí Miquel vào tam giácBGCta có:P thuộc vàođường trịn(GM N).VậyP nằm

trênđường trịnđường kínhHG.Điều phải chứng minh

C A

B

N

O P

M X

Y Z

H G T

Bài toán (Sharygin Geometry Olympiad 2015.)Cho tam giácABC nhọn, không cân Gọi A1, B1, C1 lần lượt trung điểm của BC, CA, AB Gọi B2, C2 lần lượt trung điểm của

BA1, CA1.GọiB3, C3lần lượt làđiểmđối xứng củaC1quaBvà củaB1 quaC.Chứng minh

rằng giaođiểm các đường tròn ngoại tiếp tam giácBB2B3, CC2C3 nằm trênđường tròn

ngoại tiếp tam giácABC

Lời giải. GọiP làđiểm nằm đường tròn ngoại tiếp tam giác ABC choAP đường

đối trung tam giác GọiP′ làđiểmđối xứng củaP quaBC.Theo tính chất ta suy raP′

làđiểmA−Humpty tam giácABC vàP′nằm trênđoạnAA

1.Ta có !

P A1B =AA!1B =ACP" =P BB!3

Suy AB tiếp tuyến đường tròn ngoại tiếp tam giác BA1P Từ đó, ta có P BA" = !

CA1P = CA!1P′ Lại có P!′CA1 = B!1AP nên ∆P BA ∼ ∆P′A1C Suy ∆P BA ∼ ∆P A1C

Từ đó, dễ dàng suy raP tâm vịtự quay biến CA1 thànhAB Do

B2A1

B2C

= B3B

B3A

=

2 nên

qua phép vịtựquay tâmP sẽbiếnB2 thànhB3 Suy ra:B!2P B3 =A!1P B =ABB!2hayP

thuộcđường tròn(BB2B3)

(126)

C A

B

A1

O

P

B1

C1

B2 C2

B3 C3

P′

Bài toán (G6, IMO Shortlist 2014) Cho tam giác nhọnABC Xét cácđiểm E, F lần lượt nằm cạnhCA, AB GọiM là trungđiểm củaEF.Trung trực củaEF cắtBC tạiK Trung trực củaM K cắtAC, AB theo thứtựtạiS, T.GiảsửcặpđiểmE, F được gọi thú vị nếu tứ giácKSAT nội tiếp Giảsửcác cặp E1, F1 E2, F2 là cặpđiểm thú vị Chứng minh

rằng E1E2

AB = F1F2

AC .

Lời giải. Với cặpđiểm thú vị(E, F), ta sẽnói tam giácKEF tam giác thú vị A

E

F M

K

C

B

S

T

N G

L Y I X

(ω1)

(ω)

Tam giácEF Klà tam giác thú vị Trước hết ta chứng minh rằngKEF! =KF E! =A!,điềuđó

(127)

Kí hiệu(ω) làđường tròn qua điểm A, K, S T.Đường thẳngAM cắt ST đường

trịn(ω)tạiN vàđiểmL

Vì EF ∥ T S M trungđiểm EF, N trung điểm ST Do K, M đối xứng với

nhau quaST nênKN S! = M N S! =LN T " Vì vậyK, Lđối xứng với qua trung trực ST Vì vậyLK ∥ ST.Gọi G làđiểmđối xứng vớiK quaN.KhiđóGnằm trênđường thẳng EF ta có thểgiảsửGnằm tiaM F.Ta có:

!

KGE =KN S! =SN M! =KLA" = 1800−KSA"

(nếuK ≡ L gócKLA" hiểu góc giữaALvà tiếp tuyến củađường trịn (ω)tạiL)

Điều có nghĩa làK, G, E, Sđồng viên VìKSGT hình bình hành nên

!

KEF =!KSG= 1800−T KS" =A!

VìKE =KF nên ta có:KEF! =KF E! =A!Suy raAK làđối trung tam giácAST A

Q

P

C B

Z

R F1

E1

K1

F2

E2

K2

Gọi Y giao điểm BE CF.Xét bộsáu điểm(EEY F F A)có K = EE ∩F F, B =

EY ∩F A, C =F Y ∩EAthẳng hàng, nên theođịnh lí Pascalđảo ta có:Y thuộcđường trịn

(AEF).Đường caoBP, CQcủa tam giácABC cắt tạiZ nên

!

BY C =!EY F = 1800−BAC" =BZC" ⇒Y ∈(BZC)

GọiRlà trungđiểm củaBC, ARgiao vớiđường trịn(BZC)tạiđiểm thứhaiH KhiđóH

điểmA−Humpty tam giácABC.Ta có:

!

F Y H =HBC! =HAF!⇒H ∈(AEF)

MàH ∈ (AP Q)nên theo phép vịtựquay tâmH, tỉsố HQ

HP, góc quayQHP! biếnQthànhL,

biếnF thànhE biếnF1thànhE1, biếnF2 thànhE2 Dođó

E1E2

F1F2

= HP

HQ = AP AQ =

(128)

Bài toán 10 (Đềxut 30/4/2016, THPT Chuyên Lê Hng Phong, TPHCM.) Cho tam giác

nhọnABCvàđiểmDdiđộng tiađối tiaCB.Đường tròn(I)nội tiếp tam giácABD đường tròn (J) nội tiếp tam giác ACD Giả sử H, K là giao điểm hai đường trịn

(I),(J).Chứng minhHK lnđi qua mộtđiểm cố định khiDdiđộng.

Lời giải. Gọi N, M điểm tiếp xúc đường tròn nội tiếp (I) với cạnh

AD, BD.Q, P cácđiểm tiếp xúc củađường tròn nội tiếp(J)với cạnhAD, BD.S

là giaođiểm củaHKvớiAD

Theo định lí điểm Humpty ta suy raE, F trung điểm M P N Q Do đó: EM =

EP =F N =F Q

GọiT hình chiếu tâm bàng tiếpứng vớiđỉnhB tam giácABC Suy raT cố định

GọiF giaođiểm củaAT vớiHK.Ta có: CP = CA+CD−AD

2 , CT =

CA+AB −BC

2

⇒P T =CT −CP = AB+AD−BD

2 =AN

Từ suy raET =AS, ED=SD.Áp dụngđịnh lí Menelaus vào tam giácAT Dvới bađiểm S, F, E thẳng hàng ta có

F A F T ·

ET ED ·

SD

SA = 1⇒F A=F T

Suy raF trungđiểm củađoạnAT,làđiểm cố định

C A

B D

I

M

J

P H

K N L

E

F V Q

T S

U

Bài toán 11 Cho tam giác ABC cân tại A M điểm trong tam giác ABC sao cho

!

ABM =BCM ! GọiH, I, K lần lượt hình chiếu vng góc củađiểmM lênAB, BC, CA Gọi E là giao điểm củaM B IH, F là giao điểm của M C IK Giảsử haiđường tròn ngoại tiếp tam giácM EH M F Kcắt tạiđiểm thứhaiN Chứng minh rằngđường thẳngM N đi qua trungđiểm củaBC

(129)

Lời giải. Dễthấy tứgiácHBIM, KCIM nội tiếp, suy

!

HIM =ABM! =BCM!;KIM! =ACM! =CBM !

Do đóEM F! = EIF" = EM F! +HIM! +KIM! = BM C! +BCM! +CBM! = 180◦ nên

EM F I nội tiếp, ta có:M F E! = !M IE = M BH! = BCM! Suy EF ∥ BC Gọi P, Qlần

lượt giaođiểm củaM N vớiEF vàBC.Ta có:P N E!=M HE! =M BC! =M EP !

Tương tự ta chứng minh đượcP N F! = M F P ! Suy M điểmN− Humpty tam giác N EF.DođóP trungđiểmEF, suy raQlà trungđiểmBC

A

Q P

C B

M H

K

I E F

N

Bài toán 12 Cho tam giác nhọnABC (AB < AC) với trung tuyếnAM. Đường thẳng AM cắtđường tròn ngoại tiếp tam giácABCtạiđiểm thứhaiD.Đường thẳngAB cắtđường thẳng CD tạiE,đường thẳngAC cắtđường thẳngBDtạiF.Đường tròn ngoại tiếp tam giácABF cắtđường tròn ngoại tiếp tam giácACE tại hai điểmA P Gọi (S1) đường trònđi qua

C và tiếp xúc vớiAB tạiA, (S2)làđường tròn đi quaB tiếp xúc vớiAC tạiA GọiO là tâm

đường tròn ngoại tiếp tam giácABC Qlà giaođiểm thứhai của(S1)(S2) Chứng minh

tam giácOP Qlà tam giác vuông.

(Đềthi chọnđội tuyển HSG quốc gia TP Hà Nội, 2016 - 2017) Lời giải. Nhận thấyđiểmQchính làđiểmA−Dumpty tam giácABC.Ta có

(P E, P F) = (P E, P A) + (P A, P F) = (CE, CA) + (BA, BF)

= (CD, CA) + (BA, BD) = (CD, CA) + (CA, CD) = (mod π)

DođóE, P, F thẳng hàng Áp dụngđịnh lýMenelaus cho tam giácABM với cát tuyếnEDC

và tam giácACM với cát tuyếnF BDta có EA

EB · CB CM ·

DM

DA = F A F C ·

BC BM ·

(130)

Suy EA

EB = F A

F C hayBC ∥EF Khiđó

!

P BD=!DEP =BCD,!

dẫnđếnP Blà tiếp tuyến củađường tròn(O) Chứng minh tương tựta cóP C tiếp tuyến

của đường trịn (O) GọiQ′ là hình chiếu của O trên AP thì O, Q′, B, C, P cùng nằm trên

đường trịnđường kínhOP

A

O

B

E

C M

F Q

P D

Suy

!

ABQ′ =ABC" −Q!′BC =ABC" −Q!′P C =ABC" −AP C"

=ABC" −AEC" =BCD!=!BAD

Mặt khác, BC ∥ EF nên !BAD = BCD! = CEP" = CAP" = CAQ!′. Do đó ABQ!′ = !

CAQ′.Suy raAC là tiếp tuyến của(ABQ′)nên(ABQ′) ≡ (S2) Chứng minh tương tự ta có

(ACQ′)≡(S1), suy raQ≡Q′ Vậy tam giácOP Qvng tạiQ

Bài tốn 13 (Iranian Mathematical Olympiad.)GiảsửrằngP là mộtđiểm nằm tứgiác ABCDsao cho

!

BP C = 2BAC," P CA" =!P AD,!P DA=P AC" Chứng minh rằngP BD!=|P CA" −BCA"|

Lời giải. Ta dễdàng nhận thấyP làđiểmA−Dumpty tam giácADC.Kí hiệu(ω1),(ω2)

theo thứtựlàđường tròn ngoại tiếp tam giácAP C, AP D Qlà giaođiểm thứhai củaP B

vớiđường tròn(ω1), DBcắt(ω2), AQtheo thứtựtạiX, Y.GiảsửrằngZlàđiểm thứhai

AB với(ω1)

Đặt!P AD=P CA" =β,P DA!=P AC" =θ

(131)

B ≡ X, đó!ABD =AXD! =!AP D = 1800−(β+θ).Xét tam giácABD ta nhậnđược "

BAC +!ADB = 00,điều chỉxảy khiA, B, C, D thẳng hàng ( vơ lí) )

Ta có:

"

P QA=P CA" =!P AD=P XD!

Suy tứgiácP XQY nội tiếp Suy raBX ·BY =BP ·BQ

C A

D

Y

Z

P

Q

B

X

(ω1) (ω2)

Lại có: PB/(ω1) = BP ·BQ = BZ ·BA nên BX ·BY = BZ ·BA Điều có nghĩa

AXZY nội tiếp Dođó,

"

AZY =AXY! = 180◦−AXD!

Mặt khác, AXD! = !AP D = 180◦ − (β +θ) nên AZY" = β +θ Hơn nữa, AZC" = "

P AC+P CA" =β+θ.Vậy nênAZC" =AZY" =β+θ.MàZAC"+ZAY" =QAC" =!BP C = 2BAC" ⇒ ZAC" = ZAY" =αnên suy ra∆AZC = ∆AZY ⇒ AC =AY Điều dẫn tới

∆BAC =∆BAY ⇒BCA" =BY A." Vì

!

P BD=QBY!=|BQA" −BY A"|=|P CA" −BCA"|

3 Bài t

p t

luy

n.

Bài tập (USA TSTST 2015)Cho tam giácABC khôngđều GọiKa, La, Ma theo thứ tựlà

giaođiểm củaBC với phân giác trong, phân giác vàđường trung tuyến tam giácứng

với gócA Đường trịn ngoại tiếp tam giácAKaLa cắtAMa tạiđiểm thứ haiXa Định nghĩa

tương tựvới Xb, Xc Chứng minh tâmđường tròn ngoại tiếp tam giác XaXbXc nằm đường thẳng Euler tam giácABC

Bài tập (IMO 2010/4, Modified)Cho tam giácABCvới trực tâmH GiảsửP hình chiếu

củaHlên trung tuyếnCM tam giácABC Giaođiểm thứhai củaAP, BP, CP vớiđường

(132)

Bài tập (USA TST 2008)Cho tam giácABC với trọng tâmG P làđiểm diđộng trênđoạn

thẳngBC GọiQ, R cácđiểm trênAC, AB choP Q ∥ AB, P R ∥ AC Chứng

minh khiP diđộng trênBC,đường trịn ngoại tiếpAQRlnđi quađiểm cố địnhXthỏa

mãnBAG" =!CAX

Bài tập (EGMO 2016/4)Haiđường trịnω1,ω2có bán kính nhau, cắt haiđiểm

X1, X2 Xét đường tròn ω tiếp xúc vớiω1 tạiT1 tiếp xúc với ω2 T2 Chứng

minh rằngX1T1, X2T2cắt mộtđiểm nằm trênđường trònω

Bài tập (Mathematical Reflections O371)Cho tam giácABC với AB < AC.Gọi D, E

lần lượt chânđường vng góc củaB, CxuốngAC, AB.GọiM, N, P trungđiểm

củaBC, M D, M E GọiSlà giaođiểm củaN P vàBC.Đường thẳng quaAsong song vớiBC

cắtDEtạiT Chứng minh rằngST tiếp xúc vớiđường tròn ngoại tiếp tam giácADE

Bài tập (EGMO Shortlist 2012/G7) Cho tam giác ABC(AB < AC) nhọn với O tâm

ngoại tiếp tam giác.Q giao điểm phân giác ngồi gócA với BC P làđiểm bên

tam giácABC cho∆BP A∼∆AP C Chứng minh rằngQP A" +!OQB = 900.

Bài tập (Iranian Geometry Olympiad 2014) Tiếp tuyến A đường tròn ngoại tiếp

tam giác nhọn ABC(AB < AC) cắt BC P Gọi X điểm OP cho !AXP = 900. ĐiểmE, F lần lượt nằm cạnhAB, AC và ởcùng phía so vớiOP sao choEXP! = !

ACX,F XO! = ABX K, L! giaođiểm củaEF vớiđường tròn ngoại tiếp tam giácABC

Chứng minh rằngOP tiếp tuyến củađường tròn ngoại tiếp tam giácKLX

Bài tập 8.ChoP mộtđiểm trênđườngđối trung tam giácABC GọiO1, O2, Olần lượt

là tâm đường tròn ngoại tiếp tam giác AP B, CAP, ABC.Chứng minh rằngAO chiađôi O1O2

Bài tập 9.GọiM, N cácđiểm nửađường trịnđường kínhAB, tâmO X giao

điểm M N vớiAB GọiK giaođiểm thứhai hai đường trònM BO, N AO Chứng

minh rằngXK⊥KO

Bài tập 10.Cho tam giácABCnội tiếpđường trịn(O)cóđường caoAD.Đường trịn với tâm

trên ADvà tiếp xúc ngồi với đường tròn (BOC)tạiX Chứng minh rằngAX làđường đối

trung tam giácABC

Bài tập 11 (Dựa theođề đềxuất Luxembourg, IMO Shortlisted 2017,G2Cho haiđiểm

RvàSphân biệt trênđường trònΩvà∆là tiếp tuyến củaΩtạiR.GọiR′làđiểmđối xứng của

R quaS.LấyđiểmI cung nhỏcủa cungRS củađường trònΩ Giả sử đường tròn ngoại

tiếp tam giácISR′cắtđường thẳng∆tại haiđiểm phân biệt vàAlà mộtđiểm cắt gầnđiểmR.

Đường thẳngAIcắtΩtạiđiểm thứhaiJ vàKlà giaođiểm thứhai củaR′I vớiΩ Chứng minh

rằngSlàđiểmR′−Dumpty tam giácT AK

Bài tập 12 (IMO Shortlisted 2017, G3)Cho tam giác ABC nội tiếpđường tròn(O) H

(133)

thứtự P, Q.GiLlØim A-HumptycatamgicABC Chngminhrng LnmtrnØngtrnØngknh

HTviTltmØngtrnngoitiptamgicHPQ

Bài tập 13.Cho tam giácABC có trực tâmH.GọiM trungđiểm củađoạnBC, N giao

điểm củađoạn AM vớiđường tròn ngoại tiếp tam giácBHC D làđiểm nằm tam giác HBC.GọiX ∈ BD, Y ∈ CD choN trungđiểm củađoạnXY.GiảsửAD, BY, CX

đồng quy tạiL.Chứng minh rằngAlà trungđiểm củaLDvà bốnđiểmL, Y, X, D đồng viên

Bài tập 14 (Toán học tuổi trẻ, T12/492). Cho tam giác ABC nhọn với đường cao BE, CF ST dây cung củađường tròn ngoại tiếp tam giácAEF Hai đường tròn qua S, T tiếp xúc vớiđường thẳngBC tạiP, Q Chứng minh P E, QF cắt

đường tròn ngoại tiếp tam giácAEF

Bài tập 15. Trong mặt phẳng cho haiđường tròn (ω1) (ω2)cắt A B Một tiếp

tuyến chung haiđường tròn tiếp xúc với(ω1)ởP và(ω2)ởT Các tiếp tuyến tạiP vàT

đường tròn ngoại tiếp tam giác AP T cắt tạiS GọiH làđiểmđối xứng củaB quaP T

Chứng minhA, H, Sthẳng hàng

Bài tập 16 (USAMO 2008.) Cho tam giác ABC Gọi M, N, P trung điểm AB, AC, BC.Trung trực củaAB, AC cắt trung tuyếnAP theo thứtựtạiD, E GọiF giao

điểm củaBDvàCE Chứng minh bốnđiểmA, M, F, N đồng viên

Bài tập 17 (VN TST 2012.)Trên mặt phẳng, chođường tròn (O) haiđiểm cố định B, C

trên đường tròn cho BC khơng đường kính (O) Gọi A điểm di động

đường trịn (O)và Akhơng trùngB, C GọiD, K, J trungđiểm củaBC, CA, AB

vàE, M, N hình chiếu vng góc củaA, B, CtrênBC, DJ, DK Chứng minh

các tiếp tuyến tạiM, N củađường tròn ngoại tiếp tam giác EM N cắt tạiT cố định

khiAthayđổi trên(O)

Tài li

u

[1] Anant Mudgal,Gunmay Handa,A Special Point On The Median

[2] Tạp chí Tốn học & tuổi trẻ, ,Nhà xuất giáo dục Việt Nam

[3] Các toán diễnđànhttps://artofproblemsolving.com,https:// diendantoanhoc.net

[4] Trần Quang Hùng,Mỗi tuần tốn hình học, NXBĐHQG Hà Nội [5] Nguyễn Văn Linh,108 toán hình học sơcấp, NXBĐHQG Hà Nội

(134)

M

T S

B

Ổ ĐỀ

H

U D

NG TI

P C

N L

I GI

I

TRONG CÁC BÀI TỐN HÌNH H

C

Lê Vi

ế

t Ân

(TP H

Chí Minh)

G

IỚI THIỆU

Trong viết này, xin giới thiệu sốbổ đềrất hữu ích có thểáp dụng tốn hình học,đặc biệt có thểáp dụng tốt hình thi học sinh giỏi Các bổ đề phát biểuđơn giản lại cho góc nhìnđơn giản vẽthêm hình phụ, chíđặc biệt hóa chúngđểáp dụng vào tốn khác

1 B

ổ đề

1.

Đị

nh l

ý

Reim

Trong viết [1] tác giảngười Pháp Jean-Louis Ayme cóđưa kết quảvà gọi

định lí Reim nhưsau:

Bổ đề Cho haiđường tròn(O1)(O2)cắt tạiAB Mộtđường thẳngđi quaAcắt

(O1)(O2)thứtựtạiA1 A2; mộtđường thẳngđi quaB cắt(O1)(O2)theo thứtựtại

B1B2 KhiđóA1B1 ∥A2B2.

Chng minh. Có nhiều trường hợp xảy cho hình vẽ, tác giảchỉchứng minh cho trường hợp nhưhình 1.1 Các trường hợp khác bạnđọc chứng minh tương tự

Ta cóA!1B1B = 180◦−A!1AB(vìABB1A1 nội tiếp)=A!2AB = 180◦−A!2B2B(vìABB2A2

nội tiếp)

Suy raA!1B1B +A!2B2B = 180◦ DođóA1B1//A2B2 (theo dấu hiệu hai góc phía

bù nhau)

Nhận xét Ngoài bổ đề1 vẫnđúng trường hợp suy biến cácđiểm trùng và

khiđó ta xem cạnh cặpđiểm trùng tiếp tuyến cặpđiểm trùng nhauđó hoặc haiđường trịn (O1)(O2)tiếp xúc với Các trường hợpđược minh họa hình vẽsauđây(xem hình 1.a, 1.b, 1.c, 1.d, 1.e, 1.f, 1.g, 1.h).

Điều ngược lại bổ đề1 cũngđúng Và ta gọi làđịnh lí Reimđảo với nội dung nhưsau:

(135)

Hình 1.1

(136)

Hình 1.b

(137)

Hình 1.d

(138)

Hình 1.f

(139)

Hình 1.h

lấy cácđiểmA2, B2 khác tươngứng cho A1B1 ∥ A2B2 thì khiđó tồn tạiđường tròn(O2)

đi qua bốnđiểmA, B, A2, B2

Từbổ đề1 (định lí Reim thuận) định lí Reimđảo, ta có hai hệ quảrất hay dùng để chứng minhđường thẳng tiếp xúc vớiđường tròn haiđường tròn tiếp xúc với

Hệquả1 Cho tam giácABC nội tiếpđường tròn(O) Mộtđường tròn(O′)đi qua cácđỉnhB

C lần lượt cắt cácđường thẳngAB AC tạiB′ C′ khácA.Đường thẳngtđi quađỉnh A Khiđóttiếp xúc với(O)khi chỉkhit ∥B′C′.

Hệ Cho tam giácABC CácđiểmB′ C′ lần lượt nằm cácđường thẳngAB

ACvà khác với cácđỉnh tam giác Khiđó haiđường trịn ngoại tiếp tam giácABC

AB′C′tiếp xúc với nhau(tạiA)khi chỉkhiBC ∥B′C′.

Bây giờtađi xét sốthí dụsau:

Ví dụ (P1, IMO 2018) ChoΓlàđường tròn ngoại tiếp tam giác nhọnABC Cácđiểm D E lần lượtđược chọn các đoạn thẳngAB AC sao choAD = AE Cácđường

trung trực của BD CE lần lượt cắt cung nhỏAB AC củaΓ tại cácđiểmF G.

Chứng minh rằngDE F Gsong song với nhau.

Li gii.. Từnội dung bổ đề1, rõ ràng có hai hướng vẽ đường phụ đểáp dụng bổ đề1 Hướng thứnhất, xét hai giaođiểm cặpđường thẳngF DvàGE vớiđường trònΓ;

hướng thứhai xét hai giaođiểm cặpđường thẳngF EvàGD vớiΓ Sauđó tìm cách

chứng minh hai giao điểm hướng với điểmD E đồng viên Tuy nhiên, vẽ

hình quan sát chúýrằng từgiảthiết cóF D =F B(tươngứngGE =GC) Do

đó hướng thứhai hướng tốt nhấtđểcó thểsửdụngđượcF D =F B Cụthểchúng tađiđến

lời giải nhưsau (xem h 1.2):

(140)

Hình 1.2

Dễ thấy △F BD ∼ △AHD(g.g) Kết hợp vớiF B = F D suy AH = AD Tương tự, ta

cũng cóAK =AE DođóAH =AD=AE =AK, suy bốnđiểmD, E, H, Kcùng thuộc

đường tròn⊙(A;AD)

Áp dụng bổ đề cho hai đường tròn ⊙(A;AD)và Γ cắt H K với hai cát tuyến HDF vàKEGthìDE ∥F G

Ví dụ (P4, IMO 2015) Cho tam giácABCnội tiếpđường tròntâmO.Đường trònΓtâm

A cắt đoạn thẳngBC tại các điểmD E sao choB, D, E C đôi phân biệt nằm

trên đường thẳngBC theođúng thứ tự GọiF G là giaođiểm củaΓ, cho

A, F, B, C Gnằm trêntheo thứ tự GọiK là giaođiểm thứhai củađường trịn ngoại

tiếp tam giácBDF vàđoạn thẳngAB GọiLlà giaođiểm giaođiểm thứhai củađường tròn

ngoại tiếp tam giác CGE đoạn CA Giả sửcác đường thẳngF K GL phân biệt cắt

nhau tạiđiểmX Chứng minh rằngX nằm trênđường thẳngAO.

Li gii.. Giảsửcácđường thẳngF D, GE, F K, GLcắt lạiΩtại cácđiểmP, Q, Y, Z (xem h

1.3)

Áp dụng bổ đề cho hai đường tròn Γvà Ωcắt F G với hai cát tuyến F DP GEQ, ta cóP Q//DE ≡BC Dođó

BQ=CP (1)

Áp dụng bổ đề1 cho haiđường tròn⊙(BDF)vàΩcắt tạiB vàF với hai cát tuyếnBDC

vàF KY, ta có

(141)

Hình 1.3

Áp dụng bổ đề1 cho haiđường tròn⊙(BDF)vàΩcắt tạiB vàF với hai cát tuyếnBKA

vàF DP, ta có

KD ∥AP (3)

Từ(2) (3) suy raY C ∥AP Dođó

AY =CP (4)

Chứng minh tương tự, ta có

AZ =BQ (5)

Từ(1), (4) (5), suy raAY = AZ, suy raAO ⊥ Y Z Từ đóY Z ∥ F GnênF GY Z hình

thang cân Suy giaođiểm haiđường chéoF KvàGLphải nằm trênđường trụcđối xứng

hình thang, trung trực haiđáyF GvàY Z Tức làX ∈AO

Nhận xét Hầu hết lời giải cho tốn cách biếnđổi gócđểchứng minhXF =

XG Tuy nhiên cách sửdụng bổ đề1 cách khéo léo, chúng tađã có lời giải rất

nhẹnhàng cho tốn mà khơng cần qua bước biếnđổi góc Hơn với cách giải tương tự, cịn có thể mở chứng minh cho tốn mởrộng tốn cách thayđường trịnΓtâmAbởiđường trịnΓcó tâm nằm trênđường thẳngAO.

Ví dụ (P3, IMO 2013) Đường trịn bàng tiếp gócAcủa tam giácABC tiếp xúc với cạnh BC tạiđiểm A1 . ĐiểmB1 trên CA vàđiểm C1 trên AB đượcđịnh nghĩa cách tương tự bằng cách xét đường trịn bàng tiếp góc B và gócC tươngứng Giảsử tâmđường trịn ngoại

(142)

Hình 1.4

Li gii.. GiảsửIa, Ib, Iclà tâmđường trịn bàng tiếp gócA, B, Ccủa tam giácABC Gọi

Ma, MbvàMc thứtựlà trungđiểm củaIbIc, IcIavàIaIb

DễthấyA, B, C chân cácđường cao tam giácIaIbIc (xem h 1.4)

Vì sáu điểmA, B, C, Ma, Mb, Mc nằm đường tròn Euler tam giác IaIbIc Ta có

"

BAC = BM!aC nênBM!aC1 = CM!aB1 Lại cóM!aBC1 = M!aBA = M!aCA = M!aCB1

MaB =MaCnên△MaBC1 =△MaCB1 Suy raMaB1 =MaC1

Chứng minh tương tự, ta có MbC1 = MbA1 McA1 = McB1 Các điều chứng tỏ

rằng cácđường trung trực cácđoạn thẳngB1C1, C1A1, A1B1 vớiđường tròn ngoại tiếp tam

giác ABC điểm bađiểm Ma, Mb, Mc Do giả thiết tâmđường tròn ngoại tiếp

tam giácA1B1C1 nằm trênđường trịn ngoại tiếp tam giácABC, suy có mộtđiểm ba

điểmMaMbMc tâmđường tròn ngoại tiếp tam giácA1B1C1

Khơng tính tổng qt, ta giảsửMalà tâmđường trịn ngoại tiếp tam giácA1B1C1 Khi

đóMaMc ⊥A1B1 MàMaMc ∥AC vàIbB ⊥CAnên

A1B1 ∥IbB (6)

GọiOlà tâmđường trịn ngoại tiếp tam giácIaIbIc Vì bốnđiểmIb, Ic, B, Ccùng nằm

đường trịnđường kínhIbIc nên theoOIa ⊥BC(một kết quảquen thuộc), màIaA1 ⊥BC

A1 nên bađiểmIa, A, Othẳng hàng Từ bốnđiểmC, B1, A1 vàOcùng nằm trênđường trịn

đường kínhOC Lại từ(6) nên theo phầnđảo bổ đề1, ta có bốnđiểmIb, B, C, Ocùng nằm

trên mộtđường tròn, suy raOnằm trênđường trònđường kínhIbIchayI"bOIc = 90◦ Từ

ýrằngOIb ⊥CAvàOIc ⊥AB nênBAC" =I"bOIc = 90◦(cặp góc có cạnh tươngứng vng

góc)

Nhận xét Có thểnói rằng, bổ đề1 như phầnđảo hữu hiệu tốn mà

(143)

Ví dụ (P3, IMO 2015) Cho tam giác nhọn ABC vớiAB > AC KýhiệuΓlàđường tròn

ngoại tiếp, H là trực tâm và F là chân đường cao hạtừ A của tam giácđó Gọi M là trung

điểm củaBC GọiQ điểm trên Γ sao choHQA! = 90◦, gọi K điểm trên Γ sao cho

!

HKQ = 90◦ Giả sử rằng cácđiểm A, B, C, K Q đôi phân biệt nằm trên Γ theo

đúng thứtự Chứng minh rằngđường trịn ngoại tiếp tam giácKHQF KM tiếp

xúc với nhau.

Li gii.. Kẻ đường kínhAA′ củaΓthì ta cóCH ∥BA′ (vì vng góc vớiAB ) Tương

tựBH ∥CA′ DođóBHCA′ là hình bình hành Suy raM là trungđiểmHA′(xem h 1.5).

Hình 1.5

Lại có AQA!′ = 90◦ = AQH!nên ba điểm Q, H, A′thẳng hàng Từ đó bốn điểm Q, H, M, A′

thẳng hàng

Đường thẳng AH cắt lại Γ tạiH′ AHA!′ = 90◦ Do đó nếu gọiD := QK ∩A′H′ thì từ

!

HKD=DH!′H = 90◦, ta có tứgiácDKHH′nội tiếpđường trịnđường kínhDH.

Vẽ đường trịn (N)đường kínhDH N trung điểm củaDH Ta cóBC ∥ A′D(vì cùng

vng góc vớiAH), kết hợp vớiBCđi qua trungđiểm HA’ , suy BC làđường trung bình

của tam giácHA′DnênBCđi qua trungđiểm củaHDhayN ∈BC.

Áp dụng bổ đề1 cho hai bộbốn điểm nằm đường tròn dạng suy biến⊙(HKQ)và

⊙(HKD)với hai cát tuyếnHHD vàKQDthì ta cóQH ∥DD(với kýhiệuDDlà tiếp tuyến

của(N)tạiD), mà theo tính chất tiếp tuyến thìDD⊥N D, suy raN D ⊥QH ≡QA′ tạiH

Từ ta cóN K2 = N D2 =N F N M (hệthức lượng cho tam giác vuôngHM N).Điều này

chứng tỏ

(144)

Lại vìN H ⊥QH nênN H tiếp xúc vớiđường trịnđường kínhQH (cũng làđường trịn

ngoại tiếp HKQ), suy N KH! = N HK! = HQK! nênN K tiếp xúc với đường tròn

ngoại tiếpHKQ, kết hợp với (7) ta suy rađpcm

Nhận xét Khéo léo áp dụng bổ đề1ởdạng suy biến cho lời giảiđẹp.

2 B

ổ đề

2 B

ổ đề

góc th

nh

t

Trong kì thi BrMO 2012-2013, vịng có toán sau:

Bổ đề Cho tam giác ABC điểm P nằm tam giác cho ABP" = ACP" Dựng

hình bình hànhP BQC KhiđóQAB" =CAP".

Đây kết liên quan chặt chẽ đến tứ giác nội tiếp đường tròn,điều thấy rõ ta gọi giaođiểm cácđường thẳngBP, CP vàAC, AB tươngứng

giaođiểmđó với cácđỉnhB, C nằm mộtđường tròn Sauđây điđến

một sốchứng minh cho bổ đề2:

Chng minh. Cách 1.Nhận xét yếu tốhình bình hành "gợi" cho vềquan hệsong song cạnhđối diện hình bình hành Dođó sẽ"tận dụng" cặp góc so le,đồng vịtrong biếnđổi góc, cácđoạn thẳng nhauđểxem xét hai tam giác Vì sẽdựng thêm hình bình hànhđểtạo sựthuận lợi phép chứng minh Cụ

thểnhưsau:

Dựng hình bình hành AP BR(xem h.2.1) Dễ thấy ACQR hình bình hành △P AC =

(145)

△BRQ(c.c.c) DođóP CA" =BQR" ChúýrằngP BA" =BAR" (so le trong) Vậy

"

BQR=P CA" =P BA" =BAR."

Suy tứgiácAQBRnội tiếp Dođó

"

P AB =ABR" =AQR" =QAC."

Cách 2.Chúýrằng hình bình hành có tâmđối xứng Dođó, xem xét cấu hình bổ đề1, sẽtận dụng phépđối xứng tâm với tâm trungđiểm củaBC Cụthểnhưsau:

GọiM trungđiểm cạnhBC; dựng hình bình hànhABA′C; kẻtừP cácđường song song với

các cạnhAB, AC cắtAB, AC, A′BthứtựtạiE, F, G(xem h 2.a).

Hình 2.a

Dễthấy△P BE ∼ △P CF (g.g) hình bình hànhAEP F, BEP GvàA′CF G Dođó

EP EA =

EP F P =

EB F C =

GP GA′,

kết hợp vớiP EA" =AF P" =P GA!′.

Suy ra△P EA∼ △P GA′(c.g.c) DođóP AE" =P A!′G.

Xét phép đối xứng tâm M: qua phép biến hình điểm A, B, C, P theo thứ tự biến

thànhA′, C, B, QnênQAC" =P A!′B.

VậyP AB" =P AE" =P A!′B =QAC".

(146)

sẽcho nhìn nhanh chóng việcđịnh hướng lời giải vẽthêm hình phụ Cụ

thểnhưsau:

GọiB′ =BP ∩AC vàC′ =CP ∩AB Dễthấy△ABC ∼ △AB′C′ Dođó

dựngđiểmP′ sao cho△ABC∪{P}∼ △AB′C′∪{P′}(xem h.2.b).

Hình 2.b

Dễ thấyB′P′CP là hình bình hành Dođó△ABC ∪{P;Q} ∼ △AB′C′∪{P′;P} Suy ra

"

QAC =P AC!′ =P AB".

Cách 4. Từ giả thiếtP BA" = P CA" ý !P BQ = P CQ" Suy raQBA" = QCA" Do

chúng ta sẽnghỉ đếnđịnh lí sin tam giác Thật vậy, áp dụngđịnh lí sin cho tam giác

ABP, ACP (xem h.2.1 h.2.a h.2.b), ta có: P B

sinP AB" =

P A

sinP AB" =

P A

sinP AC" =

P C

sinP AC" =⇒

sinP AB"

sinP AC" = P B P C

Tương tự, áp dụngđịnh lí sin cho tam giácABQ, ACQta thuđược

sinQAC"

sinQAB" = QC QB

ChúýP B =QC vàP C =QB, suy

sinP AB"

sinP AC" =

sinQAC"

sinQAB"

Đếnđây, kết hợp vớiP AB" +P AC" =BAC" =QAC" +QAB" Dođó ta phải cóP AB" =QAC"

và P AC" = QAB" (xem thêm viết ứng dụng tỉ số đoạn thẳng tỉ số lượng giác,

(147)

Nhận xét Chúýrằng bổ đề2 vẫnđúng khiđiểmP nằm phía gócBAC chứkhông nhất

thiếtđiểmP phải nằm tam giácABC.

Ta điểm qua sốbài tốn có thểáp dụngđược bổ đề1 sau:

Ví dụ (P5, ELMO 2012) ChoABC là tam giác nhọn vớiAB < AC, vàDElà các

điểm nằm cạnhBCsao choBD=CEDnằm giữaB E GiảsửcóđiểmP nằm bên

trong tam giácABC sao choP D ∥AEP AB" =EAC" Chứng minh rằngP BA" =P CA".

Li gii.. Dựng hình bình hànhBP CQ(xem h 2.3)

Hình 2.2

Khiđó dễthấy từBD = CE, suy raQE ∥ P D ∥ AE suy bađiểmA, E Qthẳng

hàng

Áp dụng bổ đề2, ta suy rađượcABQ" =ACQ" Và dođóABP" =ACP"

Ví dụ6 (IMOSL G2, 2012) ChoABCDlà tứgiác nội tiếp với haiđường chéoACBD

cắt tạiE Cácđường thẳngADBC kéo dài vềphíaAB cắt tạiF ChoG

điểm choECGD là hình bình hành, và H làđối xứng củaE quaAD Chứng minh rằng D, H, F, Gđồng viên.

Hướng dn. (xem h 2.3) Từ F CE" = F DE! Dođó theo bổ đề 2, ta có CF G" = !DF E =

!

DF H Dođó tứgiácDGF H nội tiếp chỉkhiDGH!+DHF! = 180◦ Tuy nhiên, chúý

rằngF HG! =F ED!(tính chấtđối xứng trục) và!F EB +F ED! = 180◦ Dođó táo quy về

chứng minhF EB!=F GD!.Điều nàyđồng nghĩvới việc chứng minh△F EB ∼ △F GD, tức

là ta chỉcần chứng minh!F BE =!F DG

(148)

Hình 2.3

Nhận xét Rõ ràng, biếtđến bổ đề2 ví dụ6 thấy rõđược hướngđi lời

giải cách nhanh chóng.

Ví dụ ( G4, IMOSL 2009 ) Cho tứgiác nội tiếpABCDcó cácđường chéoAC BDcắt

nhau tại E, cácđường thẳngAD BC cắt tạiF Gọi G H theo thứ tựlà trung

điểm củaAB CD Chứng minh rằngEF là tiếp tuyến tạiE củađường trònđi qua cácđiểm E,GH.

Li gii.. Dựng hình bình hànhAEBM vàCEDN (xem h 2.4)

(149)

KhiđóG, H trungđiểm củaEM vàF N DođóGH ∥M N Suy

!

EHG=EN M ! (8)

Áp dụng bổ đề2 cho tam giác F AB với chúýF AE" =!F BE, cho tam giácF CD với

ý F CE" = !F DE Suy M F A! = !EF B = EF C" = N F D! Suy bađiểm A, M, N thẳng

hàng

Dễthấy△EAB∪{M;E;G}∼ △F CD∪{E;H;N} Suy

"

F EG=F N H.! (9)

Từ (8) (9), suy raF EG" = EF G", tức làEF tiếp xúc với đường tròn ngoại tiếp tam giác EF G

Nhận xét Bài toán khiE, F, Gkhông thẳng hàng.Điều tươngđương vớiAB CD không song song với nhau.

Ví dụ8 (Đài Loan TST 2014) Cho tam giácABC vàđiểP nằm tam giác, cácđường

thẳngAP, BP, CP theo thứtựcắt lạiđường tròn ngoại tiếp tam giácABCtạiT, S, R ChoU

là mộtđiểm nằm trên đoạn thẳngP T Một đường thẳng quaU song song vớiAB cắtCR tại W, vàđường thẳng khác quaU song song vớiAC cắtBStạiV Cácđường thẳng quaB song

song vớiCP, quaC song song vớiBP cắt tạiQ GiảsửrằngRS V W song song với

nhau, chứng minh rằngCAP" =BAQ".

Hướng dn. GọiE =BP ∩AC vàF =CP ∩AB(xem h 2.5)

Hình 2.5

(150)

Áp dụngđịnh lí Thales, ta có P V P E =

P U P A =

P W

P F Suy raV W ∥EF DođóRS ∥V W ∥EF

Ta có tứ giácBCSR nội tiếp với ý V W ∥ RS nên theo định lí Reim đảo, ta có tứ giác BCW V nội tiếp Tiếp tục áp dụngđịnh lí Reimđảo cho tứgiácBCSRvới chúýEF ∥RS, ta

có tứgiácBCEF nội tiếp

3 B

ổ đề

3.

Đị

nh l

ý

Koutras

Dướiđây tác giảxin trình bày tiêu chuẩn vng góc haiđường thẳngđược biếtđến với tên gọi làđịnh lí Koutras Kết mặt dùđã biết đến trước năm 1980, nhiên Stathis Koutras, nhà nghiên cứu người Hy Lạpđã có áp dụngđẹp cho tiêu chuẩnđơn giản (hơn 300 tốn có thểáp dụng nó) phổbiến kết quảnày Nội dung củađịnh lí

sau:

Bổ đề3 (Định lí Koutras) Cho tam giácABCvàđoạn thẳngM N GọiM1, N1lần lượt hình chiếu củaM, N lênAB;M2, N2lần lượt hình chiếu củaM, N lênAC KhiđóM N ⊥ BC khi chỉkhi AB

M2N2 =

AC M1N1.

Chứng minh. Có nhiều chứng minh cho bổ đề 3(xem thêm [5]), tác giả xin giới thiệu cách chứng minh trụcđẳng phương (xem h 3.1)

Hình 3.1

Đường thẳng quaM theo thứtựsong song vớiAB, AC cắtN N1, N N2 tươngứng tạiH, K

cắtBCtạiP, Q Gọi(I),(J)làđường tròn ngoại tiếp tam giácHP N, KQN

Dễ thấy hai tam giác ABC M P Q có cặp cạnh tương ứng đôi song song

(151)

giác Suy

AB M P =

AC

M Q (10)

Mặt khác, lại có

M H =M1N1 vàM K =M2N2 (11)

DễthấyI, J trungđiểm củaN P, N Qnên

IJ ∥P Q≡BC (12)

Vậy AB M2N2 =

AC

M1N1 (vì (10) (11))⇐⇒ M H.M P = M K.M Q⇐⇒ PM/(I) =PM/(J) ⇐⇒

M N trụcđẳng phương của(I)và(J)⇐⇒M N ⊥IJ ⇐⇒M N ⊥BC(vì (12))

Tađi xem xét sốví dụminh họa cho sựáp dụng bổ đề3:

Ví dụ (Định lí Brahmagupta) Đường thẳng nối giaođiểm haiđường chéo vng góc trong tứgiác nội tiếpđường tròn với trungđiểm cạnh bên ln vng góc với cạnh bênđối diện.

Li gii. Xét tứgiác ABCDnội tiếp cóAC ⊥ BDtại E M trungđiểm củaAB Ta cần

chứng minhM E ⊥BC(xem h 3.2)

Hình 3.2

GọiH, K hình chiếu vng góc củaM lênAE, BE KhiđóH, K trung

điểm củaAE, BE.ChúýtứgiácABCDnội tiếp nên EA.EC =EB.ED=⇒ EH

EK = EA EB =

ED EC

(152)

Nhận xét Cách chứng minhđịnh lí Brahmaguptaởtrên hướngđi lạ Khơng những

thế, cịn có thểmởrộng kết quảnày chođườngđối song tam giácởví dụtiếp theo sauđây mà cách chứng minh hồn tồn tương tự.

Ví dụ 10 Cho tam giác ABC nội tiếpđường tròn(O) Chứng minh rằngđườngđối songứng

vớiđỉnhAcủa tam giác vng góc vớiAO(đườngđối songứng vớiđỉnhAlàđường thẳng có

ảnhđối xứng quađường phân giácđỉnhAcủa tam giác song song vớiBC).

Hướng dn. Choℓlà mộtđườngđối song củaứng vớiđỉnhAcủa tam giác, ta cần chứng minh AO⊥ℓ GiảsửCA∩ℓ =EvàAB∩ℓ=F KhiđóB, C, E, F đồng viên Phần bạn

đọc chứng minh hồn tồn tương tựnhưví dụtrước

Nhận xét Ví dụnày kết quả cơbản củađườngđối song tam giác Kết quả này

cũng có nhiều áp dụng bạnđọc nên nhớ.

Ví dụ11 Cho haiđường thẳngxx′ không song song với Cácđiểm phân biệtA, B, C

thuộcxvà cácđiểm phân biệtA′, B′, C′thuộcx′ Gọia, b, clà cácđường thẳng lần lượtđi qua

A, B, C và vng góc với x; cácđường thẳnga′, b′, c′ lần lượtđi quaA′, B′, C′ và cùng

vng góc vớix′ GọiM =a∩a′,N =b∩b′ P =c∩c′ Chứng minh rằngM, N, P thẳng

hàng chỉkhi AB AC =

A′B′

A′C′.

Lời giải Gọi Q = x∩x′; tồn cặp điểm D ∈ x D′ ∈ x′ cho D ̸= D′ DD′ ⊥M N (xem h 3.3)

Hình 3.3 Áp dụng bổ đề3, từM N ⊥DD′ suy ra

AB QD′ =

A′B′

QD (13)

Ta cóM, N, P thẳng hàng⇐⇒P M ⊥DD′⇐⇒ AC QD′ =

A′C′

QD ⇐⇒

AB AC =

A′B′

(153)

Nhận xét Ví dụnày kết quảcơbản có nhiều áp dụng bạn đọc nên nhớ.

Hơn cịn có thểchứng minh kết quảtổng qt nhưsau:

Cho gócBAC và cácđoạn thẳng M N KL Gọi M1, N1, K1, L1 theo thứ tự là hình chiếu của M, N, K, Llên AB, và M2, N2, K2, L2 là hình chiếu vng góc củaM, N, K, LlênAC. Khiđó M1N1

K1L1 =

M2N2

K2L2 khi chỉkhi haiđường thẳngM N KLcùng phương (song song hoặc

trùng nhau).

Ví dụ 12 (P5, IMO 1985) Mộtđường tròn tâmOđi qua cácđỉnhAC của tam giácABC

và cắtAB BC lần lượt cácđiểm thứhaiK N GọiM là giaođiểm khácB của các

đường tròn ngoại tiếp tam giácABCKBN Chứng minh rằngOM B! = 90◦.

Li gii. VìM khácB nên tam giácABC khơng cân tạiB Khơng tính tổng qt, giảsử AB < BC (xem h 3.4)

Hình 3.4

Ta cóBM, KN, ACđồng quy tâmđẳng phương bađường trịn(O),⊙(ABC),⊙(BKN)

GọiP :=BM ∩KN ∩AC;BB′ làđường kính của⊙(ABC)vàL=BB′∩KN.

Áp dụng tính chấtđiểm Miquel cho tứgiác tồn phầnACN KBF, ta cóM ∈ ⊙(CN P)

Và áp dụng tính chấtđườngđối song (xem ví dụ10) cho tam giácABC, ta cóBB′ ⊥ KN LnênB!′LN = 90◦ =B!′CN Suy bốnđiểmB′, C, N, Lđồng viên Dođó

BL.BB′ =P

B/⊙(B′CN L)=BC.BN =PB/⊙(CN P) =BP BM

Suy raB′, L, M, P đồng viên hayB′M ⊥BP Dođó tốn sẽ được chứng minh nếu ta chỉ

rađượcO ∈B′M hayB′O ⊥BP

Thật vậy, gọi O′ tâm ⊙(ABC);G, H trungđiểm củaAC, CN; R,S lần

(154)

củaACnênAR∥O′H ∥BS nên

RS = 2HS (14)

Mặt khác, lại có

N C = 2GC (15)

Áp dụng bổ đề với ý BB′ ⊥ P N Ta có N CRS = BCP C Kết hợp với (14) (15), suy

HS GC =

BC

P C Dođó lại theo bổ đề3, ta cóB′O⊥BP Bài tốnđược chứng minh

Nhận xét Bài tốnđược xửlí theo hướng sử dụng bổ đề3 cho ta cách nhìn thú vịvà

tựnhiên.

Ví dụ 13 (IMOSL G5, 2005) Cho tam giác nhọnABCvớiAB ̸=AC,H là trực tâm tam

giác và M là trungđiểm củaBC Các điểmDtrênAB E trênAC và choAE =AD

D, H, E thẳng hàng Chứng minh rằngHM vng góc với dây cung chung cácđường

tròn ngoại tiếp tam giácABC ADE.

Li gii.. Gọi(O),(K)là cácđường tròn ngoại tiếp tam giácABC, ADE;Glà giaođiểm

khácAcủa(O)và(K) Ta cần chứng minhHM ⊥AG(xem h 3.5)

Hình 3.5

DễthấyOK trung trực củaAGnênOK ⊥AG Dođó ta chỉcần chứng minhHM ∥OK

Thật vậy, gọiO1, K1, M1, H1 hình chiếu củaO, K, M, H lênAB, O2, K2, M2, H2

lần lượt hình chiếu củaO, K, M, H lênAC

Ta cóO1K1 =O1A−K1A= CA2 − EA2 = CE2 Tương tự,O2K2 = BD2 Suy

O1K1

O2K2

= CE

(155)

Mặt khác, lại cóAEH! =ADH! (vìAE = AD) ECH! =DBH! nên△ECH ∼ △DBH

DođóEHC! =DHB!, kết hợp vớiD, H, Ethẳng hàng Suy raDElà phân giác tạiđỉnh H tam giácBCH Dođó

HEvàHDtheo thứtựlà phân giác tam giácH1CH vàH2BH (17)

Dễdàng cóđược

hai tam giácH1CH vàH2BH đồng dạng ngược hướng (18)

Từ(17) (18) suy

CE BD =

CH1

BH2

= 2M1H1 2M2H2

= M1H1

M2H2

(19)

Từ(16) (19) suy O1K1

O2K2 =

M1H1

M2H2 DođóM H ∥OK VậyM H ⊥AG(đpcm)

Nhận xét Có thểáp dụng kết quảví dụ12đểgiải ví dụ13 Và ngược lại có thểsử dụng cách

chứng minh ví dụ13đểgiải ví dụ12.

Ví dụ 14 (Định lí "Bướm đơn") Cho tứ giác ABCD nội tiếp đường trịn(O)có hai đường

chéoACBDcắt tạiE.Đường thẳng quaE vng góc vớiOE cắtAD, BCthứtựtại P Q Chứng minh rằngEP =EQ.

Li gii. Gọi M, N thứ tự trung điểm AD, BC; H, K thứ tự hình chiếu E lên AD, BD; gọiF =AD∩BC(điểmF ln tồn haiđiểmP vàQxácđịnh) (xem h

3.6)

(156)

VìOE ⊥P Qnên áp dụng bổ đề3, ta có M H KN =

F Q

F P (20)

Vì△EAD ∼ △EBCvới chúýM, Nlà trungđiểm cặp cạnhAD, BCvàEH, EK

là haiđường cao hai tam giác Dođó△EAD∪{H;P}∼ △EBC∪{K;N} Dođó EH

EK = M H

KN (21)

Từ(20) (21), suy raEH.F P =EK.F Q Tức diện tích hai tam giácEF P vàEF Qbằng

nhau DođóElà trungđiểm củaP Q

Nhận xét Chứng minh định lí "Bướmđơn" có nhiều cách khác nhau, nhiên góc

nhìn cách sửdụng bổ đề3 hướngđi lạ.

4 B

ổ đề

4 B

ổ đề

góc th

hai

Tại facebook [6] nhóm hình học PERU GEOMETRICO, trongđó có tốn tác giảMiguel Ochoa Sanchez nhưsau:

Bổ đề Cho tam giácABCvà cácđiểmP thuộcAB QthuộcAC.Đường tròn ngoại tiếp

các tam giácABQACP cắt tạiDkhácA GọiM, N lần lượt trungđiểm củaBC

P Q Khiđó(AB, N M)≡(AC, AD)( mod π).

Chứng minh. GọiGlà trungđiểm củaBD(xem h 4.1)

Dễ thấy (P B, P D) ≡ (P A, P D) ≡ (CA, CD) ≡ (CQ, CD)( mod π) (BP, BD) ≡ (BA, BD)≡(QA, QD)≡(QC, QD)( mod π) Dođó

hai tam giácDBP vàDQCđồng dạng hướng (22)

Suy

DB DQ =

BP QC =

2GN

2GM = GN GM,

(−−→GM ,−−→GN)≡(−→QC,−−→BP)≡(−−→DQ,−−→DB)(vì (22))( mod 2π)

Dođó hai tam giácGM N vàDQB đồng dạng hướng

Suy ra(M N, AB)≡(N M, N G)≡(BQ, BD)≡(AC, AD)( mod π)

(157)

Hình 4.1

Bổ đề4ởdưới dạng tổng quát, dođóđặc biệt hóa kết quảnày sẽthuđược sốbài tốn cóý

nghĩa

Nếu choM N quađỉnhAcủa tam giác khiđóP Q∥BC Theo bổ đề4 khiđóADlà

đườngđối trungđỉnhAcủa tam giác Dođó thuđược tốn sau

Ví dụ 15 (P2, Balkan MO, 2009) Cho tam giácABC, cácđiểmM N lần lượt nằm trên

các canhj AB AC sao choM N ∥ BC GọiP là giao điểm củaBN CM. Đường tròn

ngoại tiếp tam giácBM P CN P cắt haiđiểm phân biệtP Q Chứng minh

rằngBAQ" =CAP".

Li gii. DễthấyQlàđiểm Miquel tứgiác tồn phầnAM P N BC DođóP ≡ ⊙(ABN)∩

⊙(ACM)∩ ⊙(BP M)∩ ⊙(CP N)(xem h 4.2)

GọiD :=AP ∩BC vàF :=AP ∩M N Khiđó, từM N ∥BCsuy raD, Elà trungđiểm BC vàM B

Áp dụng bổ đề4, ta cóBAQ" =EAC" =P AC"

Bây giờ,ởcấu hình bổ đề4, choADlà phân giác củaBAC" Khiđó,DB =DQ, DC = DP vì△DCQ ∼ △DP B Suy BP = CQ Hơn nữa, theo kết quảbổ đề M N tạo với haiđường thẳngAB, AC cặp góc nênM N song song vớiAD Ta

thuđược kết quảkinhđiển nhưsau

Ví dụ 16 Cho tam giácABC CácđiểmE F theo thứtự nằm tiaCABA sao

choBF =CE GọiM N theo thứtựlà trungđiểm củaBCEF Chứng minh rằngM N

(158)

Hình 4.2

(159)

Lời giải. GọiG=CF ∩BE;K =M N ∩ACvàDlàđiểm Miquel tứgiácAF GEBC

Áp dụng bổ đề4, ta có

!

BAD=M KC.! (23)

Dễ thấy △DBF ∼ △DEC, màBF = CE nên DB = DE Dođó AD phân giác

△BAE Suy

!

BAD =BAC." (24)

Từ(23) (24) suy raM KC! =!DAC DođóM N ≡M K ∥AD

Nhận xét Bài toán có nhiều cách giảiđơn giản trên, nhiên chúng tađi từbổ đề4

cho ta góc nhìn khácđểdẫnđến ví dụnày.

Ởcấu hình bổ đề4, chọn cácđiểmP, Qsao choP Qlàđườngđối song tam giácABC

thì thấy bổ đề4 mởrộng bổ đề2

Bây giờ, từcấu hình bổ đề4, chođường thẳngADđi qua tâmđường trịn ngoại tiếp

chúng ta thuđượcM N ⊥BC DođóN thuộc trung trực củaBC Tađiđến tốn sau

Ví dụ 17 Cho tam giácABC nội tiếpđường tròn(O). ĐiểmDnằm trênAOkhácA.Đường

tròn ngoại tiếp tam giác ABD, ACDtheo thứtựcắtAB, AC tạiE, F Chứng minh rằng

trungđiểm củaEF cáchđều haiđỉnhB C.

Li gii. GọiM, Nlần lượt trungđiểm củaBC, EF; vàHlà hình chiếu củaAlênBC(xem

h 4.4)

(160)

Áp dụng bổ đề4 với chúýrằngAH vàAOlàđẳng giác quaBAC", ta có

((M N, AB)≡(AC, AD)≡(AC, AO)≡(AH, AB)( mod π)

Suy raM N ∥AH ⊥BC DođóM N trung trực củaBC nênN B =N C (đpcm)

Cuối cùng, điđến áp dụngđẹp bổ đề4 thông qua tốn sauđây

Ví dụ 18 Cho tam giác khơng cân ABC nội tiếpđường trịn Γ, vàđường trịn nội tiếp tâmI

tiếp xúc với cạnhBC, CA, AB theo thứtựtạiD, E, F TiaAI cắtΓtạiL, vàđường thẳng EF cắtBC tạiG Cácđường tròn ngoại tiếp tam giácGBE GCF cắt tạiK

khácG Giảsử đường tròn ngoại tiếp tam giácDKLcắt lại cácđường thẳng BC AL thứ

tựtạiP Q Chứng minh rằngđường thẳngP Qchiađôi cácđoạn thẳngAG, BECF.

Li gii. GọiH, M, N trungđiểm củaAG, BE, CF thìH, M, N nằm

đường thẳng Gauss tứgiácBCEF; tađịnh nghĩa lạiđiểmP Qlà giaođiểm củađường

thẳng HM N với BC AL Do tốn trở thành việc chứng minh năm điểm D, K, L, P, Qcùng nằm mộtđường trịn (xem h 4.5)

Hình 4.5

Trước hết, gọiS giao điểm khácA củađường tròn (AEF)và Γ KhiđóS điểm Miquel

của tứgiác tồn phầnBCEF AG DođóS ∈ ⊙(CEG) Áp dụng bổ đề4, ta có

(P Q, P D)≡(HM, GC)≡(CA, CS)( mod π) (25)

Mặt khác, dễthấy có phép vịtựquayS1biến△SBF thành△SCE ChúýrằngBF =BD

vàCE =CD Suy SBSC = BFCE = BDCD DođóSDlà phân giác tam giácSBCnênSD

quađiểmLlà trungđiểm cungBC khơng chứaAcủaΓ Dođó

(161)

Từ(25) (26) suy ra(P Q, P D)≡(LQ, LD)( mod π) Vậy

bốnđiểmD, L, P, Qđồng viên (27)

GọiR=AL∩EF thìE trungđiểm củaEF KhiđóRM, RN cácđường trung bình

các tam giácBEF CEF Do đóRM ∥ AB, RN ∥ AC RM = BF

2 RN =

CE

2 Kết

hợp vớiAQlà phân giác củaBAC", suy raRQ phân giác tam giácRM N Suy

QM QN =

RM RN =

BF CE =

BD

CD ChúýcácđiểmD, Qcùng nằm trongđoạnBC vàM N nên

QM QN =

DB

DC (28)

GọiJ :=BE∩CF Áp dụngđịnh lí Miquel cho tứgiác tồn phầnBCEF GJ ta cóK làđiểm

Miquel tứgiác DođóKlà giaođiểm khácJcủa haiđường trịn⊙(JBC)và⊙(JEF)

Dođó tồn phép vịtựquayS2 biến△KBE thành△KCF Chú ýrằngM, N theo thứ

tự trung điểm BE CF Do qua phép biến hình S2 M biến thành N Vì

(KM, KN)≡(BE, CF)≡(IB, IC)≡(IM, IN)( mod π) DođóK ∈ ⊙(IM N) Dođó

tồn phép vịtựquayS3 biến△KBC thành△KM N (29)

Từ(28) (29) suy qua phép biến hìnhS3biếnDthànhQ Dođó(DP, DK)≡(DC, DK)≡

(QN, QK)≡(QP, QK)( mod π) Dođó

bốnđiểmD, K, P, Qđồng viên (30)

Từ(27) (30) suy nămđiểmD, K, L, P, Qcùng nằm mộtđường trịn Ta cóđpcm

5 Bài t

p

Bài toán (IMOSL G1, 2015) Cho tam giác nhọn ABC với trực tâm H Lấy điểm G sao

cho tứgiácABGH là hình bình hành GọiI làđiểm trênđường thẳng GH sao choAC chia

đôiHI Giảsử đường thẳngACcắtđường tròn ngoại tiếpGCI tạiCJ Chứng minh rằng IJ =AH.

Bài toán (P2, AMPO 2017) Cho tam giácABC với AB < AC Gọi Dlà giao điểm thứ

hai phân giác gócBAC vàđường trịn ngoại tiếp tam giácABC GọiZ là giao

điểm trung trực củaAC với phân giác ngồi gócBAC Chứng minh trungđiểm

củađoạn thẳngAB nằm trênđường tròn ngoại tiếp tam giácADZ.

Bài toán (P1, Advanced, IGO 2018) Haiđường trònω1,ω2 cắt tạiAB GọiP Q một tiếp tuyến chung haiđường trịn vớiP ∈ω1Q∈ω2 MộtđiểmXbất kì nằm trênω1

Đường thẳngAX cắtω2 tạiđiểm thứhaiY. ĐiểmY′ ̸=Y nằm trênω2 sao cho QY = QY′.

Đường thẳngY′B cắtω

(162)

Bài toán (P4, Iranian TST 2018) ChoABC là tam giác(A!̸= 90◦). BE, CF là các

đường cao tam giác Phân giác gócAcắtEF,BC lần lượt tạiM, N ChoP là mộtđiểm

sao choM P ⊥EF N P ⊥BC Chứng minh rằngAP đi qua trungđiểm củaBC.

Bài toán (P3, USAMO 2005) ChoABClà ttam giác nhọn, vàP Qlà haiđiểm nằm

trên cạnhBC LấyđiểmC1sao cho tứgiác lồiAP BC1nội tiếp,QC1 ∥CAvà cácđiểmC1

Q1 nằm phía so vớiAB LấyđiểmB1sao cho tứgiác lồiAP CB1nội tiếp,QB1 ∥BA cácđiểmB1 Q1nằm phía so vớiAC Chứng minh cácđiểmB1, C1, P Qcùng nằm mộtđường tròn.

Bổ đề5 (P3, China TST 2018) Chođường trònωtiếp xúc với cạnhAB, ACcủa tam giác ABC lần lượt tại D, E, cho D ̸= B, E ̸= C BD+CE < BC Cácđiểm F, Gnằm

trên cạnhBCsao choBF =BDCG=CE Cácđường thẳngDGEF cắt tạiK.

ĐiểmLnằm cung nhỏDEcủađường trònω, cho tiếp tuyến củaωtạiLsong song với BC Chứng minh tâmđường tròn nội tiếp tam giácABC nằm trênđường thẳngKL.

Bài toán (P5, China TST 2018) Cho tam giác ABC với BAC >" 90◦, cho O là tâm

đường tròn ngoại tiếp vàω làđường tròn ngoại tiếp tam giác Tiếp tuyến củaω tạiA cắt

tiếp tuyến củaωtạiBClần lượt tạiP Q GọiE, F lần lượt chânđường cao kẻtừP, Q

đếnBC.F, Glà haiđiểm nằm trênđoạnP QkhácAsao choA, F, B, E A, G, C, Dlà các

bộ điểmđồng viên GọiM là trungđiểm củaDE Chứng minh rằngDF, OM, EGđồng quy.

Bài toán (P3, Intermediate, IGO 2015) Trong tam giácABC,M, N, P theo thứtựlà trung

điểm cạnhBC, CA, AB Dựng nữađường trịnđường kínhAB, AC ra bên ngoài

tam giác ABC. M K, M N cắt nữa đường tròn thứ tự tại X, Y Tiếp tuyến nữa

đường tròn tạiX, Y cắt tạiZ Chứng minh rằngAZ ⊥BC.

Bài toán (Bài O286, Mathematical reflections) Cho tam giác ABC với trực tâmH Gọi HM làđường trung tuyến vàHSlàđườngđối trung tam giácBHC GọiP là hình chiếu

vng góc củaAtrênHS Chứng minh rằngđường tròn ngoại tiếp tam giácM P S ABC tiếp xúc với nhau.

Bài toán (P4, IMO 2013) Cho tam giác nhọn ABC với trực tâmH, W là mộtđiểm tùy ý

nằm cạnhBC W nằm giữaB C CácđiểmM N lần lượt chânđường vuông

cao hạtừ cácđỉnhB C Kí hiệuω1 làđường trịn ngoại tiếp tam giácBW N, gọi X

điểm trênω1sao choW Xlàđường kính củaω1 Tương tựkí hiệuω2 làđường trịn ngoại tiếp tam giác CW M, gọi Y điểm trênω2 sao choW Y làđường kính củaω2 Chứng minh rằng bađiểmX, Y, H thẳng hàng.

Bài toán 10 (P2, EGMO 2016) Cho tứgiác nội tiếpABCD, cácđường chéoAC BD

cắt tạiX Gọi C1, D1 M theo thứ tựlà trung điểm cácđoạn thẳng CX, DX

CD Các đường thẳngAD1 BC1 cắt tạiY, vàđường thẳngM Y cắt cácđường chéo

AC BDthứtựtại cácđiểm phân biệtE F Chứng minh rằngđường thẳngXY tiếp xúc

vớiđường trònđi quaE, F X.

Bài toán 11 (Việt Nam TST 2006) Cho tam giác nhọnABCvà trực tâmH Phân giác ngoài

của tam giác ABC cắt cạnh AB, AC thứtự tạiD E. Đường phân giác góc BACcắtđường trịn ngoại tiếp tam giácADEtạiK Chứng minh rằngHKđi qua trungđiểm

(163)

Bài toán 12 Cho tam giác cânABC vớiAB =AC. T là mộtđiểm trong đoạnCA Trên tia BT lấy điểm S sao cho BS = BA Gọi O, I theo thứ tự tâm đường tròn ngoại tiếp tâm

đường tròn nội tiếp tam giácABT Chứng minh rằngOI vng gócCS.

Bài tốn 13 Cho tam giác khơng cânABCO, Itheo thứtựlà tâmđường trịn ngoại tiếp và

tâmđường tròn nội tiếp tam giác GọiM, N theo thứthứtựlàđối xứng cácđỉnhB, C

tươngứng quaCI, BI Chứng minh rằngOI vng góc vớiM N.

Bài tốn 14 Cho tam giác ABC không cân tại A và nội tiếp đường tròn (O). Đường trung

tuyến kẻtừ đỉnhA của tam giác cắt BC tạiM (O)tạiGkhácA.GọiH, K thứtự là hình

chiếu củaGlênAB, AC GọiN P theo thứtựlà trungđiểm củaBK CH Chứng minh

rằngAGtiếp xúc vớiđường trònđi quaM, N, P.

Bài toán 15 (P7, USA TSTST 2012) Cho tam giác ABC nội tiếpđường tròn Phân giác

trong gócAcắtBCthứtựtạiDL(khác vớiA) GọiM là trungđiểm cạnhBC.Đường

trịn ngoại tiếp tam giác ADM cắt cạnhAB AC thứ tự tạiQ P khácA GọiN

trungđiểm củaP Q, vàH là chân đường vng góc hạtừ N xuốngđường thẳngN D Chứng

minh rằngđường thẳngM Ltiếp xúc với ngoại tiếp tam giácHM N.

L

i k

ế

t

Bài viếtđã giới thiệu sốbổ đề điển hìnhđểbạnđọc có thêm cơng cụtrong việc giải tốn hình họcởcác kì thi HSG Tất nhiênđểgiải tốn hình học, cần phải khéo léo, tư trừu tượng nhạy bén.Để đạt nhữngđiều việc rèn luyện thường xuyênđểtăng thêm kinh nghiệm cho thân Vẫn nhiều bổ đềkhác tác giảsẽgiới thiệu cho bạnđọc vào viết khác Rất mong nhậnđược sựgópýcủa bạnđọc vềnội dung Cuối cùng, tác giả gửi lời cảmơnđến thầy Trần Nam Dũng bạn Ban biên tập

đọc lại thảo vàđưa gópýxácđángđểbài viếtđược hồn thiện

Tài li

u

[1] http://jl.ayme.pagesperso-orange.fr/Docs/Anton%20Reim.pdf [2] http://www.cut-the-knot.org/m/Geometry/Reim1.shtml

[3] https://www.imo-official.org/ [4] Diễnđàn AoPS, topic

https://artofproblemsolving.com/community/c6_high_school_ olympiads

(164)

B

ÀI

T

OÁN

H

AY

LỜ

I

G

I

I

Đ

P

Nguy

n Duy Liên

(Tr

ườ

ng THPT Chuyên V

ĩ

nh Phúc)

G

IỚI THIỆU

Giảiđược tốn Sốhọc hay khó, tađã cảm thích thú Nhưng tốn Sốhọc hay khó mà giảiđược nhiều cách mà từ ta có thểgiảiđược, hay tạo sốbài tốn lớp tốnđó niềm vui cịn nhân lên nhiều lần Bài viết này, xin giới thiệu với bạn4cách giải cho toán số2vềSốhọc hay khó kỳ thi Olympic Tốn học Hoa Kỳ2005(USAMO2005)

Bài tốn Chứng minh hệphương trình

(

x6Cx3Cx3yCy D147157 x3Cx3yCy2CyCz9 D157147

khơng có nghiệm ngunx; y; z:

Lời giải Ta sửdụng modulo19đểchứng minh hệphương trình khơng có nghiệm ngun

Ta có152D8!19; 157" #147 mod 19/:Bảng giá trịcủa5n.n2N!/theo mod19/: n 10 11 ! ! !

5n #8 #2 #3 ! ! !

Dođó 147157 " #5/13 " #54 " mod19/và 157147 " 53 " #8 mod 19/: Viết lại hệ

phương trình (

x6Cx3Cx3yCy D147157 x3Cx3yCy2CyCz9 D157147

Cộng tươngứng theo vếtađược !

x3CyC1"2Cz9 D147157C157147C1:

Lại thấy147157C157147C1" #5 mod 19/;từ suy

(165)

Mặt khác theođịnh lýFemat’s ta cóz18 "0; mod 19/suy raz9 " #1; 0; mod 19/do

đóđể(1) xảy thì.x3CyC1/2 " #4;#5; #6 mod 19/vơ lý, với sốnguyênathì a2 " #8; #3; #2; 0; 1; 4; 5; 6; 7; mod 19/: Vậy (1) xảy hệ phương

trình khơng có nghiệm nguyênx; yvàz:

Lời giải Ta sửdụng modulo13:Cộng vếtheo vếtađược

!

x3CyC1"2Cz9 D147157C157147C1:

Theođịnh lýFemat’s ta cóa12 "1 mod 13/;8a2 Z; a6"0 mod 13/;ta có 147157 "4157 "41 mod 13/; 157147 "1147 "1 mod 13/:

Lại có147157C157147C1"6 mod 13/;từ suy

!

x3CyC1"2Cz9 "6 mod 13/: (2)

Mặt khác với sốnguyênathìa3" #5; #1; 0; 1; mod 13/:Viết phương trìnhđầu tiên

của hệthành !

x3C1" !x3Cy"D147157 "4 mod 13/;

suy rax3 6" #1 mod 13/nên chỉcó thểx3 " 0; 1; 5; #5 mod 13/khiđó sẽtươngứng với x3Cy "4; 2; 5; #1 mod 13/;suy ra!x3CyC1"2 "12; 9; 10; mod 13/:

Vậyđểxảy (2) thìz9"7; 10; 9; mod 13/vơ lý, vìz3" #5; #1; 0; 1; mod 13/;suy

raz9 "5; 12; 0; 1; mod 13/:Vậy (2) khơng thểxảy ra, ta cóđiều phải chứng minh.

Lời giải Ta sửdụng modulo7; modulo8vàđịnh lýMihailescu’s Viết lại hệphương trình

( !

x3Cy" !x3C1"D147157 1/

!

x3Cy".yC1/D157147#z9 2/

Từphương trình.1/tađượcx3Cyˇˇ147157:Ta xét ba trường hợp sau:

$ x3Cy D ˙1WTừ.1/suy ra˙!x3C1"D147157;cho nên 147157#1Dx3_147157#1D.#x/3;

điều vơ lývìxchẵnx3 ".#x/3

"0 mod8/mà

147157#1D2 mod 8/; 147157C1D4 mod8/:

(166)

$ 3ˇˇx3Cy W Từ 1/ suy ˙3k!x3C1" D 147157; k Z; từ dẫn đến x3 C1 D

˙147157

3k ;cho nên3

157"k!72#157C1D.#x/3

_ 3157"k!72#157#1Dx3;suy raxchẵn,

và ta cóx3".#x/3 "0 mod 8/:

Phương trình3157"k!72#157C1D.#x/3 khơng xảy ra3157"k!72#157C1"2; mod 8/:

Phương trình3157"k!72#157#1Dx3đểxảy thì157#kD2r; r 2N!khiđó trởthành

32#r !72#157#1Dx3 vơ lý(định lýMihailescu’s1)

Từcác trường hợp trênđều không xảy ra, ta cóđiều phải chứng minh

Lời giải Ta sửdụng modulo9; modulo37và Bổ đềLTE Viết lại hệphương trình

(

.x3Cy/.xC1/.x2#xC1/D73143157 1/ x3Cy/.yC1/D.15749#z3/.15798C15749z3Cz6/ 2/

Ta thấy

$ x2#xC1D.x C1/2#3 xC1/C3suy gcd!xC1; x2#xC1""3

$ x C1/!x2#xC1" Dx3C1D7a!3b; a; b 2 N:

$ x2 !2xkhijxj!2tươngđươngx2#xC1!xkhijxj!2

Nếujxj< W

$ x D1_x D#1khiđó vếtrái của.1/chẵn suy ra147157 "0 mod 2/vô lý

$ x D0thay vào.1/tađượcyC1D147157;suy ray D147157#1;thếvào.2/tađược 147157!147157C1"D157147 #z9:

Mà148ˇˇ.147157C1/và148D4!37;suy ra37ˇˇ.147157C1/ ;dođó37ˇˇ.157147#z9/ ;

tươngđương vớiz9 "157147;hayz9 "9147 mod 37/;vơ lývì 1"z36 "!9147"4 "264 "26 mod 37/:

Nếujxj!2WTừ.1/chúýrằng cóxC1 D ˙3k !7j với cặp sốnguyên dương.j; k/

có nghĩa làˇˇx2#xC1ˇˇ " 3;điềuđó khơng thể Vì chỉcó thểxC1 D ˙3k; ˙7k

tươngứng vớix2#xC1D3!7m; 3mvớim; k 2N!:

$ NếuxC1 D ˙7k;thì.xC1/2#3 x C1/C3 D 3m;dođó 72k ˙3!7k C3 D 3m;

cho nên72k "0 mod 3/;vơ lý

1Định lýMihailescu’s:Nghiệm ngun dương của phương trìnhxa#yb D 1;vớia; b > 1x; y > 0

(167)

$ NếuxC1D ˙3k;thì.xC1/2#3 xC1/C3D3!7m;dẫnđến32k˙3kC1C3D3!7m;

cho nên 32k"1˙3k C1 D 7m;vì thế3k!3k"1˙1" D 7m#1:Theo bổ đềLTE ta có v3

!

3k!3k"1˙1""Dv3.7m#1/ ;suy rak D2Cv3.m/"2Clog3m:Vì

7m#1D3k#3k"1˙1$"32Clog2m!31Clog2m˙1"D9m 3m˙1/ : (3)

Vớim > 2thì (3) vơ lývì7m#1 > 9m 3m˙1/qua quy nạp

Với0"m"2W

ı VớimD2;từphương trình7m#1D3k.3k"1˙1/ta thấy khơng tồn tạik 2Z:

ı Vớim D1từphương trình7m#1D3k.3k"1˙1/;suy rakD1; x D#4;thếvào 1/; 2/ta có3155 ˇˇ.x3Cy/ nên

3155ˇˇ!15749#z3" !15798C15749z3Cz6": (4)

Để (4) chia hết hai thừa số vế phải (4) Nếu 9j15749#z3;thìz3"15749 "449 "4 mod 9/;vơ lý

Cịn 9j15798C15749z3Cz6;thì 15798

C15749z3Cz6 " 0 mod9/:Tuy

nhiên

15798C15749z3Cz6"7C4z3Cz6 "!z3C2"2C36"0 mod 9/:

ı Với m D từphương trình7m#1 D 3k!3k"1˙1"; suy k D 1; x D

giống nhưtrường hợp khimD1không tồn nghiệm nguyên hệ

Vậy dođó khơng có trường hợp hệcó nghiệm ngun

Bốn lời giải cho ta vẻ đẹp số, cách chọn modulo cho thậtđắt giá nhưviệc chọn modulo19ởlời giải1;chọn modulo 13ởlời giải2:Sựkết hợp nhẹnhàng logic

các modulođan xen vàođó phép biếnđổiđại số, lập luận sốhọc cho lời giải gần gũi hơnđối với học trị qua lời giải3và4:Đặc biệt tơiấn tượng với lời giải3đích cuối

lời giải dẫnđến tốn mớiđịnh lýMihailescu’s, mộtđịnh lýrất mớiđược hồn thiện cơng bốnăm2004;tình u với số thơi thúc vềtìm tịi lạ đãđưa tơiđên

(168)

HƯỚNG TỚI KỲ

THI VMO 2018-2019

Lê Phúc L

(L

p Cao h

c Khoa h

c t

nhiên TP.HCM)

L

ỜI BAN BIÊN TẬP

Kỳthi chọn HSG cấp quốc gia (viết tắt VMO) năm diễn vào ngày 13 14 tháng 01/2019 Trong viết này, tác giảsẽ đưa giới thiệu sốnội dung hướngđến rèn luyện chuẩn bịcho kỳthi này, bao gồm: phân tích sốbài tốn trongđềthi HSG tỉnh, giới thiệu sốbài toán hay,đềthi thửvàđáp án chi tiết

1 Phân tích m

t s

bài tốn trong

đề

thi t

nh 2018

1.1 Bài C

p s

c

ng - C

p s

nhân

đề

Ninh Bình

Ta xét tốn sau trongđềthi Ninh Bình

Bài toán Cho cấp sốcộng (an) và cấp số nhân(bn)đều có sốhạng dương Biết rằng

a1 =b1, an=bn.Chứng minh rằngak ≥bk với mọik = 2,3, , n−1

Bài tốn đềchọn kỹ sư tài củaĐH Bách Khoa Hà Nội 2012, có xuất trongđềthi HSG lớp 11 khơng chun TP.HCM năm trước với trường hợpn = 5.Bài

tốnđịi hỏi so sánh sốhạng "nằm giữa" hai CSC, CSN hữu hạn có sốhạngđầu cuối Nhưthếta cần tìm mối liên hệgiữa số ởgiữađó số ởhaiđầu?

an=a1+ (n−1)d;ak =a1+ (k−1)dhayak =

(n−k)a1+ (k−1)an

n−1

Tương tựthìbk= n−1

!

bn−k1 bk−1

n Áp dụng trực tiếp bấtđẳng thức AM-GM chon−1sốdương,

ta cóđpcm

Xem lại tốn trướcđó trongđềthi HSG QG 2012,đã có bàiđa thức tương tự, thực chỉlà giải phương trình bậc hai có liên quanđến CSC này; nămđó, nhiều thí sinhđã lúng túng, khơng tìmđược mối liên hệnhưtrên nên khơng xửlý tốn

Bài tốn (VMO 2012) Cho hai CSC (an),(bn)và xét dãy đa thức Pk(x) = x2 +a

kx+bk

với ≤ k ≤ n Giả sử P1(x), Pn(x) vô nghiệm Chứng minh các đa thức Pk(x) với

(169)

Tóm tắt lời giải VMO 2012 Theo giảthiết a2

1 −4b1 < 0, a2n−4bn < Ta cần chứng

minha2

k−4bk<0với1< k < n.Dùng công thức mối liên hệ ởtrên, tađưa

"

(n−k)a1+ (k−1)an

n−1

#2

−4(n−k)b1+ (k−1)bn

n−1 <0

⇔(n−k)2a21+ (k−1)2a2n+ 2(n−k)(k−1)a1an−4(n−1) [(n−k)b1+ (k−1)bn]<0

Theo bấtđẳng thức AM-GM thìa1an ≤ a

2 1+a2n

2 nên

2(n−k)(k−1)a1an ≤(n−k)(k−1)(a21+a2n)

Chúý(n−k)2+(n−k)(k−1) = (n−k)(n−1)và(k−1)2+(n−k)(k−1) = (k−1)(n−1)

nên ta viết thành

(n−k)(n−1)(a21−4b1) + (k−1)(n−1)(a2n−4bn)<0

Bấtđẳng thức cuốiđúng nên ta cóđpcm

Từ ýtưởng vềcơng thức mối liên hệ ởtrên, ta có thể"chếbiến"đềnhưsau:

• Xây dựng hai CSCứng với tungđộ, hồnhđộcủa dãyđiểm mặt phẳngOxy

• Giấuđi yếu tốCSC cách dùng cơng thứcđặc trưngan+1 = an+2an+2

• Chuyển thành dạng thức, lũy thừađểlàm khó yếu tốtuyến tính Từ ta có tốn sau (bài nàyđãđược dùngđềthi chọnđội tuyển củaẢRập)

Bài toán Trong mặt phẳng tọa độ Oxy, cho dãy các điểm phân biệt có tọa độ dương An(xn;yn)vớin ≥1,trongđó dãy sốthực dương(xn),(yn)xácđịnh công thức

xn+1 = $

x2

n+x2n+2

2 , yn+1 =

%√y

n+√yn+2

2

&2

Biết cácđiểmO, A1, A2015 cùng nằm mộtđường thẳngd Chứng minh không tồn tại

hai chỉsối, j với1< i < j <2015sao chođoạn thẳngAiAj cắtd.

Lời giải. Theo giảthiết, ta thấy hai dãy(xn),(yn)thỏa mãn x1

y1 =

x2015

y2015 = k > 0.Ta sẽchứng

minh với mọii= 2,2014thì xiyi > k Thật vậy, ta thấy xn+1 =

$ x2

n+x2n+2

2 ⇔x

2

n+1 = x2

n+x2n+2

2

nên dãy(x2

n)lập thành cấp sốcộng Dođó, với mọii= 2,2014,đặtαi = 20152014−i,βi =

i−1 2014

thìαi+βi = 1,2≤i≤2014và ta có

x2i =αix21+βix22015 ⇔xi =

!

αix2

(170)

Tương tự, ta có

yi = (αi√y1+βi√y2015)2 nênkyi = (αi√x1+βi√x2015)2

Ta cần có !

αix2

1 +βix22015 >(αi

x1+βi√x2015)2

với mọi2≤i≤2014(*) Tuy nhiên, (*)đúng theo bấtđẳng thức Cauchy-Schwarz (αi+βi)

'

αix21+βix22015 (

≥(αix1+βix2015)2 ≥(αi√x1+βi√x2015)4

Dođây cácđiểm phân biệt nênA1 ̸= A2015 ⇔ x1 ̸=x2015 đẳng thứcở(*) không xảy

Từ suy tất cảcácđiểmA2, A3, , A2014 nằm dướiđường thẳngd Ta cóđpcm

Cuối cùng, ta xét toán tương tựsau:

Bài toán Chứng minh tồn tại4037sốnguyên dương, bao gồm sốa1, a2, , a2019

là sốhạng cấp sốnhân vàb1, b2, , b2018 là sốhạng cấp sốcộng cho a1 < b1 < a2 < b2 < < a2018 < b2018 < a2019

1.2 V

bài toán

đ

a th

c trong

đề

KHTN

Ta xét toán sau trongđềthi KHTN Hà Nội

Bài tốn Tìm tất đa thứcP(x)hệsốthực cho

P2(x)−P(x2) = cx2018,∀x

Bài toán túy bàiđồng hệsố; nhiên, ta không thểlàm cách tổng quát từ đầuđược.Đểxửlýtriệtđể, ta dùng nhận xét quan trọng sau:

Bổ đề.Nếuđa thứcP(x)là tổng nhất3đơn thức (khác bậc) thìP2(x)−P(x2)có nhất

hai sốhạng

Chứng minh. Thật vậy, gọiaxm+bxnlà haiđơn thức có bậc thấp nhất củaP(x)vớim > n ≥0;

khiđó, trongP2(x)sẽcó chứa sốhạng2abxm+n Tuy nhiên, vì2n < m+n <2mnên sốhạng

này chắn không thểbịtriệt tiêu

Tương tựnếu xét haiđơn thức bậc cao Bổ đề chứng minh Trởlại toán, ta xét trường hợp sau

1 Nếu P(x) = axn thì P2(x)− P(x2) = (a2 − a)x2n nên đồng nhất hệ số có 2n = 2018, a2−a =cnênc≥ −1

(171)

2 NếuP(x) =axn+bxmthì

P2(x)−P(x2) = (a2−a)x2n+ 2abxm+n+ (b2−b)x2m

thì phải cóa =b = 1và2xm+n =cx2018 nênc= 2, m+n= 2018.

Vậy nếuc < −1

4 khơng tồn tạiđa thức thỏađề; nếuc≥ −

4, c ̸= 2thì cóđúng mộtđa thức

thỏa mãn có dạngaxnvớia2−a=c; nếuc= 2thì có cácđa thức thỏađềlà

−x1009,2x1009, xm+xn

vớim+n= 2018, m, n∈N.Bài toán nhẹnhàng

Tiếp theo, ta xét toán tương tựtrongđềthi chọnđội tuyển TPHCM 2014:

Bài tốn Tìm tất đa thứcP(x), Q(x)hệsốnguyên cho

P(Q(x)) =x2013+ 2014x+ 1,∀x∈R

Bài tương tựbài phải tìm cách thu hẹp sốtrường hợp cần xét khiđồng hệ

số; khó hơnởchỗta khơng thểdùng túyđại số đểlàmđiềuđó Ta xửlýnhưsau:

Đạo hàm hai vế, ta có

Q′(x)·P′(Q(x)) = 2013x2012+ 2014

Dùng tiêu chuẩn Eisensteinứng với sốnguyên tốp = 2, ta có vếphải bất khảquy Dođó, degP(x) > 1,degQ(x) > 1thì vơ lý Vì nên cần xét trường hợp degP(x) = 1hoặc degQ(x) =

Bài tốnđếnđó chỉcần xét trường hợp làm cẩn thận bướcđồng hệsốlàđược Dù có nhiều kỹthuậtđi kèm toán đa thức việcđồng bậc, hệsốcủađa thức công cụthuần túy hiệu Chẳng hạn tiêu chuẩn Eisenstein định lý đa thức nguyên cũngđược chứng minh bằngđồng hệsố

Tiếp theo, ta xét toán sau vềviệcđồng hệsố:

Bài tốn Tìm tất đa thức khác thỏa mãn

P(x)P(x+ 1) =P(x2+x+ 1),∀x∈R

Một lời giải kinhđiển dùng sốphức, chứng minh chỉcó nghiệm làx=±i

Ở đây, ta xét cách tiếp cận khác

Trước hết, rõ ràng P(x) vơ nghiệm có nghiệm x0 có nghiệm khác x2

0 +x0 + > x0 Điều cho phép xây dựng dãy vô hạn nghiệm P(x), kéo theo P(x) ≡ 0,không thỏa không xét đa thức Từ suy radegP(x)luôn chẵn

(172)

Ta sẽchứng minh tất đa thức thỏađềcó dạng(x2+ 1)n.Thật vậy,đồng nhất hệsốbậc

cao nhất, ta cóP(x)monic.ĐặtP(x) = (x2+ 1)n+Q(x)vớidegQ(x) =m <2n.Thay vào

đẳng thứcđã cho )

(x2+ 1)n+Q(x)* )(x2+ 2x+ 2)n+Q(x+ 1)* =+,(x2+x+ 1)2+ 1-n+Q(x2+x+ 1)

hay

(x2+ 1)nQ(x+ 1) + (x2+ 2x+ 2)nQ(x) = Q(x2+x+ 1)

NếuQ(x)≡0thì ta cóđpcm Nếu khơng so sánh bậc hai vế, tađược2n+m = 4n ⇒m= 2n, khơng thỏa Bài tốnđược giải

Bằng cách tương tự, ta có thểgiảiđược tốn sau:

Bài tốn Tìm tất cảcácđa thứcP(x)thỏa mãnP(x)P(x+ 1) =P(2x2+ 8x+ 6),∀x.

Đáp sốlàP(x) = (2x+ 3)n

Bài tốn Tìm tất cảcácđa thứcP(x)thỏa mãnP(x2−4x) =P2(x−6),∀x.

Đáp sốlàP(x) = (x+ 4)n

Cuối cùng, ta xét toán kinhđiển sau ước lượngđa thức:

Bài tốn Chođa thức hệsốngun, bậc chẵn, monicP(x)sao choP(n)là sốchính phương

với vơ sốn∈Z.Chứng minh rằngP(x)là bình phương mộtđa thức hệsốnguyên.

Lời giải. Bài toán trường hợp degP(x) = quen thuộc, với bậc cao

khơng dễ! Ta chứng minh bổ đềsau:

Bổ đề.Với mọiđa thức monic hệsốngunP(x)códegP chẵn tồn tạiđa thức hệsốhữu tỷ Q(x)sao cho

lim x→+∞

,/

P(x)−Q(x)-=

Thật vậy, ta có

/

P(x)−Q(x) = P(x)−Q

2(x) /

P(x) +Q(x)

Bậc mẫu lànnên cần tìmQđểbậc tử≤n−1.Nghĩa hệsốcủaxn, xn+1, , x2n bị triệt tiêu Tuy nhiên, rõ ràng ta chọn degQ(x) = n nên cón+ 1hệsố để “cơ

cấu”, tức chất giải hệphương trình cón+ 1biến

Ở đây, ta dùng cực hạn để thực điều dễ Trong đa thức monic Q(x) có degQ(x) =n, xétQ0(x)sao chod= deg(P(x)−Q02(x))min Giảsửd≥nthìP(x)−Q20(x) = axd+f(x)vớidegf < d XétQ(x) = Q

0+a2xd−nthì P(x)−Q2(x) =P(x)−,Q0(x) +

a

2x

d−n-2 =axd+f(x)−axd−nQ

0(x)−

a2x2d−2n

ChúýrằngQ0(x)monic nênaxdbịtriệt tiêu; ra2d−2n < nnênđa thức có bậc nhỏ

(173)

Ngoài ra, tồn M ∈ Z để M ·Q(x) ∈ Z[x] Xét dãy số a1 < a2 < a3 < số để P(ai)là sốchính phương, tức tươngứng tồn sốnguyên dươngb1 < b2 < b3 < để P(ai) = b2i Suy

M(Q(ai)−bi) =M

+

Q(ai)−/P(ai).+M+/P(ai)−bi

→0

khii→+∞.VìM(Q(ai)−bi)∈Znên tồn tạii0 đủlớnđể

|Q(ai)−bi|<1,∀i > i0 ⇒Q(ai) = bi,∀i > i0

Suy P(ai) = Q2(ai) với mọi i > i

0, điều với vô số i nênP(x) = Q2(x) Cuối

cùng, chúýrằng nếuQ(x)∈/ Z[x]thì tồn tạim ∈ ZvàmQ(x)làđa thức nguyên Suy

m2P(x) = (mQ(x))2 cũng làđa thức nguyên bản, vô lý Vậy nên

Q(x)∈ Z[x],bài tốnđược

giải hồn tồn

Trên cơsởlời giải trên, thửgiải toán mởrộng sau:

Bài toán GiảsửP(x)làđa thức hệsốnguyên, monic, cho tồn vô hạn cặp sốnguyên

dương(m, n)P(n) = m3+ 3m+ 1.Chứng minh rằng tồn tạiđa thức hệsốnguyênQ(x)

sao choP(x) = Q3(x) + 3Q(x) + 1.

1.3 V

bài tốn hình h

c trong

đề Đ

à N

ng

Dướiđây tốn tốn trongđềơn củaĐà Nẵng (tác giảchính thầy Trần Quang Hùng)

Bài toán Cho tam giác nhọn khơng cânABC nội tiếp(O)và một đường trịn(ω) cố định

qua A, O cắt AB, AC F, E Giả sử OE, OF cắt (O) M, N Gọi P điểm cung BC của (O) AP ∩ (ω) = L OL cắt (O) G, H (điểm G, N cùng phía so với AP). M P ∩CG= J, N P ∩BH =I.Chứng minh khiP thayđổi trênđường trịn thì(P IJ)

ln quađiểm cố định.

(174)

Ta có ∠ECD = ∠AOD2 = ∠AED2 nên tam giác ECD cân E hayEC = ED,màOC = OD

nênOElà trung trực củaCD

Suy raM trungđiểm cungCD của(O).DođóBM phân giác gócB tam giácBCD

Tương tựvớiN.Cũng cách trên, ta cóLD=LP vàOLlà trung trựcDP, kéo theoG, H

là trungđiểm hai cung của(O),suy raI, Jlà tâm nội tiếp tam giácDBP, DCP.Giảsử(P IJ)

cắt(O)ởT theo mơ hình vịtựquay (chúýrằngJM =DM, IN =DN), ta có T M

T N = JM

IN = DM

DN

Dođó tứgiácDN T M điều hòa vàT làđiểm cố định Ta cóđpcm

Bài khơng khó hình vẽhơi rối nên gây trởngại nhiều Bài tốn làm nhớ đến tốn hình học thi thửVMO 2018 năm trước với nội dung nhưsau:

Bài toán Cho tam giácABCnhọn nội tiếpđường trịn(O)cóđiểmDdiđộng cung nhỏ BC và không trùng với cácđỉnhB, C.Gọi I, J lần lượt tâmđường tròn bàng tiếp ứng với

đỉnhAcủa tam giácABD, ACD.Chứng minh khiDthayđổi thì

1 Tâmđường trịn ngoại tiếp tam giácDIJ ln thuộc mộtđường trịn cố định.

2. Đường trịn ngoại tiếp tam giácDIJ lnđi qua mộtđiểm cố định.

(175)

GọiT tâm của(DIJ)thìT, M, P thẳng hàng (cùng thuộc trung trực củaID) hayT ∈M P

Tương tựta cóT ∈N Q.Mặt khác,

∠N M P =

, ⌢

ACD+BD⌢ -=

ACB,

tương tựthì∠M N P = 12ABC⌢ Điều chứng tỏtam giácT M N có kích thước cố định (vì

có hai góc cạnh kềcố định) nênT Olà trung tuyến cố định VậyT thuộcđường tròn tâm O với bán kính khơngđổi

2) Gọi R, S trung điểm cung lớn AC, AB Dễ thấy I ∈ DS, J ∈ DR SA=SI, RA=RJ (tính chất tâmđường trịn bàng tiếp)

GọiK giaođiểm khácDcủa (DIJ)và (O).Ta có∠KSD = ∠KRD,∠KID = ∠KJD

nên∆KRJ ∼∆KSI(g.g)

Do đó, KR KS =

RJ SI =

RA

SA, điều cho thấy tứ giácARKS làđiều hòa, suy raK cố định (vì

R, S cố định) Vậy(DIJ)lnđi quađiểmK cố định

ChúýrằngK tiếpđiểm củađường trịn A – Mixtilinear ngoại tam giácABC

đối với(O).Ngoài ra, ta cịn chứng minhđược rằng:

• Đường trịnđường kínhIJ lnđi qua tâm bàng tiếp gócAcủa tam giácABC

• TrungđiểmIJ ln thuộc cung chứa góc cố định dựng trênRS

Đây toán thú vị, người thửsức Bài toán cũngđúng thay tâm bàng tiếp tâm nội tiếp tam giácABD, ACD

(176)

Trởlại tốn banđầu, ta có thểchứng minhđiểm cố định tiếpđiểm Mixtilinear nội tam giácDBCvới(O)(chẳng hạn dùng nghịchđảođối xứng, dođềVMO đềcậpđến chủ đềMixtilinear nên ta khơng khai thác khía cạnh nhiều)

Bài toán banđầu sựLẮP GHÉP tốn gốc vàđưa vào mơ hìnhđường trịn(ω)đểgây rắc

rối Thực phương pháp không làm cho tốnđẹp hơn, lại làm cho thích hợp với kiểuđềVMO (ít vẽ đường phụ, chỉnhìn khai thác mơ hình)

Qua tốn trên, ta thu bổ đề thú vị Cho tam giác nhọn khơng cân ABC nội

tiếp(O)và mộtđường trịn(ω)thayđổiđi quaA, Ocắt(O), AB, ACởD, E, F Khiđó ta có ED=EC, F D =F C.Ta có thểkhai thác bổ đề đóđể“chếbiến” thành toán sau:

Bài toán Cho tam giác nhọn khơng cânABCnội tiếp(O)và mộtđường trịn(ω)thayđổiđi

quaA, O cắt(O), AB, AC D, E, F Tiếp tuyếnởD của (O)cắtBC Lvà trung trựcDL

cắtEF K.Chứng minh rằngKDtiếp xúc(ω)

Bài tốn Cho tam giác nhọn khơng cânABCnội tiếp(O)có trực tâmHM là trungđiểm AH.Gọi(ω)làđường trònđi quaA, M, tiếp xúc vớiAOA. Đường trịn(ω)cắtđường trịn

đường kínhAH,cắtAB, ACT, R, S.LấyD∈AB, E ∈ACsao choRT =RD, ST =SE

Chứng minh rằngDE ∥AO

1.4 V

bài toán b

t

đẳ

ng th

c trong

đề

C

n Th

ơ

Bài toán Choa, b, c≥0thỏa mãnab+bc+ca= 1.Tìm giá trịlớn của

P =

a2+ 1 +

1

b2+ 1 +

1

c2+ 1

Câu có hai hướng tiếp cận

• Thay1→ab+bc+ca, quyđồng mẫuđưa vềP = (a+2(b)(ab++bc+)(cc)+a) dùngđánh giá 8(a+b+c)(ab+bc+ca)≤9(a+b)(b+c)(c+a)

• Lượng giác hóa,đặt(a, b, c)→'tanA

2,tan

B

2,tan

C

2 (

rồi có

P =

2(cosA+ cosB + cosC+ 3)

Nếuđề đổi yêu cầu thànhminP sao? Khiđó, chắnđẳng thức khơng xảy “tâm”

nữa mà phải “biên” Ta phải dự đốn dấu xảy có sốbằng0và hai sốbằng 1, vàđưa vềP ≥ 2.Như có cần phải dồn biến? Thực chưa cần, ta chứng minhđiều

khơng khó

P ≥2⇔(a+b+c)(ab+bc+ca)≥(a+b)(b+c)(c+a)⇔abc≥0

Như miền giá trị biểu thức P hẹp, ≤ P ≤

4.Bài toán biến đổi thuận lợi

vì giảthiếtab+bc+ca = tạo điều kiện cho phép quyđồng mẫu Ta thửthay thành

(177)

Bài toán Choa, b, c≥0thỏa mãnab+bc+ca= 3 Tìm giá trịnhỏnhất của

P =

1 +a2 +

1 +b2 +

1 +c2

Lời giải. Không tính tổng quát, giảsửclà sốnhỏnhất, suy ra3ab≥ab+bc+ca = 3⇒

ab ≥1.Ta chứng minhđược

1+a2 + 1+1b2 ≥ 1+2ab tươngđương(ab−1)(a−b)

2

≥ 0.Ta

sẽchứng minh

2 +ab+

1 +c2 ≥

3

⇔2(2 + 2c2+ +ab)≥3(1 +ab+c2+abc2)

⇔c2+ ≥ab+ 3abc2

⇔c2+ab+bc+ca≥ab+ 3abc2

⇔c2+bc+ca≥3abc2

⇔c(a+b+c−3abc)≥0

Mặt khác, ta có(a+b+c)(ab+bc+ca)≥3√3

abc·3√3a2b2c2 = 9abcnêna+b+c≥3abc.

Dođó, bấtđẳng thức cuốiởtrên làđúng ta cóP ≥

2 Đẳng thức xảy khia =b =c =

hoặca=b =√3, c = 0và hoán vị

Bài toán cho thấy dấu tâm biên toán gốcđã rơi vào thờiđiểm giá trịnhỏnhất (chúýrằng biểu thức khơng có giá trịlớn nhất)

Những tốn có “hai dấu bằng” thường tốn khó vàđẹp Dưới làđềchọn

đội tuyển Quốc gia Vĩnh Long 2018:

Bài toán Choa, b, ccó tổng bằng0, tìm giá trịnhỏnhất của

P = a−1

a2+ 8 + b−1

b2+ 8 + c−1

c2+ 8

Lời giải. Dùngđánh giá ghép cặp hai ba phân số Dự đoánP ≥ −38 nênđưa chứng

minh

a−1

a2+ 8 + b−1

b2+ 8 + c−1

c2 + 8 ≥ −

3

hay

(a+ 2)2

a2+ 8 +

(b+ 2)2

b2+ 8 +

(c+ 2)2

c2+ 8 ≥

3

Giảsửab≥0thì ta có (a+ 2)2

a2+ 8 +

(b+ 2)2

b2+ 8 ≥

(a+b+ 4)2

a2+b2 + 16 ≥

(a+b+ 4)2 (a+b)2+ 16 =

(c−4)2

c2+ 16

Cuối ta chỉcần chứng minh

(c−4)2

c2+ 16 +

(c+ 2)2

c2+ 8 ≥

3 ⇔

c2(c−4)2

(c2+ 8)(c2+ 16) ≥0

(178)

Sauđây sốbài toán tương tự

Bài toán Choa, b, c >0thỏa mãnab+bc+ca= 1 Tìm GTLN củaP = a21+1+b21+1+c12+1,5 .

Gợiý. Sử dụng lượng giác (a, b, c) → (tanA2,tanB2,tanC2) đưa 2P = cosA+ cosB − 3sin2C2 + =−3'sinC2 −

3 (2

+ 163 ⇒P ≤

Bài toán Choa, b, c >0thỏa mãnab+bc+ca= 3.Tìm GTLN củaP = a21+2+b21+2+c21+2.

Gợiý. Ta cần chứng minhP ≤ 1hay a2a+22 +

b2

b2+2 +

c2

c2+2 ≥ Tuy nhiên, theo bấtđẳng thức

Cauchy-Schwarz thìV T ≥ a2(+a+b2b++cc2)+62 =

a2+b2+c2+2(ab+bc+ca)

a2+b2+c2+6 =

Cuối cùng, xét tốn hay khó sau trongđềBrazil Revenge 2013, mởrộng bốn biến toán mở đầu Tác giảcủa lời giảiđẹp bên thầy Võ Quốc Bá Cẩn

Bài toán Choa, b, c, d ≥ 0thỏa mãnab+bc+ca+ad+ac+bd = 6.Chứng minh rằng

3>

a2+1 + b21+1 +c21+1 +d21+1 ≥2

Ở trường hợp tìm giá trị lớn nhất, ta cho a = b = c = n1, d = 2n với n → +∞ có

1

a2+1 + b21+1 + c21+1 + d21+1 → 3.Ở đây, ta phân tích trường hợp tìm giá trịnhỏnhất Xét

trường hợp sau

1 Nếuac < 1thìbd < 1, ta có1 +c2 <1 +

a2 = 1+a

a2 và1 +d2 <1 + b12 = 1+b

b2 nên

P > %

1 +a2 +

a2

1 +a2 &

+

%

1 +b2 +

b2

1 +b2 &

=

2 Nếuac≥1, ta có nhận xét sau:

1+a2 +1+1c2 ≥ 1+2ac khai triển rút gọn, ta có(a−c)

2

(ac−1)≥0,đúng

• a+b+c+d≥4vì theo khai triển bình phương tổng bấtđẳng thức AM-GM,

ta có:(a+b+c+d)2 = a2+b2+c2+d2+ 2(ab+bc+cd+da+ac+bd)và

2(ab+bc+cd+da+ac+bd)≤3(a2+b2+c2+d2)nên suy raa+b+c+d≥4.

Từ đó, theo bấtđẳng thức Cauchy-Schwarz

P ≥

1 +ac+

1 +b2+

1 +d2 =

1 +ac+

1 +ac+

1 +b2+

1 +d2 ≥

9

3 + 2ac+b2+

1 +d2

Chúýrằng

2ac+b2 = (b−a)(b−c) +ab+bc+ca≤ab+bc+ca= 6−d(a+b+c)≤6−d(4−d)

DođóP ≥

9−4d+d2 +1+1d2 Xét hiệu

P −2 =

9−4d+d2 +

1

1 +d2 −2 =

2d(2−d)(d−1)2

(9−4d+d2)(1 +d2) ≥0,

đúng vì0≤d≤2

(179)

1.5 V

bài tốn t

h

p trong

đề

tr

ườ

ng

Đ

ơng mi

n Nam

Bài tốn Tìm sốhốn vị(a1, a2, , a2018)của2018sốnguyên dươngđầu tiên sau cho

i) ai+1−ai ≤1với mọii= 1,2, ,2017.

ii) Tồn tạiđúng hai chỉsốiai =i?

Ýi) toán “kinhđiển” tài liệu đếm truy hồi;ýii) thêm vào dù không mới, hay việc khai thác sựphân bốcác sốtrong hoán vị Ta có hai hướng tiếp cậnđể đếm sốhốn vị ĐẸP thỏa mãn i), gọisn số hoán vịcủan sốnguyên

dươngđầu tiên choai+1−ai ≤1vớii= 1,2,3, , n−1

Cách 1. Gọiklà vịtrí thỏa mãnak =n Ta cóak−1 ≥ak−1 = n−1nên phải cóak−1 =n−1,

cứ nhưthế, ta suy raa1 = n−k+ Còn lại sốphía sauak là1,2,3, , n−k tạo thành

hốn vị đẹp nên trường hợpak =nthì sốhốn vịthỏa mãn làsn−k Chúýrằng nếuk =n

thì chỉcó hốn vịthỏa mãn là1,2,3, , n Từ suy sốhốn vịthỏa mãn sn = +

n−1

k=1

sn−k =s1+s2+ +sn−1+

Chúýrằngs1 = 1, s2 = 2nên xửlýcơng thức truy hồi trên, ta cóđượcsn= 2n−1

Tiếp theo cách chỉdùng nguyên lýnhân

Cách 2. Đầu tiên, ta viết số1;số2sẽnằm trước sau số1nên có2cáchđặt; số3sẽnằm

phía trước sau số2 nên có2cáchđặt, Cứ thế, nếuđã xếpđược k−1số đầu

tiên, ta sẽxếp sốk vào hai vịtrí:đầu dãy sau sốk−1.Dođó, theo ngun lýnhân, ta

có2n−1 cách xếp.

Trởlại tốnđã cho, giảsửta cóai =ivàaj =jvới1≤i < j ≤2018 Ta có

aj ≤aj−1+ 1≤aj−2+ 2≤ .≤aj−(j−i)+ (j−i) = i+ (j−i) = j

Đẳng thức xảy nên tất cảcác số ởgiữa “bấtđộng”; thếnên i, j phải hai sốliên tiếp

Mặt khác,ai+1 ≤ai+ =i+ 1, không thểcóai+1 =i+ 1(do chỉcó chỉsốthỏa mãn

ii) nênai+1≤i, màai =inênai+1 ≤i−1 Tiếp theo,ai+2 ≤ai+1+ 1≤inênai+2 ≤i−1

Dođó, sốtừaj+1 đếna2018 nhận giá trịkhông vượt quái−1 Lập luận tương tự, sốtừ a1đếnai−1phải nhận giá trịkhông nhỏhơnj+ Dođó, haiđoạn hốn vịphía trướcaivà phía

sauaj phải cóđộdài Từ ta suy rai= 1009, j = 1010.Rõ ràng hốn vịphía

trước phía sauđều phải hoán vị đẹp vàđược xếpđộc lập với Vậy sốhốn vịcần tìm

s1008·s1008 = '

21007(2 = 22014

Chú ý cách tương tự, thay số 2018 → n số điểm bất động hốn vị

cùng tính chẵn lẻvớin

(180)

Bài tốn Tìm số hoán vị của a1, a2, a3, , an của số {1,2,3, , n} với n ≥ thỏa

mãnđồng thời:

i) ̸=ivới mọii= 1,2,3, , n.

ii) ai+1−ai ≤1với mọii= 1,2,3, , n−1

Trước hết, ta tính số hốn vị thỏa mãn điều kiện ii không thỏa mãn điều kiện

i, tức hốn vị có sối mà = i.Ta thấy tồn tạii ̸= j mà

ai =i, aj =j tương tựtrên, ta có:

• Với mọik mài≤k ≤jthìak =k

• Gọix, ylần lượt sốnhỏnhất, lớn thỏa mãnai =i,1≤i≤nthìx+y=n+

Với mỗixmà1≤x≤)n+1

*

, tương tựcách tính sốhốn vịthỏa mãnđiều kiện iiởtrên, ta thấy số hoán vịthỏa mãn là22(x−2) nếu x ≥ 2và bằng nếu x = 1. Khiđó, sốhốn vị thỏa mãn

điều kiện ii không thỏa mãnđiều kiện i

1 + [n+1

2 ]

0

i=2

22(x−2) = + [n+1

2 ]

0

i=2

4x−2 = + 4[ n+1

2 ]−1

3 =

4[n+12 ] +

3

Vậy sốhốn vịcần tìm

2n−1− 4[ n+1

2 ]−1+

3

Một sốbài toán tương tựvềkiểuđếm dãy sốhoặc hoán vị

Bài tốn (ẢRập 2015) Hỏi có dãy số1≤ a1 < a2 < < a12 ≤2018sao cho ≡i2(mod12)?

Hướng dẫn. Liệt kê cụthể sốdưcủa i2 khi chia cho12,ta có1,4,9,4,1,0,1,4,9,4,1,0.

Trừmỗi sốcho sốdưtrên chia cho12,tađưa vềviệcđếm sốdãy

0≤b1 ≤b2 ≤b3 < b4 < b5 < b6 ≤b7 ≤b8 ≤b9 < b10 < b11< b12 ≤168

Đếnđây, ta dùng song ánh chia kẹo Euler

Bài toán (TP.HCM 2018) Hỏi có hốn vị (a1, a2, , a164) của 164 số nguyên

dươngđầu tiên choai ̸=iai ≡i(mod41)với mọii= 1,2, ,164?

Hướng dẫn. Chia sốtừ 1,2, ,164thành41nhóm theo sốdưkhi chia cho41thì rõ ràng

mỗi nhóm có4số Theo giả thiết sốtrong bộsẽ hốnđổi vịtrí cho Ta

thấy với bộ(x1, x2, x3, x4), ta có9cách hốn vị“khơng cóđiểm cố định (có thểliệt kê

dùng cơng thức kinhđiển4!'2!1 − 3! +

1 4! (

= 9)

(181)

Bài tốn (dựa theođềtrườngĐơng miền Nam 2014) Có hốn vị(a1, a2, , a2018)

của(3,4,5, ,2020)sao choai ≥i?

Hướng dẫn. Trước hết, ta có a2018 ∈ {2018,2019,2020} nên có cách chọn Tương tự a2017 ∈{2017, ,2020}nhưng loạiđi sốdùng choa2018 nên chỉcịn3cách Cứnhưthế,

đếna3cịn3cách;a2 cịn2cách vàa1cịn1cách Tổng sốhốn vịlà2·32016

Bài tốn (bài tốn phát kẹo) Cơ giáo có10loại kẹo (mỗi loại có nhiều viên) cần phát cho 30học sinh lớp (một em nhận không quá 1viên/loại), giả sửrằng em có học lực

đơi khác Hỏi giáo có cách phát kẹo, biết học sinhAgiỏi hơn B thìB có kẹo làAcó kẹođó (tính cảtrường hợp khơng em nhậnđược kẹo)?

Hướng dẫn. Xếp30học sinh thành dãyA1, A2, , A30 theo thứtựtăng dần học

lực Xét loại kẹođầu tiên, emAk nhận tất cảcác emAk+1, Ak+2, , A30đều có;

nhưthếsốcách phát kẹo với sốcách chọn emđầu tiên tính từtrái quađược nhận kẹo, có tất cả31cách nhưthế Tương tựvới loại kẹo cịn lại, nên có3110cách phát

Bài tốn (Bài tốn nhện) Một nhện có8cái chân,8cặp vớ- giày khác (vớchỉ

dùng chung với giày tươngứng) Con nhện có thứ tự mang vớvà giàyđể sao cho chân, giày phảiđược mang vào sau vớ?

Hướng dẫn. Ta “tuyến tính hóa” cặp giày vớ,đặt là(a1, b1),(a2, b2), ,(a8, b8)

Hoán vịcủa16chữcái là16!, cặp chữcái, ta phải cóbiđứng trướcai (các

chữcái khác có xuất hiệnởgiữa chúng hay khơng khơngảnh hưởng) Dođó, ta khơng xét hốn vịcủa chữcái cặp nên chỉcòn 16!

28 cách mang giày vớcho nhện

1.6 V

bài tốn hình h

c trong

đề

tr

ườ

ng

Đ

ơng

Phú Yên

Bài toán Cho tam giác nhọnABC có trực tâmH vàđường caoBE, CF GọiM, N, P, Q

là trungđiểmBE, CF, AC, AB.GiảsửM N cắtAB, AC L, K.GọiJ là tâm của(ALK)

1 Chứng minh rằngJ, P, Qthẳng hàng.

2 Chứng minh rằng(HM N)tiếp xúc với(ALK)

Lời giải. Bài toán mộtứng dụngđẹp cho mơ hình tứ giác tồn phần, kết hợp hàng loạt hàngđiểmđiều hịa

1) GiảsửEF cắtBCởT có tứgiác toàn phầnBCEF.AT vớiđường thẳng GaussM N nên M N qua trungđiểmX củaAT.Đường caoBE, CF cắtAT ởS, R Ta có hàngđiểmđiều

hòa quen thuộc(CF, HR) =−1nên theo hệthức Maclaurin

(182)

Tương tựthìHE·HB =HM·HSnênHN·HR=HM·HS DođóM N SRlà tứgiác nội

tiếp VìM Q∥P K, QL∥P N nên XQ XP =

XM XK =

XL XN,

suy

XM ·XN =XL·XK =XR·XS =XA2

nênXAtiếp xúc với(ALK).KẻXDtiếp xúc với(J)thìXA=XD=XT gócADT vng

Ngồi tứgiácALDK điều hòa nênA(AD, LK) =−1, suy A(T D, BC) = −1dẫnđến

Dchính chânđường caođỉnhA.DođóJA=JD,điều cho thấyJthuộc trung trựcAD,

chính làP Q

2) Dễthấy5 điểmH, M, N, D, I thuộcđường tròn đường kínhHI (vớiI trung điểm BC) Dođó:(HM N),(ALK)cùngđi quaD.Theo câu thìXD2 =XL·XK =XM·XN

nênXDlà tiếp tuyến chung của2đường trịn

Một tốn tương tự có xuất trườngĐơng 2015:

Bài tốn Cho tam giác nhọn ABC khơng cân nội tiếpđường trịnω.Đường trịnω′thayđổi

đi qua B, C cắt cạnhAB, AC lần lượt tạiE, F.Đường tròn ngoại tiếp tam giác AEF cắt

lạiđường tròntạiK. KE, KF lần lượt cắt lạiđường tròntạiQ, P GọiT là giaođiểm

củaBQCP GọiM, N lần lượt trungđiểmBF, CE

1 Chứng minh rằngT thuộc mộtđường thẳng cố định khiđường trònω′thayđổi.

2 Chứng minh rằngKAlà tiếp tuyến củađường tròn ngoại tiếp tam giácAM N

(183)

Nếu tổng quát trườngĐông Phú Yên, thayEF thànhđối song bất kỳ, yếu tốsau sẽvẫn

đúng:XAtiếp xúc(AM N)và(HM N),(AKL)tiếp xúc

Đếnđây, ta lại nhớ đến toán quen thuộc trongđềIMO Shortlist 2009:

Bài tốn Cho tứ giácABCD khơng phải hình thang, nội tiếp trongđường trịn (O)

AC∩BE =EAD∩BC =F.GọiM, N lần lượt trungđiểm củaAB, CD.Chứng minh

rằngđường thẳngEF tiếp xúc vớiđường trịn(EM N)

Thếthì tốn với vừa phát biểuởtrên có liên hệgì?

Nếu tinh ý, ta đổi thứ tự điểm, từ tứgiác toàn phầnBCEF.AT, tađược tứgiác “lõm” BECF.T A Nhưthế, toán IMO Shortlist phát biểu thànhT Atiếp xúc với(AM N)

Vậy hóa tốn tổng qtởtrên cách nhìn khác củađềIMO Shortlist

Chúýrằng bổ đềnày cách khácđểchứng minh dùngđẳng giác tam giác

đồng dạng nghịch Nhưthế, quay lại mơ hình trên, ta phát mộtđiều thú vịlà:AM, AN

đẳng giác gócA; cịnDM, DN đẳng giác gócDcủa tứgiácAKDL.Nhưthế, có

thể đặt thêm câu hỏi như: Chứng minh phân giác góc ∠M AN ln qua

điểm cố định Ngồi ra, ta cịn cách tiếp cận nhẹnhàng cho tốn là:

Lời giải khác. Vì tam giác ABE, ACF đồng dạng nghịch nên AM, AN đẳng giác A, kéo theo haiđường tròn(AM N),(AKL)tiếp xúc ởA(tính chất quen thuộc xuất

hiện VMO 2015) MàXK ·XL = XM ·XN nênX thuộc trục đẳng phương hai

(184)

Nói tóm lại, thay dùngđiều hịa, việc dùngđẳng giác giúp cho tốn tổng qtởtrên, đềtrườngĐơng Phú n, giải vài dịng!

Nóiđến bổ đềnày, ta có tốn tương tựcủa tác giảTrần Quang Hùng ôn luyện chođội IMO cáchđây vài năm:

Bài tốn Cho tam giácABCvà mộtđường trịn(I)thayđổi ln quaB, CcắtđoạnAB, AC

lần lượt tạiD, E.GiảsửCDcắtBEKM, N là trungđiểmBD, CE.Đường thẳngKM

cắt(KCE)RKN cắt(KBD)S Chứng minh rằngAI vuông góc vớiRS.

Cuối cùng, xin mời người thử sức với toán trongđềchọnđội tuyểnẢRập 2017, mộtứng dụng thú vịcủa mơ hình

Bài tốn Cho tứ giác ABCD khơng phải hình thang, nội tiếp (O) AB ∩ CD =

E, AD∩BC =F AC∩BD=K.GọiG, Hlà trungđiểmAB, CDIlà tâm của(GHK).

Giảsử(I)cắt(O)M, N sao cho tứgiácM GHN lồi.ĐặtP =M G∩HN, Q=M N∩GH

1 Chứng minh rằngIK, OE song song.

2 Chứng minh rằngP K, IQvng góc.

Nếu dùng cực –đối cực, toán nhẹnhàng nếuđã biết bổ đềtrong shortlist nêu tốn cồng kềnh “hiển nhiên”

1.7 V

bài toán max trong

đề

tr

ườ

ng

Đ

ông Phú Yên

Ta xuất phát tốn tìm max sauđây, xuất trongđềkiểm tra trườngĐơngở

Phú n

Bài tốn Với cặp số thựcα,β, đặtM(α,β) = max x∈[−1;1]

|x2+αx+β|

|x|+1 Tìm giá trịnhỏ nhất

củaM(α,β)khiα,β thayđổi.

Dướiđây làđáp án củađềthi trongđềPhú Yên

Lời giải. Đặtg(x) = x2+αx+βthì|g(x)|≤M(α,β) (|x|+ 1), thay sốg(−1), g(0), g(1),

ta có max{g(−1)−g(0), g(1)−g(0)} ≤ 3M(α,β).Ngồi ra, g(−1) = 1−α+β, g(0) = β, g(1) = +α+β nên3M(α,β)≥max{1−α,1 +α}≥1 Suy raM(α,β)≥

3

Để ýrằng muốn cóđượcM = 1/3thì phải có cácđẳng thức xảy cácđánh giá trướcđó,

tức

|1−α+β|=|1 +α+β|= 3,|β|=

1

và giải hệnàyđượcα = 0,β =−13

Cuối cùng, chỉcần chứng minh rằng11x2− 1 ≤

3(|x|+ 1)với mọix∈[−1; 1]là toán kết

(185)

Dạng toán max năm 2017 - 2018 vừa quađã xuất nhiều (trongđề

thi thửTHPT QG) Quảthật lầnđầu gặp dạng ta tiếp cận theo cách nghĩ đơn giản là: tínhM(α,β)theoα,β tìm hàm hai biến miền[a;b]thuđược chắn

rơi vào bếtắc chia trường hợp Có hướng xửlýbài toán dạng Cách (theo hướngđại số)

• Thay sốthích hợpđể đánh giáđượcM ≥max{f(a), f(b), f(x0)}

• Tìm cách phối hợp thích hợp giá trịf đểtriệt tiêuđược tham tham số

2 Cách (theo hướng giải tích, dùng đồthị) Dựa trêný tưởng là: “trên đoạn AB, muốn

chọn mộtđiểm C cho khoảng cáchmax{AC, BC}là nhỏnhất” rõ ràng,C phải

trungđiểmAB.Điều hiển nhiên!

Ta xét toán sauđểthấy rõ nội dung cách 1:

Bài toán Vớia∈R,đặtM(a) = max

[−1;2] |−9x

2 + 4x+a|, tìmađểM(a)đạt giá trịnhỏnhất.

Lời giải. Đặty = −9x2 + 4x thì vớix ∈ [−1; 2], ta có y ∈ [−28;

9] Tađưa tốn tìm

GTNN củaM(a) = max [−28,49] |

y+a|.Ở hàmf cần xét làf(y) =y+a

VìM(a)≥ |f(−28)|,11f'49(11nên2M ≥ 11−28 +a−a−

1 ⇒M ≥ 128

9 KhiM = 128

9 ,

đánh giáởtrên phải xảy dấu bằng, tức

⎧ ⎪ ⎪ ⎨ ⎪ ⎪ ⎩

f(−28) = f % & = 128 (28−a)

%

4 +a

&

≥0

→a= 124

Cuối cùng, ta chỉcần chứng minh khia = 124

9 maxđúng 128

9 Bước có thểxửlýnhẹ

nhàng khảo sát hàm số

Nhưthế, trường hợp chỉcó tham sốvà biến với tham số“độc lập”, toánđược giải trọn vẹn theo cách Dướiđây sốdạng tương tự:

Bài tốn Tìm củamax|3x+m|trênD= [0; 2].Đáp sốlà:min max = 4khim =−5. Bài toán Tìm của

max

1 1 1x

2+ 2x−1 x+ +m

1 1

trênD= [0; 2].Đáp số:min max = 53 khim= 23

Câu hỏiđặt haiđầu mút giá trịthứba làx0 cần chọn giá trịnào, có phải ln

làx0 = a+2b? Thật khóđểtrảlời tổng quát chỉdùng cách thứ1, dướiđây ta sẽtiếp cận

(186)

Đồthịnày lõm nên miền[a;b], hàm sốsẽcóđúng cực trị,đồng thời cóđúng

tiếp tuyến song song vớiđoạn nối haiđầu mút

Gọi tên hai đầu mút A, B Muốn tìm đường thẳng (d)nào mặt phẳng cho

khoảng cách xa từcácđiểm phầnđường cong giới hạn giữaA, B chođến(d)là nhỏ

nhất, ta sẽ:

• Tìm vịtrịDxađoạnABnhất trênđường cong

• Chọn tiếp tuyến(d)của(C)tạiDsao cho tiếp tuyếnđó song songAB

• Đường thẳng cần tìm sẽnằm giữaAB và(d)và cáchđều haiđườngđó Đây “tinh thần” cách thứ2 Ta xét ví dụsauđểthấy rõ hơn:

Bài tốn Vớia, b∈R,đặtM(a, b) = max

[1;2] 11

x −(ax+b)

1

1, tìm GTNN củaM(a, b).

Lời giải. Đặtf(x) =

x,đặtA(1; 1)vàB(2;

1

2)là haiđầu mút phầnđồthịcủa

x miền xét Phương trìnhAB :y =−12x+ 32

Xétx0 ∈[1; 2]thìk =f′(x0) =−x12

0 hệsốgóc tiếp tuyến tạiđiểm có hồnhđộx0 Tiếp

tuyến song song vớiAB

x2

=−1

2 →x0 = √

2

Ta viếtđược phương trình tiếp tuyến là(d) :y=−1 2x+

Phương trình cần tìm song song cách haiđường thẳngAB,(d)nên có phương trình

y=−1 2x+

1

'3 +

2(và khoảng cách khiđó

1√2−3 1=

4−

2

2 Dođó, khia=− 2, b=

4 +

2

2 thìminM(a, b) = −

2

Dướiđây sốdạng tương tự:

Bài toán Vớia, b∈R,đặtM(a, b) = max

[−1;4] |x

2−(ax+b)|, tìm GTNN củaM(a, b).Đáp số

là:a= 3, b =

8 min = 25

(187)

Bài toán Vớia, b ∈ R, đặtM(a, b) = max

[0;1]

1x2+2x+1x−1 −(ax+b)11, tìm GTNN của M(a, b)

Đáp sốlà:a = 2, b= −

2, =√2−3 2.

Ở dạng chuẩn mực, cuối xin giới thiệu toán đềHướng tới VN TST năm 2011:

Bài tốn Tìm sốmnhỏnhất cho từba sốthực bất kỳthuộcđoạn[0; 1]ln tìmđược hai

sốx, ysao cho

0≤xy(x−y)≤m

Lời giải. ĐặtđoạnE = [0,1].Bài tốn tươngđương với việc tìm m= max

(a,b,c)∈E3

6

min

{x,y}⊂{a,b,c}{|xy(x−y)|}

7

Xét bộ(a, b, c).Khơng tính tổng qt, có thểgiảsửa≤b≤c.Khiđó rõ ràng ab(b−a)≤ac(c−a)vàbc(c−b)≤ac(c−a)

Dođó

min

{x,y}⊂{a,b,c}{|xy(x−y)|}= min{ab(b−a), bc(c−b)}

Mặt khác, dobc(c−b)≤b(1−b)vab(b−a)≤ b3

4 nên

min{ab(b−a), bc(c−b)}≤min

6 b3

4, b(1−b)

7

Dođó

m= max b∈E

6

min

6 b3

4, b(1−b)

77

Giải phương trình b3

4 =b(1−b)vớib ∈E, tađượcb1 =−2 +

8 Suy

6 b3

4, b(1−b)

7 = ⎧ ⎨ ⎩ b3

4 b≤b1

b(1−b) b≥b1

Từ dễdàng tìmđượcm = 10√2−14

1.8 V

bài t

h

p trong

đề

Ninh Bình

Bài tốn Viện tốn cao cấp VIASM tổchức6buổi chuyênđềchonsinh viên mà buổi có

đúng100sinh viên tham gia Biết khơng có2sinh viên mà hợp lại tham giađủ6buổi.

Tìm giá trịnhỏnhất củan

(188)

1 Nếu có sinh viên học5buổi, buổi cịn lại có100người học nên tồn hai người họcđủ6buổi, mâu thuẫn

2 Nếu có sinh viên học4buổi, tập hợp sinh viên học 2buổi cịn lại phải khơng

giao Dođó, sốsinh viên tối thiếu là201người

3 Ta sẽchứng minh sốsinh viên tối thiếu là200người, người học buổi Thật

vậy, sốcách chọn tập hợp buổiđểhọc làC3

6 = 20 Tuy nhiên, tổhợp buổi có

tổhợpđối (vì khơng có hai sinh viên hợp lạiđủcả6mơn) nên sốtổhợp có thểlựa chọn

thực tế là10tổ hợp Ta chia200sinh viên thành10nhóm20người, nhóm học1tổ

hợp,được sốbuổi là10×20×3 = 600, thỏa mãn u cầuđềbài

4 Nếu sốsinh viên hơn200,sẽcó tối thiểu người học4buổi, theo lập luậnởtrên

thì sốsinh viên là201,mâu thuẫn

Vậy giá trịnhỏnhất cần tìm là200

Câu hỏi dạng thường phổ biến toánđếm hai cách Bên xin phân tích thêm vài nội dung liên quanđến toán Xuất phát từbài IMC 2002 (khá quen thuộc):

Bài tốn Lớp học có200 học sinh , tham gia giải6 bài tốn mà có nhất120 hs

giảiđược Chứng minh có2học sinh có thể"hợp tác" với giải hết6bài.

Lời giải. Phản chứng, cặp2học sinhđều khơng giảiđược nhất1bài Tađếm sốbộ

S = # ({A, B}, C)với cặp học sinhA, B khơng giảiđược bàiC

• chọn cặpA, B,cóC2

200, chọn bàiC,có là1nênS ≥C2002

• chọn bàiC,có6cách, chọn cặp học sinh khơng giảiđược này, có khơng qC2 80

nênS≤6·C2 80

Suy raC2

200≤6·C802 , vô lý

Năm 2015, Việt Nam TST có tương tự, khơng giải theo cách nhưtrênđược!

Bài tốn Trong kỳthi vấnđáp, có100thí sinh và25giám khảo, thí sinhđược hỏi ít

nhất10giám khảo Chứng minh có7giám khảo có thể"hợp tác" với hỏi hết100thí

sinh trên.

Thực so với tương tự, theo nghĩa:

• 200học sinh và25giám khảo

• 6bài tốn và100thí sinh

(189)

• học sinh hợp tác và7giám khảo hợp tác

Nếu thực theo cách tương tựthì bấtđẳng thức cuối lại khơng bịmâu thuẫn (nếuđề đổi số

từ25thành24thì lạiđược) Dođó, cần có cách tiếp cận chặt May mắn làđiều

khơng khó, chí cịn tựnhiên

Ta giải lại bài2theo cách khác nhưsau, người có thểtựgiải bài3theo cách

Cách khác. Chọn hs giỏi Sốcặp (học sinh ,bài toán) mà học sinh giảiđược tốn là6·120 = 720; mà)720200*+ = 4nên học sinh giỏi phải giảiđược là4

bài (theo Dirichlet)

Loại học sinh với4bài bạn giảiđược; còn119học sinh và2bài Sốcặp lúc

ít là2·119 = 238,mà)238 199 *

+ = 2nên sẽgiảiđược cả2bài Dođó hai hs giỏi

nhì lớpởtrên sẽgiảiđược cả6bài

Bài tốnđềNinh Bình cho ta tình huốngđặc biệt: "tìm cực trịcủa sốhọc sinh", tức phải dấu xảy Bài toánđã chọn số lý tưởng, thuận lợi cho việc xây dựngđó Và lời giải bên thểhiệnđượcýtưởng cáchđã nêuởtrên

Cuối cùng, ta xem thử đềIMC 2013, dữkiện giống với toán banđầu

Bài tốn Trong trường có2nhọc sinh,n≥2 Mỗi tuần cónhọc sinh tham gia dã ngoại Ta

thấy rằng:2học sinh tù có tham gia chung nhất1chuyếnđi Hỏi có bao nhiêu

chuyếnđi?

Đáp sốbài toán là6

Nhận xét toán dạng có "hơi hướng"đếm hai cách Tuy nhiên, sốcho rơi vào dạngđặc biệt lớn nên việcđánh giá thơng thường sẽgặp nhiều khó khăn, chí bịngược dấu Lúcđó, ta cần phải "qn bớt" kỹthuật mạnh mà suy luận nhẹnhàng (như ởtrên dùng Dirichlet + cực hạn tựnhiên) giải quyếtđược Thửgiải hai toán tương tựsau

Bài toán Câu lạc bộcónthành viên, tham gia12buổi chuyênđềmà buổi có24thành

viên Biết rằng2thành viên bất kỳ tham gia chung khơng q buổi Tìm giá trịnhỏ nhất

củan.Câu hỏi tương tựnếu thay(12,24)bởi cặp(10,7)

Gợiý. Gọia1, a2, , anlà sốbuổi mà thành viên thứ1,2, , ntham gia Ta có haiước lượng

8

a1+a2 +· · ·+an= 288,

a21+a22 +· · ·+a2n≤420

Dùngđánh giáa2

i ≥ 3ai −2, ta có đượcn ≥ 222.Ýsau thực tương tự đánh giá

a2

(190)

1.9 V

bài s

h

c trong

đề

TPHCM

Bài toán Tồn hay khơng hốn vị (a1, a2, , a164) ∈ S, thỏa mãn với mỗi i ∈

{1, 2, , 164}luôn tồn tạibi ∈{0, 1, , 40}sao cho

a1+a2+ .+ai ≡b2i (mod 41)?

Dướiđây hai cách tiếp cận theo hướng dùng kỹthuật mạnh hoặcđi xây dựng (lời giải tham khảo tác giảNguyễn Song Minh Nguyễn Ngọc Trung diễnđàn Mathscope.org): Cách 1. Đầu tiên xây dựng dãy:

{0,1,40,2,39,4,37,5,36,8,33,9,32,10,31,16,25,18,23,20,21}

Với20sốcòn lại, ta chia thành10cặp mà tổng2sốcủa1cặp chia hết cho41.Với cặp(a, b)

màa < b, có thểkiểm trađược tồn

i∈{1,2,4,5,8,9,10,16,18,20}, j ∈{21,23,25,31,32,33,36,37,39,40}

màb+i≡j (mod 41) Chèn cặp số(b, a)vào sauilà xong

Cách 2. Giảsửrklà sốdưcủa6kkhi chia41, do6là nguyên thủy mod41và 62+ 6≡1 (mod 41), 620+ 1≡0 (mod 41)

Nên ta chia40sốnguyên dươngđầu tiên làm10bộ, bộ4số ởdạng (r2k, r2k−1, r20+2k−1, r20+2k), ∀k = 1,2, ,10

Cònai =inếu41|i, vàa41t+r = 41t+ar vớit = 0, 1, 2,3và1≤r ≤40 Để ýrằng, với mỗi0≤i≤9, 0≤t≤3sẽtồn tạik∈Z+thỏa

(a41t+4i+1, a41t+4i+2, a41t+4i+3, a41t+4i+4)

= (41t+r2k, 41t+r2k−1, 41t+r20+2k−1, 41t+r20+2k)

Vì thếnên ta có

a41t+4i+1+a41t+4i+2+a41t+4i+3+a41t+4i+4 ≡r2k+r2k−1 +r20+2k−1+r20+2k ≡62k+ 62k−1+ 620+2k−1+ 620+2k

≡'620+ 1( '62k+ 62k−1(≡0 (mod 41).

Kết hợp41|ai nếui|41, nên nếumchia41dưnthì có

Sm =a1+a2 + .+am ≡a1+a2+ .+an (mod 41)

(191)

• Nếur = 0, thìSm ≡0 (mod 41)

• Nếur = 1, tồn tạii, k ∈Z+thỏa mãn Sm ≡a4i+1 ≡r2k≡

'

6k(2 ≡(rk)2 (mod41)

• Nếur = 2, tồn tạii, k ∈Z+thỏa mãn Sm ≡a4i+1+a4i+2 ≡r2k+r2k−1

≡62k+ 62k−1 ≡62k−2'62+ 6(≡(rk−1)2 ( mod 41)

• Nếur = 3, tồn tạii, k ∈Z+thỏa mãn Sm ≡a4i+1+a4i+2+a4i+3

≡r2k+r2k−1+r20+2k−1 ≡62k+ 62k−1+ 620+2k−1

≡62k+ 62k−1'620+ 1(≡(rk)2 (mod41)

Tóm lại tồn hốn vịnhưu cầu

Tiếp theo, ta sẽnêu cách tiếp cận nhẹnhàng hơn, làýmà tác giả"chếbiến" Cách 3. Dùng bổ đề: sốnguyên tố p = 3k+ 2sẽ có tính chất{13,23, , p3}là hệthặng dư

đầyđủmodp Khiđó, ta chọnai ≡i3 (mod 41)là xong, vìđể ýrằng 13+ 23+· · ·+k3

ln sốchính phương

Chứng minh bổ đề: Giả sử có hai số i ̸= j cho i3 ≡ j3 (mod p), suy ra i3k≡j3k (mod p) Theođịnh lýFermat nhỏthìi3k+1 ≡j3k+1 (mod p)nên

i3k+1 ≡j3k+1 =j·j3k≡j ·i3k (mod p),

kéo theoi3k(i−j)chia hết chophayi≡j (mod p), vơ lývìi, j ∈{1,2, , p}vài̸=j.

Cách 4. Nhận xét

1 + + +· · ·+ 2k−1 =k2

nên cách hoán vịsauđây sẽthỏa mãn đềbài: Xếp số lẻtừ1đến 41, xếp sốchẵn từ đến40, tiếp tục xếp sốlẻtừ43đến81, cứnhưthế

Lời giải thứba cho thấy bất kỳsốnguyên tốp= 3k+ 2nào tồn hoán vịnhưthế Tuy

nhiên, lời giải cuối cho thấy, số nhưthế có nhiều Ta thử phân tíchề số thỏa mãn tồn hoán vịnhưthế, tạm gọi "số đẹp"

Theo cách giảiđó, rõ ràng bất kỳsốnguyên dương lẻ2m+ 1nào "số đẹp" Tiếp theo,

(192)

cho2m+ 1rồi xếpđến sốtiếp theo, cứthế Nhưthế, sốnguyên dương lũy

thừa của2đềuđẹp Vậy cịn lũy thừa của2thì sao?

Số2đẹp, xét hốn vị1,2 Sốn = 2k có tổng của tất cảnsốlà2k−1(2k−1), sốnày chiandư

là 2k−1 Như thếthì với k chẵn, khơng tồn tại hốn vịthỏa mãn đềbài Tuy nhiên, ta có nhận

xét nếuađẹp cácước củaacũngđẹp, số4khơngđẹp nên tất cảcác lũy thừa

khác của2cũng

Vậy nên tất cảcác số đẹp cần tìm là2cùng tất cảcác sốcóước ngun tốlẻ

Liên quanđến tính chất sốnguyên tốp= 3k+ 2, ta có tốn sau: Bài tốn Cho dãy số(an)thỏa mãn

8

a1 = 1, a2 = 2, a3 = 15, an+3 = 2a3n+2−an+1−a3n

Chứng minh tồn tại hai sốhạng liên tiếp dãy chia hết cho1019

Lời giải. Để ýrằng dãy sốtuần hồn, vì1019là sốngun tốcó dạng3k+2nên nếux3 ≡y3

(mod 1019)thì suy rax≡y (mod 1019) Dođó, dãy tuần hồn từsốhạngđầu tiên

Tính raa0 =a−1 = 0thì tốnđược giải

Liên quanđến hoán vịthỏa mãnđiều kiện cho trước, ta có tốn sau trongđềthi APMO:

Bài tốn Tìm tất cả các số nguyên dương n sao cho tồn hoán vị {a1, a2, , an} của

{1,2, , n}sao cho{a1+ 1, a2+ 2, , an+n}{a1−1, a2−2, , an−n}cũng hệ

thặng dư đầyđủtheo modulon

Lời giải. Trước hết, xét hiệu

0≡ n

0

i=1 ai−

n

0

i=1 i≡

n

0

i=1

(ai−i)≡ n

0

i=1

i= n(n+ 1)

2 (mod n)

nênnlẻ Tiếp theo, ta có

4n(n+ 1)(2n+ 1) =

9 n

i=1

(a2i +i2)

:

≡ 2n(n+ 1)(2n+ 1)

6 (mod n)

nênnkhơng chia hết cho3.Dođó(n,6) = 1.Ta chỉcần xétai ≡2i (mod n)là thỏa mãn

2.

Đề

thi th

VMO và

đ

áp án chi ti

ế

t

Nhằm chuẩn bịcho kỳthi, tác giảbài viết có biên soạn haiđềthi thửtheođúng cấu trúcđềchọn HSG quốc gia năm gần với lời giải chi tiết, nhận xét, mởrộng Các tốn có tham khảo thêm tác giả:thầy Vũ Đình Hịa (đề2, 2a), thầyĐàm Văn Nhĩ(đề2, bài 1b), thầy Nguyễn Minh Hà (đề2, 6b), thầy Lê Bá Khánh Trình (đề 2, 4b), thầy Dương Châu Dinh (đề1, 5b), thầy VũThếKhôi (đề2, 5), thầy Nguyễn Văn Linh (đề1, 6b), số đềthi thật khác Các lại tác giảtựbiên soạn thêmđểcânđối tính khó-dễ

(193)

2.1.

Đề

s

1

2.1.1.

Đề

thi

Bài toán Cho dãy số (an),(bn) xác định bởi a0, b0 ∈ R và với mỗi n ≥ 0, các số hạng an+1, bn+1 được xácđịnh theoan, bnbởi hai cách

i) an+1 = 20182019an, bn+1 = 1− 2019an hoặc

ii) an+1 =a2n, bn+1 =an.

1 Chứng minh nếua0 ∈(−1; 1)thì dãy số(an)có giới hạn hữu hạn, tìm giới hạnđó.

2 Giảsửa2018 ≤a0,tìm giá trịlớn tổngS =b1+b2+· · ·+b2018

Bài tốn Cho tam giác ABC nhọn, khơng cân, nội tiếp trongđường trịn (O).Mộtđường

trịn (O′)thayđổi, lnđi quaB, C và cắt cạnhAB, AC theo thứ tự ởD, E.GọiD′, E′

lần lượt cácđiểmđối xứng vớiD, Equa trungđiểm cạnhAB, AC

1 Chứng minh trungđiểmD′E′ luôn thuộc mộtđường thẳng cố định.

2 Trên cung nhỏvà cung lớnBCcủa(O), lấy cácđiểmR, Ssao cho(DER),(DES)

tiếp xúc với(O).Phân giác gócBRC, BSC cắt nhau K.Chứng

minh rằngđường trịn(DEK)ln tiếp xúc vớiđường thẳngBC

Bài toán Đa thức f(x) hệ số thực được gọi “đẹp” có thể biểu diễn thànhf(x) = (P(x))3−(Q(x))2, trongđóP(x), Q(x)là cácđa thức hệsốthực.

1 Chứng minh rằngf(x) = 2018x2−2019làđa thứcđẹp.

2 Hỏi có tồn hay khơngđa thứcf(x)bậc làđa thứcđẹp?

Bài tốn Một bảng vng kích thước2019×2019được phủkín hình: chữL,vng

và chữZ nhưbên (có thểxoay hình tùnhưng khơngđượcđè lên nhau).

1 Hỏi có thểphủ bảng hay khơng khơng sửdụng hình vng2×2nào?

2 Chứng minh cách phủ, tađều cần dùng nhất4039hình chữL

Bài tốn Cho sốthựcx, y, z thỏa mãnđồng thời cácđiều kiện0< z ≤ y ≤ x ≤ 8 3x+ 4y ≥max;xy;12xyz−8z<

1 Chứng minh rằng

x +

6

y +

3

z ≥3

(194)

Bài tốn Cho tam giác ABC nhọn khơng cân nội tiếp đường trịn (O) M, N lần lượt

là trungđiểmAB, AC Dlà trungđiểm cung lớnBC của(O).GiảsửK là mộtđiểm nằm

trong tam giác thỏa mãn∠KAB = 2∠KBA, ∠KAC = 2∠KCA.

1 a) Chứng minh rằngKA=KD

2 Giả sửAD cắt BC T T M cắt(BM C) R, T N cắt (BN C) S GọiP là giao

điểm của KB OM, Q là giao điểm của KC ON Chứng minh trục đẳng

phương haiđường tròn(T QR),(T P S)đi quaO

Bài toán Với sốnguyêna, bnguyên tốcùng vàa > b >1, xét dãy sốsau

un=ϕ

'

a2n−1+b2n−1( vớin= 1,2,3,

trongđóϕ(x)là sốcác sốngun dương khơng vượt qxvà ngun tốcùng vớix

1 Chứng minh nếup > 3là sốnguyên tốlẻvà có sốhạng nàođó dãy bằng 2pthìa+b = 2p+ 1hoặca+b = 2(2p+ 1)

2 Chứng minh rằngu1u2 u1009chia hết cho 2018!1009!.

2.1.2 L

i gi

i chi ti

ế

t bình lu

n

Bài tốn Cho dãy số (an),(bn) xác định bởi a0, b0 ∈ R và với mỗi n ≥ 0, các số hạng an+1, bn+1 được xácđịnh theoan, bnbởi hai cách

i) an+1 = 20182019an, bn+1 = 1− 2019an hoặc

ii) an+1 =a2n, bn+1 =an.

1 Chứng minh nếua0 ∈(−1; 1)thì dãy số(an)có giới hạn hữu hạn, tìm giới hạnđó.

2 Giảsửa2018 ≤a0,tìm giá trịlớn tổngS =b1+b2+· · ·+b2018

Lời giải. 1) Nếua0 = 0thì tất cảcác sốhạng dãyđều bằng0và rõ ràngđó giới hạn

cần tìm Nếua0 ̸= 0thì dễthấyan̸= 0,∀n, suy racn =|an|∈(0; 1)và

cn+1 =

2018

2019cnhoặccn+1 =c

2

nvớin≥0

Rõ ràngcn+1 < cn dãy(cn)bịchặn bởi0nên có giới hạn,đặt làL ∈[0; 1) Trong hai

cơng thức trên, ta chon→+∞thìL= 20182019LhoặcL=L2.

Suy raL= 0haylimcn= lim|an|= 0, kéo theoliman =

2) Ta thấy dãy sốxácđịnh theo i) thìbn+1−an+1 = 1−an, cịn xácđịnh theo ii)

(195)

Dođó, trường hợp, ta ln có

bn+1−an+1 ≤1−an⇒bn+1 ≤1 + (an+1−an)

Suy

b1+b2+· · ·+b2018 ≤2018 + (a2018−a2017) +· · ·+ (a1−a0) = 2018 +a2018−a0 ≤2018

Từ ta có giá trịlớn củaSlà2018,đạtđược chẳng hạn khia0 = 0(tất cảcác sốhạng

(an)là0, tất cảcác sốhạng của(bn)là1)

Nhận xét Chúýrằng nếulim|an| =L ̸= 0thì chưa suy rađược dãy(an)hội tụ, có thể

tiếnđếnLhoặc−L.Câu b khơng khó nhưngýtưởng mẻ, khơng dễtiếp cận.

Bài tốn Cho tam giác ABC nhọn, khơng cân, nội tiếp trongđường trịn (O).Mộtđường

trịn (O′)thayđổi, lnđi quaB, C và cắt cạnhAB, AC theo thứ tự ởD, E.GọiD′, E′

lần lượt cácđiểmđối xứng vớiD, Equa trungđiểm cạnhAB, AC

1 Chứng minh trungđiểmD′E′ luôn thuộc mộtđường thẳng cố định.

2 Trên cung nhỏvà cung lớnBCcủa(O), lấy cácđiểmR, Ssao cho(DER),(DES)

tiếp xúc với (O).Phân giác góc BRC, BSC cắt nhauở K.Chứng

minh rằngđường trịn(DEK)ln tiếp xúc vớiđường thẳngBC

Lời giải. 1) Gọi M, N, P, Q trung điểm BC, D′E′, BE′, CD′ thì dễ thấy tứ giác

M P N Q hình bình hành Vì M P, M Qlà đường trung bình nên M P = 12CE′, M Q =

1

2BD′.Ta có

M P M Q =

E′C

BD′ = ADAE = AB

(196)

Tam giácM P QcóMcố định, hai cạnh song song vớiAC, AB có tỷlệkhơngđổi nênđường

trung tuyếnM N sẽcó phương khơngđổi.Đó làđường cố định cần tìm

2) Xét ba đường trịn (DER),(O),(O′) có trục đẳng phương cặp đường tròn

DE, BC,tiếp tuyến của(O)ởRnên chúng phảiđồng quy tạiT.Chứng minh tương tựthì tiếp

tuyến của(O)ởS cũngđi quaT.Từ suy BRCS tứ giácđiều hịa nên SB SC =

RB RC nên

giaođiểmK hai phân giác của∠BRC,∠BSCthuộcBC

Ta có∠T SK=∠T SB+∠BSK =∠BCS+∠CSK =∠T KS nênT S =T K.Suy T K2 =T S2 =T B·T C =T D·T E.

Điều cho thấy(DEK)tiếp xúc vớiBC ởK

Nhận xét Ởcâu a, ta có thểchỉrađường cố định cần tìm làđườngđối trungđỉnhM của

tam giác M B′C′ vớiB′, C′ là trung điểmAC, AB.Thực tỷsố BD′

CE′ ở không thiết

phải bằng AC

AB, toán vẫnđúng thay tỷsố sốbất kỳ.Ởcâu b,đây bài

tốn nhẹnhàng khai thác mơ hình quen thuộc đườngđối song kết hợp với tính chất tứ giác

điều hịa.

Bài tốn Đa thức f(x) hệ số thực được gọi “đẹp” có thể biểu diễn thànhf(x) = (P(x))3−(Q(x))2, trongđóP(x), Q(x)là cácđa thức hệsốthực.

1 Chứng minh rằngf(x) = 2018x2−2019làđa thứcđẹp.

(197)

Lời giải. 1) Ta chứng minh tồn a, b, c ∈ R ứng với P(x) = a2x2 +b, Q(x) = a3x3+cx Thay vào khai triển, ta có

(3a4b−2a3c)x4+ (3a2b2−c2)x2+b3 = 2018x2−2019

Tađưa vềhệphương trình sau ⎧

⎪ ⎨ ⎪ ⎩

3a4b−2a3c= 3a2b2−c2 = 2018

b3 =−2019

Từphương trìnhđầu ta cóab= 23cnên3·94c2−c2 = 2018⇒c2 = 6054 Ta tínhđượcb, cnên

cũng tínhđượca̸= Vậy nênf(x) = 2018x2−2019làđa thứcđẹp.

2) Giảsửtồn tạim, n∈Rvớim̸= 0sao choP3(x)−Q2(x) =mx+n.Đặtu= degP(x), v =

degQ(x)thì dễthấydegP3(x) = 3u,degQ2(x) = 2vvà3u,2v >1nên ta cần có3u= 2v (để

hệsốcao triệt tiêu nhau), từ suy rau < v

Đạo hàm hai vế, ta có3P2(x)·P′(x)−2Q(x)·Q′(x) =m.Dođó2Q(x)·Q′(x) +mchia hết

chođa thứcP2(x).TheođềthìQ2(x) +mx+n =P3(x)cũng chia hết cho P2(x).Dođó, ta

R(x) =Q(x) [2Q(x)·Q′(x) +m]−2Q′(x))Q2(x) +mx+n* =m·Q(x)−2Q′(x)·(mx+n)

chia hết chođa thứcP2(x).Ta xét trường hợp sau:

1 NếuR(x) ≡ 0thìR(x) = m·Q(x)−2Q′(x)·(mx+n) Gọi α ̸= 0là hệsố bậc cao

nhất củaQ(x)thìmα−2vmα= ⇒2v = 1, vơ lý

2 Nếu degR(x) > degR(x) ≤ degQ(x) = v, mà R(x) chia hết cho P(x) nên degR(x)≥2 degP(x) = 2u= 43v.Điều vô lý

Vậy nên không tồn tạiđa thức bậc thỏa mãnđềbài

Nhận xét Ởcâu b, chỉdùngđánh giáđại sốtrực tiếp khó xửlý tốn cách

nhẹnhàng mà khơng sađà vào tính tốn hệsố Việc dùngđạo hàmđã cho ta thêm ràng buộc bổsung chođiều kiệnđềbài, giúp chỉra sựmâu thuẫn nhanh chóng hơn.

Bài tốn Một bảng vng kích thước2019×2019được phủkín hình: chữL,vng

và chữZ nhưbên (có thểxoay hình tùnhưng khơngđượcđè lên nhau).

(198)

2 Chứng minh cách phủ, tađều cần dùng nhất4039hình chữL

Lời giải. 1) Câu trả lời khẳng định Mấu chốt phải lát bảng hình vng kích thước lẻ, cụthểnhưhình bên

Dùng bảng9×9này, ta látđược hình vng2007×2007ởgóc bên trái, cịn lại phần viền

trắng có bềrộng12, ta lát hình chữnhật kích thước2×3(tạo hai khốiL)

2)Đánh sốbảng nhưbên

3 −1 −1 −1 −1 −1 −1 −1

3 −1 −1 −1 −1 −1 −1 −1

3 −1 −1

Dễthấy hình vng2×2hoặc hình chữZ sẽphủlên ba số−1và số3nên tổng

sốtrong hình bằng0.Cịn hình chữLsẽphủlên số3và hai số−1, ba số−1nên

tổng sốtrong hình chữLlà1hoặc−3.Gọix, ylần lượt sốlượng hình chữLmà tổng

ba sốtrênđó là1,−3.Ta tínhđược tổng sốcủa bảng

1010×(3×1010−1×1009) + 1009×(−1×2019) = 4039

Suy rax−3y = 4039 Dođó, tổng sốhình chữLlàx+y≥x−3y= 4039.Ta cóđpcm

Nhận xét Ta thấy với2khối chữL(trimino) vàk(k∈Z+)khối chữZ (tetromino), ta có

(199)

Tuy nhiên, cần phải có hình chữnhật có kích thước lẻthì câu a có thểgiải quyếtđược. Người ta cịn chứng minhđược bảng vng kích thướcn×nvớinkhơng chia hết cho3,

khi bỏ bất kỳthìđều látđược trimino.

câu b, việcđánh sốkhéo léo như trên khiến cho việcđánh giá hiệu quả và nhanh gọn, nhất việc tổng sốtrên hai hình bằng0.

Bài tốn lát gạch dùng trimino tetromino kinhđiển,được nghiên cứu nhiều việc dùng tô màu,đánh trọng sốlà cách tiếp cận phổbiếnđểxửlý Ngoài ra,đối với domino (hình2×1), người ta xét tốnđếm sốcách lát cho hìnhđặc biệt cho trước. Bài tốn Cho sốthựcx, y, z thỏa mãnđồng thời cácđiều kiện0< z ≤ y ≤ x ≤ 8 3x+ 4y ≥max;xy;12xyz−8z<

1 Chứng minh rằng

x +

6

y +

3

z ≥3

2 Tìm giá trịlớn biểu thứcP =x5+y5 +z5.

Lời giải. 1) Theo giảthiết thì3x+ 4y+ 8z ≥

2xyz ⇒ yz + zx+ xy ≥

2 Áp dụng bấtđẳng

thức(a+b+c)2 ≥3(ab+bc+ca)với mọia, b, c∈R, ta có

% x + y + z &2 ≥3 % 48 xy + 18 yz + 24 zx & = 18 % yz + zx + xy &

≥9

Dođó

x +

6

y +

3

z ≥3,đẳng thức xảy khix= 8, y = 6, z =

2) Ta có

x ≥1,

8

x+

6

y ≥2,

8

x+

6

y+

3

z ≥3nên ta dự đoán cực trịxảy khix= 8, y = 6, z=

Sửdụng khai triển Abel, ta có

85+ 65+ 35 =x5 %

85 x5

&

+y5 %

65 y5

&

+z5 %

35 z5

&

= (x5 −y5)

%

85 x5

&

+ (y5−z5)

%

85 x5 +

65 y5

&

+z5 %

85 x5 +

65 y5 +

35 z5

&

≥(x5−y5)

%

8

x &5

+ (y5−z5)

% x+ y &5 ·

16+z

5 % x + y + z &5 · 34

≥x5−y5+ 2(y5−z5) + 3z5 =x5+y5+z5

Vậy giá trịlớn củaP là85+ 65+ 35 = 40787,đạtđược khix= 8, y = 6, z = 3.

Nhận xét Tổng quát củaý2 là a x ≥1,

a x +

b y ≥2,

a x +

b y +

c

z ≥3x≥y ≥z >0, a ≥b ≥

c >0thìxk+yk+zk≤ak+bk+ckvới mọi sốnguyên dươngk.Bài tốn khơng mấy xa lạvà

(200)

Bài tốn Cho tam giác ABC nhọn khơng cân nội tiếp đường trịn (O) M, N lần lượt

là trungđiểmAB, AC Dlà trungđiểm cung lớnBC của(O).GiảsửK là mộtđiểm nằm

trong tam giác thỏa mãn∠KAB = 2∠KBA, ∠KAC = 2∠KCA.

1 a) Chứng minh rằngKA=KD

2 GiảsửADcắtBCTT Mcắt(BM C)R, T N cắt(BN C)S.GọiP là giaođiểm

củaKB OM, Qlà giaođiểm của KC ON.Chứng minh trụcđẳng phương

của haiđường tròn(T QR),(T P S)đi quaO

Lời giải. 1) TrênAB, AC lấy cácđiểmE, F choKA = KD =KE =KF theo tính

chất củaK,ta có∠KAE = 2∠KBA=∠KEAnênEK =EB Tương tựthìF K =F C

Mặt khácDB =DCvà∠DBE =∠DCF nên hai tam giácDBE, DCF nhau, kéo theo

∠DEA=∠DF AnênD∈(AEF) MàK tâm của(AEF)nên suy raKA =KD

2) Ta cóP ∈OM nênP A=P B, kéo theoAP phân giác∠KAB, tương tựthìAQlà phân

giác∠KAC.Theo tính chấtđường phân giác P K

P B = AK AB,

QK QC =

AK AC,

T B T C =

AB AC

Từ đó, theo định lý Menelaus tam giác KBC, dễ thấy P, Q, T thẳng hàng Do

∠P AK =∠P BA,∠QAK =∠QCAnênđường thẳngKAlà tiếp tuyến chung haiđường

tròn(ABP),(ACQ); suy raKA2 =KP ·KB =KQ·KC vàBP QC nội tiếp.

GọiLlà giaođiểm củaT Ovà(BOC)thìT L·T O =T B·T C.Xét phép nghịchđảoΩtâmT,

phương tíchPT /(O)

Ω:O →L, N →S, Q→P

MàO, N, Qthẳng hàng nên(SP L)đi qua tâm nghịchđảoT hay(SP T)đi quaL.Tương tựthì

(SQT)cũngđi quaLnên trụcđẳng phương haiđường trịn(SP T),(SQT)chính làT Lđi

Ngày đăng: 08/02/2021, 07:41

TÀI LIỆU CÙNG NGƯỜI DÙNG

TÀI LIỆU LIÊN QUAN

w